MATH1-ALA: Abgaben hinzugefügt

This commit is contained in:
Jim Martens 2014-12-10 23:04:13 +01:00
parent 9ca68cda89
commit cd9612c504
12 changed files with 3095 additions and 0 deletions

View File

@ -0,0 +1,165 @@
\documentclass[10pt,a4paper,oneside,ngerman,numbers=noenddot]{scrartcl}
\usepackage[T1]{fontenc}
\usepackage[utf8]{inputenc}
\usepackage[ngerman]{babel}
\usepackage{amsmath}
\usepackage{amsfonts}
\usepackage{amssymb}
\usepackage{paralist}
\usepackage{gauss}
\usepackage[locale=DE,exponent-product=\cdot,detect-all]{siunitx}
\usepackage{tikz}
\usetikzlibrary{matrix,fadings,calc,positioning,decorations.pathreplacing,arrows,decorations.markings}
\usepackage{polynom}
\polyset{style=C, div=:,vars=x}
\pagenumbering{arabic}
\def\thesection{\arabic{section})}
\def\thesubsection{\alph{subsection})}
\def\thesubsubsection{(\roman{subsubsection})}
\makeatletter
\renewcommand*\env@matrix[1][*\c@MaxMatrixCols c]{%
\hskip -\arraycolsep
\let\@ifnextchar\new@ifnextchar
\array{#1}}
\makeatother
\begin{document}
\author{Jim Martens}
\title{Hausaufgaben zum 11. April}
\maketitle
\section{} %1
\begin{equation*}
\frac{3}{x+5} \geq 3
\end{equation*}
1. Fall $x > -5$:\\
\begin{alignat*}{3}
&& \frac{3}{x+5} &\geq & 3 \\
\Leftrightarrow && 3 &\geq & 3(x+5) \\
\Leftrightarrow && 3 &\geq & 3x + 15 \\
\Leftrightarrow && -12 &\geq & 3x \\
\Leftrightarrow && -4 &\geq & x
\end{alignat*}
\\
2. Fall $x < -5$:\\
\begin{alignat*}{3}
&& \frac{3}{x+5} &\geq & 3 \\
\Leftrightarrow && 3 &\leq & 3(x+5) \\
\Leftrightarrow && 3 &\leq & 3x + 15 \\
\Leftrightarrow && -12 &\leq & 3x \\
\Leftrightarrow && -4 &\leq & x
\end{alignat*}
\\
$L = [-4]$
\section{} %2
\begin{equation*}
|3x-4| \geq 2
\end{equation*}
1. Fall $x \geq \frac{4}{3}$:\\
\begin{alignat*}{3}
&& |3x-4| &\geq & 2 \\
\Leftrightarrow && 3x-4 &\geq & 2 \\
\Leftrightarrow && 3x &\geq & 6 \\
\Leftrightarrow && x &\geq & 2
\end{alignat*}
\\
2. Fall $x < \frac{4}{3}$:\\
\begin{alignat*}{3}
&& |3x-4| &\geq & 2 \\
\Leftrightarrow && -(3x-4) &\geq & 2 \\
\Leftrightarrow && -3x + 4 &\geq & 2 \\
\Leftrightarrow && -3x &\geq & -2 \\
\Leftrightarrow && x &\leq & \frac{2}{3}
\end{alignat*}
\\
$L = (-\infty,\frac{2}{3}] \cup [2,\infty)$
\section{} %3
\subsection{} %a
\begin{alignat*}{3}
&& |a_{n} - a| &=& |\frac{2n-1}{n+3} - 2| \\
\Leftrightarrow && &=& |\frac{2n-1}{n+3} - \frac{2(n+3)}{n+3}| \\
\Leftrightarrow && &=& |\frac{2n-1 - 2n - 6}{n+3}| \\
\Leftrightarrow && &=& |\frac{-7}{n+3}| \\
\Leftrightarrow && &=& \frac{7}{n+3}
\end{alignat*}
\subsection{} %b
Es sei $\varepsilon > 0$. Aufgrund von a) gilt:\\
\begin{alignat*}{3}
&& |a_{n} - a| &<& \varepsilon \label{eq:1}\tag{1}\\
\Leftrightarrow && |a_{n} - a| = |\frac{-7}{n+3}| = \frac{7}{n+3} &<& \varepsilon \\
\Leftrightarrow && -7 &<& \varepsilon (n+3) \\
\Leftrightarrow && \frac{-7}{\varepsilon} &<& n+3 \\
\Leftrightarrow && \frac{-7}{\varepsilon} - 3 &<& n
\end{alignat*}
\\
Wählt man $N > \frac{-7}{\varepsilon} - 3$, so ergibt sich aus \eqref{eq:1}, dass $|a_{n} - a| < \varepsilon$ für alle $n \geq N$ gilt. Das zeigt $(a_{n}) \rightarrow a = 2$.
\subsection{} %c
Es sei $\varepsilon = \frac{1}{10}$:\\
\begin{alignat*}{3}
&& \frac{-7}{\frac{1}{10}} - 3 &<& n \\
\Leftrightarrow && -70 - 3 &<& n \\
\Leftrightarrow && -73 &<& n
\end{alignat*}
Wählt man $N = -72$, so ergibt sich aus \eqref{eq:1}, dass $|a_{n} - a| < \varepsilon$ für alle $n \geq N$ gilt.\\
\\
Es sei $\varepsilon = \frac{1}{100}$:\\
\begin{alignat*}{3}
&& \frac{-7}{\frac{1}{100}} - 3 &<& n \\
\Leftrightarrow && -700 - 3 &<& n \\
\Leftrightarrow && -703 &<& n
\end{alignat*}
Wählt man $N = -702$, so ergibt sich aus \eqref{eq:1}, dass $|a_{n} - a| < \varepsilon$ für alle $n \geq N$ gilt.\\
\\
Es sei $\varepsilon = \frac{1}{100000}$:\\
\begin{alignat*}{3}
&& \frac{-7}{\frac{1}{100000}} - 3 &<& n \\
\Leftrightarrow && -700000 - 3 &<& n \\
\Leftrightarrow && -700003 &<& n
\end{alignat*}
Wählt man $N = -700002$, so ergibt sich aus \eqref{eq:1}, dass $|a_{n} - a| < \varepsilon$ für alle $n \geq N$ gilt.
\section{} %4
\textbf{Behauptung:} Die folgende Aussage gilt für alle $n \in \mathbb{N}$:\\
\begin{equation*}
0 \leq a_{n} < \frac{1}{2} \label{eq:2}\tag{2}
\end{equation*}\\
Die Folge ($a_{n}$) sei rekursiv definiert durch \\
\begin{alignat*}{2}
a_{1} &=& \frac{2}{5} \label{eq:3}\tag{3}\\
a_{n+1} &=& a_{n}^{2} + \frac{1}{4} \label{eq:4}\tag{4}
\end{alignat*}
\textbf{Beweis:} Durch vollständige Induktion.\\
Mit $A(n)$ sei die Aussage \eqref{eq:2} bezeichnet.\\\\
\underline{Induktionsanfang:} \\
$A(1)$ ist richtig, da die Aussage \eqref{eq:2} für \eqref{eq:3} wie folgt gilt:
\begin{alignat*}{2}
0 \leq \frac{2}{5} = \frac{4}{10} < \frac{1}{2} = \frac{5}{10}
\end{alignat*}\\
\underline{Induktionsannahme:}\\
Die Aussage \eqref{eq:2} gilt für ein beliebig fest gewähltes $n \in \mathbb{N}$.\\\\
\underline{Zu zeigen:}\\
$A(n+1)$ gilt, d. h. Folgendes gilt für die Aussage \eqref{eq:4}:\\
\begin{equation*}
0 \leq a_{n+1} < \frac{1}{2} \label{eq:5}\tag{5}
\end{equation*}
\underline{Induktionsschluss:}\\
Aus \eqref{eq:5} folgt für $0 \leq a_{n+1}$ Folgendes:\\
\begin{alignat*}{3}
&& 0 &\leq & a_{n}^{2} + \frac{1}{4}
\end{alignat*}
Diese Aussage gilt, da $\frac{1}{4}$ auf triviale Weise die Aussage erfüllt und $a_{n}^{2}$ immer positiv oder gleich Null sein muss, da eine beliebige Zahl zum Quadrat immer größer gleich Null ist.\\
Für $a_{n+1} < \frac{1}{2}$ ergibt sich Folgendes:\\
\begin{alignat*}{5}
&& a_{n}^{2} + \frac{1}{4} &<& \frac{1}{2} && \;&|& -\frac{1}{4} \\
\Leftrightarrow && a_{n}^{2} &<& \frac{1}{4} && && \\
\Leftrightarrow && a_{n} \cdot a_{n} &<& \frac{1}{4} && &&
\end{alignat*}
Aufgrund der Induktionsannahme gilt $a_{n} < \frac{1}{2}$. Daher ist das Quadrat von $a_{n}$ auf jeden Fall kleiner als $\frac{1}{4}$.\\
Nach dem Induktionsprinzip folgt aus dem Induktionsanfang und dem Induktionsschluss die Behauptung. \hfill $\Box$\\
Es ist zu zeigen, dass $a_{n+1} \geq a_{n}$ für alle $n \in \mathbb{N}$ gilt. Es ergibt sich Folgendes:\\
\begin{alignat*}{5}
&& a_{n+1} &\geq & a_{n} && && \\
\Leftrightarrow && a_{n}^{2} + \frac{1}{4} &\geq & a_{n} && \;&|& -a_{n} \\
\Leftrightarrow && a_{n}^{2} - a_{n} + \frac{1}{4} &\geq & 0 && \;&|& \text{Binomische Formel erzeugen} \\
\Leftrightarrow && (a_{n} - \frac{1}{2})^{2} &\geq & 0 && &&
\end{alignat*}
Diese Aussage gilt, da ein Quadrat einer beliebigen Zahl immer größer gleich Null ist.
\end{document}

View File

@ -0,0 +1,253 @@
\documentclass[10pt,a4paper,oneside,ngerman,numbers=noenddot]{scrartcl}
\usepackage[T1]{fontenc}
\usepackage[utf8]{inputenc}
\usepackage[ngerman]{babel}
\usepackage{amsmath}
\usepackage{amsfonts}
\usepackage{amssymb}
\usepackage{paralist}
\usepackage{gauss}
\usepackage{pgfplots}
\usepackage[locale=DE,exponent-product=\cdot,detect-all]{siunitx}
\usepackage{tikz}
\usetikzlibrary{matrix,fadings,calc,positioning,decorations.pathreplacing,arrows,decorations.markings}
\usepackage{polynom}
\polyset{style=C, div=:,vars=x}
\pgfplotsset{compat=1.8}
\pagenumbering{arabic}
\def\thesection{\arabic{section})}
\def\thesubsection{\alph{subsection})}
\def\thesubsubsection{(\roman{subsubsection})}
\makeatletter
\renewcommand*\env@matrix[1][*\c@MaxMatrixCols c]{%
\hskip -\arraycolsep
\let\@ifnextchar\new@ifnextchar
\array{#1}}
\makeatother
\begin{document}
\author{Jim Martens (6420323)}
\title{Hausaufgaben zum 27. Juni}
\maketitle
\section{} %1
\subsubsection{} %i
\begin{alignat*}{2}
f(x,y) &=& 2x^{2}y^{2} - 3xy + 4x + 2 \\
f_{x} &=& 4xy^{2} - 3y + 4 \\
f_{y} &=& 4x^{2}y - 3x
\end{alignat*}
\subsubsection{} %ii
\begin{alignat*}{2}
f(x,y) &=& \cos(x^{2}y) \cdot e^{xy} \\
f_{x} &=& -\sin(x^{2}y) \cdot 2xy + \cos(x^{2}y) \cdot e^{xy} \cdot y \\
f_{y} &=& -\sin(x^{2}y) \cdot x^{2} + \cos(x^{2}y) \cdot e^{xy} \cdot x \\
\end{alignat*}
\subsubsection{} %iii
\begin{alignat*}{2}
f(x,y) &=& \frac{\sin x + \cos y}{x^{2} + y^{2}} \\
f_{x} &=& \frac{\cos x \cdot (x^{2} + y^{2}) - (\sin x + \cos y) \cdot 2x}{(x^{2} + y^{2})^{2}} \\
f_{y} &=& \frac{-\sin y \cdot (x^{2} + y^{2}) - (\sin x + \cos y) \cdot 2y}{(x^{2} + y^{2})^{2}}
\end{alignat*}
\subsubsection{} %iv
\begin{alignat*}{2}
f(x,y) &=& \sqrt{1 - x^{2} - y^{2}} = (1 - x^{2} - y^{2})^{\frac{1}{2}} \\
f_{x} &=& \frac{1}{2}(1 - x^{2} - y^{2})^{-\frac{1}{2}} \cdot (-2x) \\
f_{y} &=& \frac{1}{2}(1 - x^{2} - y^{2})^{-\frac{1}{2}} \cdot (-2y)
\end{alignat*}
\section{} %2
\begin{alignat*}{2}
f(x,y) &=& x^{2}y^{3} + y \cdot e^{x^{2}y} \\
f_{x} &=& 2xy^{3} + y \cdot e^{x^{2}y} \cdot 2xy\\
&=& 2xy^{3} + e^{x^{2}y} \cdot 2xy^{2} \\
f_{y} &=& 3x^{2}y^{2} + e^{x^{2}y} + y \cdot e^{x^{2}y} \cdot x^{2} \\
f_{xx} &=& 2y^{3} + e^{x^{2}y} \cdot 2xy \cdot 2xy^{2} + e^{x^{2}y} \cdot 2y^{2}\\
&=& 2y^{3} + e^{x^{2}y} \cdot 4x^{2}y^{3} + e^{x^{2}y} \cdot 2y^{2} \\
f_{yx} &=& 6xy^{2} + e^{x^{2}y} \cdot x^{2} \cdot 2xy^{2} + e^{x^{2}y} \cdot 4xy\\
&=& 6xy^{2} + e^{x^{2}y} \cdot 2x^{3}y^{2} + e^{x^{2}y} \cdot 4xy \\
f_{xy} &=& 6xy^{2} + e^{x^{2}y} \cdot 2xy + y \cdot (e^{x^{2}y} \cdot 2xy \cdot x^{2} + e^{x^{2}y} \cdot 2x)\\
&=& 6xy^{2} + e^{x^{2}y} \cdot 2x^{3}y^{2} + e^{x^{2}y} \cdot 4xy \\
f_{yy} &=& 6x^{2}y + e^{x^{2}y} \cdot x^{2} + e^{x^{2}y} \cdot x^{2} + y \cdot e^{x^{2}y} \cdot x^{2} \cdot x^{2} \\
&=& 6x^{2}y + e^{x^{2}y} \cdot 2x^{2} + e^{x^{2}y} \cdot x^{4}y
\end{alignat*}
\section{} %3
\setcounter{subsubsection}{0}
\subsubsection{} %i
\begin{alignat*}{2}
f(x,y) &=& 2x^{2} + y^{2} - 2xy -2x -4y + 5 \\
I f_{x} &=& 4x - 2y - 2 = 0 \\
II f_{y} &=& 2y - 2x - 4 = 0 \\
f_{xx} &=& 4 \\
f_{xy} &=& - 2 \\
f_{yx} &=& - 2 \\
f_{yy} &=& 2 \\
I + II &=& 2x -6 = 0 \\
&\Leftrightarrow & 2x = 6 \\
&\Leftrightarrow & x = 3 \\
\intertext{in II einsetzen}
&\Rightarrow & 2y - 2 \cdot 3 -4 = 0 \\
&\Leftrightarrow & 2y - 6 - 4 = 0 \\
&\Leftrightarrow & 2y - 10 = 0 \\
&\Leftrightarrow & 2y = 10 \\
&\Leftrightarrow & y = 5 \\
\intertext{Einsetzen von beiden Werten in I}
&\Rightarrow & 4 \cdot 3 - 2 \cdot 5 - 2 = 0 \\
&\Leftrightarrow & 12 - 10 - 2 = 0 \\
&\Leftrightarrow & 0 = 0 \\
\intertext{Die einzige kritische Stelle befindet sich an (3,5). Aufstellen der Hesse-Matrix}
H &=& \begin{pmatrix} 4 & -2 \\
-2 & 2 \end{pmatrix} \\
\bigtriangleup_{1} &=& 4 > 0 \\
\bigtriangleup_{2} &=& 4 > 0 \\
\intertext{Die Hesse-Matrix ist positiv definit und damit befindet sich an der kritischen Stelle ein lokales Minimum.}
\end{alignat*}
\subsubsection{} %ii
\begin{alignat*}{2}
f(x,y) &=& x^{2} + 2y^{2} - 3xy -x -y +7 \\
I f_{x} &=& 2x - 3y - 1 = 0 \\
II f_{y} &=& 4y - 3x - 1 = 0 \\
f_{xx} &=& 2 \\
f_{xy} &=& -3 \\
f_{yx} &=& -3 \\
f_{yy} &=& 4 \\
II + I &=& -x + y -2 = 0 \\
&\Leftrightarrow & y = x + 2 \\
\intertext{Einsetzen in I}
&\Rightarrow & 2x - 3(x+2) - 1 = 0 \\
&\Leftrightarrow & 2x - 3x - 6 - 1 = 0 \\
&\Leftrightarrow & -x - 7 = 0 \\
&\Leftrightarrow & x = -7 \\
\intertext{Einsetzen in I}
&\Rightarrow & 2 \cdot (-7) - 3y - 1 = 0 \\
&\Leftrightarrow & -14 - 3y - 1 = 0 \\
&\Leftrightarrow & 3y = -15 \\
&\Leftrightarrow & y = -5 \\
\intertext{Einsetzen beider Werte in II}
&\Rightarrow & 4 \cdot (-5) - 3 \cdot (-7) - 1 = 0 \\
&\Leftrightarrow & -20 + 21 - 1 = 0 \\
&\Leftrightarrow & 0 = 0 \\
\intertext{Die einzige kritische Stelle befindet sich an (-7, -5). Aufstellen der Hesse-Matrix}
H &=& \begin{pmatrix} 2 & -3 \\
-3 & 4 \end{pmatrix} \\
\bigtriangleup_{1} &=& 2 > 0 \\
\bigtriangleup_{2} &=& -1 < 0 \\
\intertext{Die Hesse-Matrix ist damit weder positiv noch negativ definit und daher infinit. Daher liegt an der kritischen Stelle kein lokales Extremum vor.}
\end{alignat*}
\subsubsection{} %iii
\begin{alignat*}{2}
f(x,y) &=& 2x^{3} + y^{3} - 12x -27y +2 \\
I f_{x} &=& 6x^{2} -12 = 0\\
II f_{y} &=& 3y^{2} - 27 = 0\\
f_{xx} &=& 12x \\
f_{xy} &=& 0 \\
f_{yx} &=& 0 \\
f_{yy} &=& 6y \\
I &\Rightarrow & 6x^{2} - 12 = 0 \\
&\Leftrightarrow & 6x^{2} = 12 \\
&\Leftrightarrow & x^{2} = 2 \\
&\Rightarrow & x_{1} = \sqrt{2} \\
&\Rightarrow & x_{2} = -\sqrt{2} \\
I + II &=& 6x^{2} + 3y^{2} - 39 = 0 \\
\intertext{Einsetzen von den x-Werten und berechnen von y}
x_{1} &\Rightarrow & 6 \cdot 2 + 3y^{2} -39 = 0 \\
&\Leftrightarrow & 3y^{2} - 27 = 0 \\
&\Leftrightarrow & 3y^{2} = 27 \\
&\Leftrightarrow & y^{2} = 9 \\
&\Leftrightarrow & y \pm 3 \\
x_{2} &\Rightarrow & 6 \cdot 2 + 3y^{2} - 39 = 0 \\
&\Leftrightarrow & 3y^{2} = 27 \\
&\Leftrightarrow & y \pm 3 \\
\intertext{Es gibt also vier kritische Stellen: $(\sqrt{2}, 3), (\sqrt{2}, -3), (-\sqrt{2}, 3)$ und $(-\sqrt{2},-3)$. Aufstellen der Hesse-Matrix}
H &=& \begin{pmatrix}12x & 0 \\
0 & 6y\end{pmatrix} \\
\intertext{Berechnen der Definitheit für erste kritische Stelle:}
\bigtriangleup_{1} &=& 12 \cdot \sqrt{2} > 0 \\
\bigtriangleup_{2} &=& 12 \cdot \sqrt{2} \cdot 18 \\
&=& 216 \cdot \sqrt{2} > 0 \\
\intertext{Die Hesse-Matrix ist für die erste kritische Stelle positiv definit. An der ersten kritischen Stelle liegt also ein lokales Minimum vor. Berechnen der Definitheit für die zweite kritische Stelle:}
\bigtriangleup_{1} &=& 12 \cdot \sqrt{2} > 0 \\
\bigtriangleup_{2} &=& 12 \cdot \sqrt{2} \cdot (-18) \\
&=& -216 \cdot \sqrt{2} < 0 \\
\intertext{Die Hesse-Matrix ist für die zweite kritische Stelle infinit. An der zweiten kritischen Stelle liegt also kein lokales Extremum vor. Berechnen der Definitheit für die dritte kritische Stelle:}
\bigtriangleup_{1} &=& -12 \cdot \sqrt{2} < 0 \\
\bigtriangleup_{2} &=& -12 \cdot \sqrt{2} \cdot 18 \\
&=& -216 \cdot \sqrt{2} < 0 \\
\intertext{Die Hesse-Matrix ist für die dritte kritische Stelle infinit. An der dritten kritischen Stelle liegt also kein lokales Extremum vor. Berechnen der Definitheit für die vierte kritische Stelle:}
\bigtriangleup_{1} &=& -12 \cdot \sqrt{2} < 0 \\
\bigtriangleup_{2} &=& -12 \cdot \sqrt{2} \cdot (-18) \\
&=& 216 \cdot \sqrt{2} > 0 \\
\intertext{Die Hesse-Matrix ist für die vierte kritische Stelle negativ definit. An der vierten kritischen Stelle liegt also ein lokales Maximum vor.}
\end{alignat*}
\section{} %4
\subsection{} %a
\begin{alignat*}{2}
C(x,y) &=& 0.01x^{2} + 0.02xy + 0.16y^{2} + 5x + 6y + 120 \\
&=& \frac{1}{100}x^{2} + \frac{1}{50}xy + \frac{4}{25}y^{2} + 5x + 6y + 120 \\
\intertext{Aufstellen der Gewinnfunktion}
G(x,y) &=& 12x + 28y - C(x,y) \\
&=& 12x + 28y - \frac{1}{100}x^{2} - \frac{1}{50}xy - \frac{4}{25}y^{2} - 5x - 6y - 120 \\
&=& -\frac{1}{100}x^{2} - \frac{4}{25}y^{2} - \frac{1}{50}xy + 7x + 22y - 120 \\
I\, G_{x} &=& -\frac{1}{50}x - \frac{1}{50}y + 7 = 0 \\
II\, G_{y} &=& -\frac{8}{25}y - \frac{1}{50}x + 22 = 0 \\
G_{xx} &=& -\frac{1}{50} \\
G_{xy} &=& 0 \\
G_{yx} &=& 0 \\
G_{yy} &=& -\frac{8}{25} \\
I &\Rightarrow & -\frac{1}{50}x - \frac{1}{50}y + 7 = 0 \\
&\Leftrightarrow & \frac{1}{50}x = 7 - \frac{1}{50}y \\
&\Leftrightarrow & x = 350 - y \\
\intertext{Einsetzen in II}
II &\Rightarrow & -\frac{8}{25}y - \frac{1}{50} (350-y) + 22 = 0 \\
&\Leftrightarrow & \frac{8}{25}y = 22 - \frac{1}{50} (350 - y) \\
&\Leftrightarrow & \frac{8}{25}y = 22 - 7 + \frac{1}{50}y \\
&\Leftrightarrow & \frac{15}{50}y = 15 \\
&\Leftrightarrow & \frac{3}{10}y = 15 \\
&\Leftrightarrow & y = 50 \\
\intertext{Einsetzen in I}
&\Rightarrow & x = 350 - 50 = 300
\intertext{Die einige kritische Stelle von G(x,y) befindet sich an $\left(300, 50\right)$. Aufstellen der Hesse-Matrix:}
H &=& \begin{pmatrix}-\frac{1}{50} & 0 \\
0 & -\frac{8}{25} \end{pmatrix} \\
\bigtriangleup_{1} &=& -\frac{1}{50} < 0 \\
\bigtriangleup_{2} &=& \frac{4}{625} > 0 \\
\intertext{Die Hesse-Matrix ist an der kritischen Stelle negativ definit. An der kritischen Stelle befindet sich daher ein Maximum. Der höchste Gewinn ist demnach mit 300 Einheiten des Gutes A und 50 Einheiten des Gutes B zu erreichen.}
\end{alignat*}
\subsection{} %b
\begin{alignat*}{2}
G(x,y) &=& -\frac{1}{100}x^{2} - \frac{4}{25}y^{2} - \frac{1}{50}xy + 7x + 22y - 120 \\
n(x,y) &=& x + 2y = 320 \\
&\Leftrightarrow & x= 320 - 2y \\
\intertext{Einsetzen in G(x,y)}
G(y) &=& -\frac{1}{100} \cdot (320-2y)^{2} - \frac{4}{25}y^{2} - \frac{1}{50} \cdot (320 - 2y)y + 7(320-2y) + 22y - 120 \\
&=& -\frac{1}{100} \cdot (102400 -1280y + 4y^{2}) - \frac{4}{25}y^{2} - \frac{1}{50} \cdot (320y -2y^{2}) + 2240 - 14y + 22y - 120 \\
&=& -1024 + \frac{128}{10}y - \frac{1}{25}y^{2} - \frac{4}{25}y^{2} - \frac{32}{5}y + \frac{1}{25}y^{2} + 8y + 2120 \\
&=& \frac{64}{5}y - \frac{5}{25}y^{2} - \frac{32}{5} + \frac{1}{25}y^{2} + 1096 + 8y \\
&=& -\frac{4}{25}y^{2} + \frac{72}{5}y + 1096 \\
G'(y) &=& -\frac{8}{25}y + \frac{72}{5} = 0 \\
&\Leftrightarrow & \frac{8}{25}y = \frac{72}{5} \\
&\Leftrightarrow & y = 45 \\
G''(y) &=& -\frac{8}{25} < 0 \\
\intertext{Unter der Nebenbedingung n(x,y) gibt es ein lokales Maximum für 45 Einheiten von Gut B.
Einsetzen von y in die Nebenbedingung:}
n(x) &=& x + 2 \cdot 45 = 320 \\
&\Leftrightarrow & x + 90 = 320 \\
&\Leftrightarrow & x = 230
\intertext{Die optimalen Mengen des Outputs liegen bei 230 Einheiten von Gut A und 45 Einheiten von Gut B.}
\end{alignat*}
\subsection{} %c
Berechnen des maximalen Gewinns für Fall a)\\
\begin{alignat*}{2}
G(300,50) &=& -\frac{1}{100} \cdot 300^{2} - \frac{4}{25} \cdot 50^{2} - \frac{1}{50} \cdot 300 \cdot 50 + 7 \cdot 300 + 22 \cdot 50 - 120 \\
&=& -900 - \frac{4}{25} \cdot 2500 - 300 + 2100 + 1100 - 120 \\
&=& 1880 - 400 \\
&=& 1480 \\
\intertext{Der maximale Gewinn im Fall a) beträgt 1480 Geldeinheiten.}
\end{alignat*}\\
Berechnen des maximalen Gewinns für Fall b)\\
\begin{alignat*}{2}
G(230,45) &=& -\frac{1}{100} \cdot 230^{2} - \frac{4}{25} \cdot 45^{2} - \frac{1}{50} \cdot 230 \cdot 45 + 7 \cdot 230 + 22 \cdot 45 - 120 \\
&=& -529 - \frac{4}{25} \cdot 2025 - \frac{1}{50} \cdot 10350 + 1610 + 990 - 120 \\
&=& 1951 - 324 - 207 \\
&=& 1420
\intertext{Der maximale Gewinn im Fall b) beträgt 1420 Geldeinheiten.}
\end{alignat*}
\end{document}

View File

@ -0,0 +1,222 @@
\documentclass[10pt,a4paper,oneside,ngerman,numbers=noenddot]{scrartcl}
\usepackage[T1]{fontenc}
\usepackage[utf8]{inputenc}
\usepackage[ngerman]{babel}
\usepackage{amsmath}
\usepackage{amsfonts}
\usepackage{amssymb}
\usepackage{paralist}
\usepackage{gauss}
\usepackage{pgfplots}
\usepackage[locale=DE,exponent-product=\cdot,detect-all]{siunitx}
\usepackage{tikz}
\usetikzlibrary{matrix,fadings,calc,positioning,decorations.pathreplacing,arrows,decorations.markings}
\usepackage{polynom}
\polyset{style=C, div=:,vars=x}
\pgfplotsset{compat=1.8}
\pagenumbering{arabic}
\def\thesection{\arabic{section})}
\def\thesubsection{\alph{subsection})}
\def\thesubsubsection{(\roman{subsubsection})}
\makeatletter
\renewcommand*\env@matrix[1][*\c@MaxMatrixCols c]{%
\hskip -\arraycolsep
\let\@ifnextchar\new@ifnextchar
\array{#1}}
\makeatother
\begin{document}
\author{Jim Martens (6420323)}
\title{Hausaufgaben zum 4. Juli}
\maketitle
\section{} %1
\subsection{} %a
\begin{alignat*}{2}
f(x,y,z) &=& 2x^{2} + y^{2} + 4z^{2} - 2yz - 2x - 6y + 8 \\
I f_{x} &=& 4x - 2 = 0\\
II f_{y} &=& 2y - 7z - 6 = 0\\
III f_{z} &=& 8z - 2y = 0 \\
I &\Rightarrow & 4x - 2 = 0 \\
&\Leftrightarrow & 4x = 2 \\
&\Leftrightarrow & x = \frac{1}{2} \\
III &\Rightarrow & 8z - 2y = 0 \\
&\Leftrightarrow & 8z = 2y \\
&\Leftrightarrow & 4z = y \\
II &\Rightarrow & 2 \cdot 4z - 7z - 6 = 0 \\
&\Leftrightarrow & 8z - 7z = 6 \\
&\Leftrightarrow & z = 6 \\
III &\Rightarrow & 4 \cdot 6 = y \\
&\Leftrightarrow & 24 = y \\
\intertext{Die einzige kritische Stelle befindet sich an ($\frac{1}{2}, 24, 6$).}
f_{xx} &=& 4 \\
f_{yx} &=& 0 \\
f_{zx} &=& 0 \\
f_{xy} &=& 0 \\
f_{yy} &=& 2 \\
f_{zy} &=& -7 \\
f_{xz} &=& 0 \\
f_{yz} &=& -2 \\
f_{zz} &=& 8 \\
\intertext{Aufstellen der Hesse-Matrix}
H &=& \begin{pmatrix}4 & 0 & 0 \\
0 & 2 & -7 \\
0 & -2 & 8\end{pmatrix} \\
\bigtriangleup_{1} &=& 4 > 0\\
\bigtriangleup_{2} &=& 8 > 0\\
\bigtriangleup_{3} &=& 64 - 56 = 8 > 0\\
\intertext{Die Hesse-Matrix ist positiv definit. Daher liegt an der kritischen Stelle ein Minimum vor.}
\end{alignat*}
\subsection{} %b
\begin{alignat*}{2}
grad\,f(1,1,1) &=& (4 - 2, 2 - 7 - 6, 8 - 2) \\
&=& (2, -11, 6) \\
||grad\,f(1,1,1)|| &=& \sqrt{2^{2} + 11^{2} + 6^{2}} \\
&=& \sqrt{4 + 121 + 36} \\
&=& \sqrt{161} \\
\intertext{In Richtung von (2, -11, 6) steigt die Temperatur von (1,1,1) aus am stärksten an. In Richtung (-2, 11, -6) sinkt die Temperatur am stärksten. Die Größe des stärksten Anstiegs beträgt $\sqrt{161}$.}
\end{alignat*}
\section{} %2
\subsection{} %a
\begin{alignat*}{2}
A &=& \begin{pmatrix}-i & -1 \\
3 & i \end{pmatrix} \\
B &=& \begin{pmatrix} i \\
1 + i\end{pmatrix} \\
C &=& \begin{pmatrix}-i & i \end{pmatrix} \\
AB &=& \begin{pmatrix}-i \cdot i + (1+i)\cdot (-1) \\
3i + i(1+i) \end{pmatrix} \\
&=& \begin{pmatrix}-i^{2} - 1 - i \\
3i + i + i^{2} \end{pmatrix} \\
&=& \begin{pmatrix}- 1 - i \\
4i - 1 \end{pmatrix} \\
\intertext{AC existiert nicht, da A mehr Spalten hat, als C Zeilen hat.}
BC &=& \begin{pmatrix}i \cdot (-i) & i \cdot i \\
(1+i) \cdot (-i) & (1+i)i \end{pmatrix} \\
&=& \begin{pmatrix}-i^{2} & i^{2} \\
-i - i^{2} & i + i^{2} \end{pmatrix} \\
&=& \begin{pmatrix}1 & -1 \\
-i +1 & i -1 \end{pmatrix} \\
CB &=& \begin{pmatrix}-i \cdot i + i(1+i) \end{pmatrix} \\
&=& \begin{pmatrix}-i^{2} + i + i^{2} \end{pmatrix} \\
&=& \begin{pmatrix}1 + i -1 \end{pmatrix} \\
&=& \begin{pmatrix}i \end{pmatrix}
\end{alignat*}
\subsection{} %b
\begin{alignat*}{2}
\overline{z} &=& \frac{3 + 2i}{4 - 3i} \\
&=& \frac{(3 + 2i)(4+3i)}{(4 - 3i)(4+3i)} \\
&=& \frac{12 + 9i + 8i + 6i^{2}}{16 + 12i - 12i - 9i^{2}} \\
&=& \frac{12 - 6 + 17i}{16 + 9} \\
&=& \frac{6 + 17i}{25} \\
&=& \frac{6}{25} + \frac{17}{25}i \\
z &=& \frac{6}{25} - \frac{17}{25}i \\
a &=& \frac{6}{25} \\
b &=& \frac{17}{25}
\end{alignat*}
\subsection{} %c
\begin{alignat*}{2}
z_{1} &=& -1 - i = (-1, -1) \\
z_{2} &=& \sqrt{2} \cdot \cos \frac{\pi}{4} + \sqrt{2} \cdot i \cdot \sin \frac{\pi}{4} \\
&=& \sqrt{2} \cdot \cos \left(\frac{1}{2} \cdot \frac{\pi}{2} \right) + \sqrt{2} \cdot \sin \left(\frac{1}{2} \cdot \frac{\pi}{2} \right) \\
&=& 1 + i = (1, 1) \\
z_{3} &=& (-1 -i)(1+ i) \\
&=& -1 - i - i - i^{2} \\
&=& -1 + 1 - 2i = 0 - 2i = (0, -2) \\
z_{4} &=& -1 + i = (-1, 1)
\end{alignat*}
\begin{tikzpicture}[>=stealth]
\begin{axis}[
ymin=-5,ymax=5,
x=1cm,
y=1cm,
axis x line=middle,
axis y line=middle,
axis line style=->,
xlabel={$\Re$},
ylabel={$\Im$},
xmin=-5,xmax=5
]
\node at (axis cs: -1,-1) {$z_{1}$};
\node at (axis cs: 1,1) {$z_{2}$};
\node at (axis cs: 0,-2) {$z_{3}$};
\node at (axis cs: -1,1) {$z_{4}$};
\end{axis}
\end{tikzpicture}
\subsection{} %d
\subsubsection{} %i
In dieser Teilmenge sind alle komplexen Zahlen enthalten, die sich auf der Geraden befinden, die durch die Punkte (1,1) und (2,0) geht.
\subsubsection{} %ii
In der Teilmenge sind alle komplexen Zahlen enthalten, die sich auf der Kreislinie eines Kreises mit dem Radius 1 um den Punkt (1,1) befinden.
\section{} %3
\begin{alignat*}{2}
f(x,y) &=& -\frac{1}{5}x^{2} - xy - \frac{25}{10}y^{2} + 48x + 235y - 88 \\
g(x,y) &=& \frac{1}{5}x + y = 40
\end{alignat*}
\subsection{} %a
\begin{alignat*}{2}
g_{x} &=& \frac{1}{5} \\
g_{y} &=& 1 \\
\begin{pmatrix}\frac{\partial g}{\partial x} (x,y) & \frac{\partial g}{\partial y} (x,y)\end{pmatrix} &=& \begin{pmatrix}\frac{1}{5} & 1 \end{pmatrix} \\
\intertext{Der Rang dieser Matrix ist für alle $x,y$ gleich 2. Damit ist die Regularitätsbedingung erfüllt.}
L(x,y,\lambda) &=& -\frac{1}{5}x^{2} - xy - \frac{25}{10}y^{2} + 48x + 235y - 88 + \lambda(\frac{1}{5}x + y - 40) \\
I L_{x} &=& - \frac{2}{5}x - y + 48 + \lambda \cdot \frac{1}{5} = 0 \\
II L_{y} &=& - x - 5y + 235 + \lambda = 0 \\
III L_{\lambda} &=& \frac{1}{5}x + y - 40 = 0 \\
II &\Rightarrow & \lambda = x + 5y - 235 \\
I &\Rightarrow & - \frac{2}{5}x - y + 48 + (x + 5y - 235) \cdot \frac{1}{5} = 0\\
&\Leftrightarrow & - \frac{2}{5}x - y + 48 + \frac{1}{5}x + y - 47 = 0 \\
\intertext{Beachten der dritten Gleichung}
&\Leftrightarrow & - \frac{1}{5}x - \frac{1}{5}x - y + 48 - 7 = 0\\
&\Leftrightarrow & - \frac{1}{5}x + 1=\frac{1}{5}x + y - 40 \\
\intertext{Beachten der dritten Gleichung}
&\Leftrightarrow & - \frac{1}{5}x + 1 = 0 \\
&\Leftrightarrow & \frac{1}{5}x = 1 \\
&\Leftrightarrow & x = 5 \\
\intertext{Einsetzen in I}
&\Rightarrow & - \frac{1}{5} \cdot 5 - \frac{1}{5} \cdot 5 - y + 48 - 7 = 0 \\
&\Leftrightarrow & - 1 - 1 - y + 41 = 0 \\
&\Leftrightarrow & -y + 39 = 0 \\
&\Leftrightarrow & y = 39 \\
\intertext{Einsetzen in III}
\lambda &=& \frac{1}{5} \cdot 5 + 39 - 40 \\
&=& 1 + 39 - 40 \\
&=& 0
\intertext{Die einzige kritische Stelle befindet sich an (5, 39).}
\end{alignat*}
\subsection{} %b
\begin{alignat*}{2}
L_{xx} &=& - \frac{2}{5} \\
L_{yx} &=& - 1 \\
L_{xy} &=& - 1 \\
L_{yy} &=& - 5 \\
\overline{H} &=& \begin{pmatrix}0 & \frac{1}{5} & 1 \\
\frac{1}{5} & -\frac{2}{5} & -1\\
1 & -1 & -5 \end{pmatrix} \\
det \, \overline{H} &=& -\frac{1}{5} - \frac{1}{5} + \frac{2}{5} + \frac{1}{5} \\
&=& \frac{1}{5} > 0 \\
\intertext{An der kritischen Stelle liegt ein Maximum vor.}
\end{alignat*}
\section{} %4
\begin{alignat*}{2}
f(x,y) &=& -\frac{1}{5}x^{2} - xy - \frac{25}{10}y^{2} + 48x + 235y - 88 \\
g(x,y) &=& \frac{1}{5}x + y -40 = 0 \\
&\Leftrightarrow & y = -\frac{1}{5}x + 40 \\
\intertext{Einsetzen in f(x,y)}
f(x) &=& -\frac{1}{5}x^{2} - x \cdot (-\frac{1}{5}x + 40) - \frac{25}{10} \cdot (-\frac{1}{5}x + 40)^{2} + 48x + 235 \cdot (-\frac{1}{5}x + 40) - 88 \\
&=& -\frac{1}{5}x^{2} + \frac{1}{5}x^{2} - 40x - \frac{25}{10} \cdot (\frac{1}{25}x^{2} -16x + 1600) + 48x -47x + 9400 - 88 \\
&=& - 40x - \frac{1}{10}x^{2} + 40x - 4000 + 48x -47x + 9400 - 88 \\
&=& - \frac{1}{10}x^{2} + x + 5312 \\
f'(x) &=& -\frac{1}{5}x + 1 = 0 \\
&\Leftrightarrow & \frac{1}{5}x = 1 \\
&\Leftrightarrow & x = 5 \\
f''(x) &=& -\frac{1}{5} < 0 \\
\intertext{Unter der Nebenbedingung g(x,y) gibt es ein lokales Maximum für x = 5.
Einsetzen von x in die Nebenbedingung:}
g(y) &=& \frac{1}{5} \cdot 5 + y -40 = 0 \\
&\Leftrightarrow & 1 + y - 40 = 0 \\
&\Leftrightarrow & y - 39 = 0 \\
&\Leftrightarrow & y = 39 \\
\intertext{An der Stelle (5, 39) ist der Gewinn maximal.}
\end{alignat*}
\end{document}

View File

@ -0,0 +1,102 @@
\documentclass[10pt,a4paper,oneside,ngerman,numbers=noenddot]{scrartcl}
\usepackage[T1]{fontenc}
\usepackage[utf8]{inputenc}
\usepackage[ngerman]{babel}
\usepackage{amsmath}
\usepackage{amsfonts}
\usepackage{amssymb}
\usepackage{paralist}
\usepackage{gauss}
\usepackage{pgfplots}
\usepackage[locale=DE,exponent-product=\cdot,detect-all]{siunitx}
\usepackage{tikz}
\usetikzlibrary{matrix,fadings,calc,positioning,decorations.pathreplacing,arrows,decorations.markings}
\usepackage{polynom}
\polyset{style=C, div=:,vars=x}
\pgfplotsset{compat=1.8}
\pagenumbering{arabic}
\def\thesection{\arabic{section})}
\def\thesubsection{\alph{subsection})}
\def\thesubsubsection{(\roman{subsubsection})}
\makeatletter
\renewcommand*\env@matrix[1][*\c@MaxMatrixCols c]{%
\hskip -\arraycolsep
\let\@ifnextchar\new@ifnextchar
\array{#1}}
\makeatother
\begin{document}
\author{Jim Martens (6420323)}
\title{Hausaufgaben zum 11. Juli}
\maketitle
\section{} %1
\begin{alignat*}{2}
\iint\limits_{I} f(x,y)\, d(x,y) &=& \iint\limits_{I} (2x^{2}y)\, d(x,y) \\
\intertext{Version 1}
&=& \int\limits_{1}^{2} \left( \int\limits_{-1}^{3} (2x^{2}y) \, dy \right) \, dx \\
&=& \int\limits_{1}^{2} \left( 2x^{2} \int\limits_{-1}^{3} y \, dy \right)\, dx \\
&=& \int\limits_{1}^{2} \left( 2x^{2} \left[\frac{1}{2}y^{2}\right]_{-1}^{3} \right)\, dx \\
&=& \int\limits_{1}^{2} \left( 2x^{2} \left[\frac{9}{2} - \frac{1}{2}\right] \right)\, dx \\
&=& \int\limits_{1}^{2} \left( 2x^{2} \cdot 4 \right)\, dx \\
&=& 8 \int\limits_{1}^{2} x^{2}\, dx \\
&=& 8 \left[\frac{1}{3}x^{3} \right]_{1}^{2} \\
&=& 8 \left[\frac{8}{3} - \frac{1}{3} \right] \\
&=& 8 \cdot \frac{7}{3} = \frac{56}{3} \\
\intertext{Version 2}
&=& \int\limits_{-1}^{3} \left( \int\limits_{1}^{2} (2x^{2}y) \, dx \right) \, dy \\
&=& \int\limits_{-1}^{3} \left( 2y\int\limits_{1}^{2} x^{2} \, dx \right) \, dy \\
&=& \int\limits_{-1}^{3} \left( 2y\left[\frac{1}{3}x^{3} \right]_{1}^{2} \right) \, dy \\
&=& \int\limits_{-1}^{3} \left( 2y\left[\frac{8}{3} - \frac{1}{3} \right] \right) \, dy \\
&=& \int\limits_{-1}^{3} \left( 2y \cdot \frac{7}{3} \right) \, dy \\
&=& \frac{14}{3}\int\limits_{-1}^{3} y \, dy \\
&=& \frac{14}{3}\left[\frac{1}{2}y^{2}\right]_{-1}^{3} \\
&=& \frac{14}{3}\left[\frac{9}{2} - \frac{1}{2}\right] \\
&=& \frac{14}{3} \cdot 4 = \frac{56}{3}
\end{alignat*}
\section{} %2
\subsubsection{} %i
\begin{alignat*}{2}
\iint\limits_{G} f(x,y) \, d(x,y) &=& \iint\limits_{G} (xy^{2}) \, d(x,y) \\
&=& \int\limits_{0}^{1} \left(\int\limits_{0}^{3x} xy^{2}\, dy \right) \, dx \\
&=& \int\limits_{0}^{1} \left(x\int\limits_{0}^{3x} y^{2}\, dy \right) \, dx \\
&=& \int\limits_{0}^{1} \left(x \left[\frac{1}{3}y^{3} \right]_{0}^{3x} \right) \, dx \\
&=& \int\limits_{0}^{1} \left(x \left[\frac{1}{3} \cdot 27x^{3} \right] \right) \, dx \\
&=& \int\limits_{0}^{1} \left(x \cdot 9x^{3} \right) \, dx \\
&=& \int\limits_{0}^{1} \left(9x^{4} \right) \, dx \\
&=& 9\int\limits_{0}^{1} x^{4} \, dx \\
&=& 9\left[\frac{1}{5}x^{5} \right]_{0}^{1} \\
&=& 9 \cdot \frac{1}{5} = \frac{9}{5}
\end{alignat*}
\subsubsection{} %ii
\begin{alignat*}{2}
\iint\limits_{G} f(x,y) \, d(x,y) &=& \iint\limits_{G} (xy^{2}) \, d(x,y) \\
&=& \int\limits_{0}^{1} \left(\int\limits_{3x}^{3} xy^{2} \, dy \right)\, dx \\
&=& \int\limits_{0}^{1} \left(x\int\limits_{3x}^{3} y^{2} \, dy \right)\, dx \\
&=& \int\limits_{0}^{1} \left(x \left[\frac{1}{3}y^{3} \right]_{3x}^{3} \right)\, dx \\
&=& \int\limits_{0}^{1} \left(x \left[\frac{27}{3} - \frac{27}{3}x^{3} \right] \right)\, dx \\
&=& \int\limits_{0}^{1} \left(9x - 9x^{4} \right)\, dx \\
&=& \int\limits_{0}^{1} \left(9(x - x^{4}) \right)\, dx \\
&=& 9\int\limits_{0}^{1} x - x^{4}\, dx \\
&=& 9\int\limits_{0}^{1} x \, dx - 9\int\limits_{0}^{1} x^{4}\, dx \\
&=& 9 \left[\frac{1}{2}x^{2} \right]_{0}^{1} - 9\left[\frac{1}{5}x^{5} \right]_{0}^{1} \\
&=& 9 \cdot \frac{1}{2} - 9 \cdot \frac{1}{5} \\
&=& \frac{9}{2} - \frac{9}{5} \\
&=& \frac{45}{10} - \frac{18}{10} = \frac{27}{10}
\end{alignat*}
\section{} %3
\subsection{} %a
Klarerweise gilt $f_{4}(n) = O(f_{5}(n))$. Ebenfalls gilt $f_{3}(n) = O(f_{1}(n))$. Außerdem ist klar, dass $f_{1}(n) = O(f_{4}(n))$ gilt.
Damit ergibt sich die Reihenfolge $f_{3}, f_{1}, f_{4}, f_{5}$.
Es müssen noch $f_{2}$ und $f_{6}$ eingeordnet werden. $f_{2}(n) = O(f_{1}(n))$ gilt ebenso wie $f_{3}(n) = O(f_{2}(n))$. $f_{2}$ kann demnach zwischen $f_{3}$ und $f_{1}$ eingeordnet werden, womit sich die Reihenfolge $f_{3}, f_{2}, f_{1}, f_{4}, f_{5}$ ergibt.
Abschließend muss noch $f_{6}(n)$ eingeordnet werden. $n^{2}$ kommt als Faktor auch in $f_{1}$ vor. Es bleibt daher die Frage, ob $\sqrt{n}$ schneller wächst als $\log_{2}(n)$. Dem ist so, da Wurzelfunktionen allgemein schneller wachsen als Logarithmusfunktionen. Daher gilt $f_{6}(n) = O(f_{1}(n))$. Gleichzeitig gilt, dass $\sqrt{2n}$ langsamer wächst als $f_{6}(n)$, womit auch $f_{2}(n) = O(f_{6}(n))$ gilt. Die fertige Reihenfolge ist daher $f_{3}, f_{2}, f_{6}, f_{1}, f_{4}, f_{5}$.
\subsection{} %b
\section{} %4
\subsection{} %a
\subsection{} %b
\begin{alignat*}{2}
\intertext{Es gilt $n = \lfloor x \rfloor, x \geq 1$ für beide Funktionen:}
f(n) &=& n \\
g(n) &=& n^{1+ \lceil \sin (x) \rceil}
\end{alignat*}
\end{document}

View File

@ -0,0 +1,216 @@
\documentclass[10pt,a4paper,oneside,ngerman,numbers=noenddot]{scrartcl}
\usepackage[T1]{fontenc}
\usepackage[utf8]{inputenc}
\usepackage[ngerman]{babel}
\usepackage{amsmath}
\usepackage{amsfonts}
\usepackage{amssymb}
\usepackage{paralist}
\usepackage{gauss}
\usepackage[locale=DE,exponent-product=\cdot,detect-all]{siunitx}
\usepackage{tikz}
\usetikzlibrary{matrix,fadings,calc,positioning,decorations.pathreplacing,arrows,decorations.markings}
\usepackage{polynom}
\polyset{style=C, div=:,vars=x}
\pagenumbering{arabic}
\def\thesection{\arabic{section})}
\def\thesubsection{\alph{subsection})}
\def\thesubsubsection{(\roman{subsubsection})}
\makeatletter
\renewcommand*\env@matrix[1][*\c@MaxMatrixCols c]{%
\hskip -\arraycolsep
\let\@ifnextchar\new@ifnextchar
\array{#1}}
\makeatother
\begin{document}
\author{Jim Martens (6420323)}
\title{Hausaufgaben zum 18. April}
\maketitle
\section{} %1
\subsubsection{} %(i)
\begin{alignat*}{3}
\underset{n \rightarrow \infty}{\text{lim}} \left(\frac{-3n^{4}+2n^{2}+n+1}{-7n^{4}+25} \right) &\Rightarrow & \frac{-3n^{4}+2n^{2}+n+1}{-7n^{4}+25} && \\
\intertext{Ausklammern von $n^{4}$}
&\Leftrightarrow & \frac{-3 + \frac{2}{n^{2}} + \frac{1}{n^{3}} + \frac{1}{n^{4}}}{-7 + \frac{25}{n^{4}}} &\rightarrow & \frac{3}{7}
\end{alignat*}
\subsubsection{} %(ii)
\begin{alignat*}{3}
\underset{n \rightarrow \infty}{\text{lim}} \left(\frac{-3n^{4}+2n^{2}+n+1}{-7n^{5}+25} \right) &\Rightarrow & \frac{-3n^{4}+2n^{2}+n+1}{-7n^{5}+25} && \\
\intertext{Ausklammern von $n^{4}$ im Zähler und $n^{5}$ im Nenner}
&\Leftrightarrow & \frac{1}{n} \cdot \frac{-3 + \frac{2}{n^{2}} + \frac{1}{n^{3}} + \frac{1}{n^{4}}}{-7 + \frac{25}{n^{5}}} &\rightarrow & 0
\end{alignat*}
\subsubsection{} %(iii)
\begin{alignat*}{3}
\underset{n \rightarrow \infty}{\text{lim}} \left(\frac{-3n^{5}+2n^{2}+n+1}{-7n^{4}+25} \right) &\Rightarrow & \frac{-3n^{5}+2n^{2}+n+1}{-7n^{4}+25} && \\
\intertext{Ausklammern von $n^{5}$ im Zähler und $n^{4}$ im Nenner}
&\Leftrightarrow & n \cdot \frac{-3 + \frac{2}{n^{3}} + \frac{1}{n^{4}} + \frac{1}{n^{5}}}{-7 + \frac{25}{n^{4}}} &\rightarrow & \infty
\end{alignat*}
\subsubsection{} %(iv)
\begin{alignat*}{3}
\hspace{-2.5cm}
\underset{n \rightarrow \infty}{\text{lim}} \left( \frac{6n^{3}+2n-3}{9n^{2}+2} - \frac{2n^{3}+5n^{2}+7}{3n^{2}+3} \right) &\Rightarrow & \frac{6n^{3}+2n-3}{9n^{2}+2} - \frac{2n^{3}+5n^{2}+7}{3n^{2}+3} && \\
\intertext{Auf gleichen Nenner bringen}
\hspace{-2.5cm}
&\Leftrightarrow & \frac{(6n^{3}+2n-3)(3n^{2}+3) - (2n^{3}+5n^{2}+7)(9n^{2}+2)}{(9n^{2}+2)(3n^{2}+3)} && \\
\intertext{Klammern auflösen und zusammenfassen}
\hspace{-2.5cm}
&\Leftrightarrow & \frac{-45n^{4} + 20n^{3}-82n^{2}+6n-23}{27n^{4}+33n^{2}+6} && \\
\intertext{Ausklammern von $n^{4}$}
\hspace{-2.5cm}
&\Leftrightarrow & \frac{-45 + \frac{20}{n}-\frac{82}{n^{2}}+\frac{6}{n^{3}}-\frac{23}{n^{4}}}{27+\frac{33}{n^{2}}+\frac{6}{n^{4}}} &\rightarrow & \frac{-45}{27} = \frac{-5}{3}
\end{alignat*}
\subsubsection{} %(v)
\begin{alignat*}{3}
\underset{n \rightarrow \infty}{\text{lim}} \left( \frac{\sqrt{9n^{4}+n^{2}+1}-2n^{2}+3}{\sqrt{2n^{2}+1} \cdot \sqrt{2n^{2}+n+1}} \right) &\Rightarrow & \frac{\sqrt{9n^{4}+n^{2}+1}-2n^{2}+3}{\sqrt{2n^{2}+1} \cdot \sqrt{2n^{2}+n+1}} && \\
\intertext{Anwendung der Wurzelgesetze}
&\Leftrightarrow &\frac{\sqrt{9n^{4}+n^{2}+1}-2n^{2}+3}{\sqrt{(2n^{2}+1) \cdot (2n^{2}+n+1)}} && \\
\intertext{Zusammenfassen}
&\Leftrightarrow &\frac{\sqrt{9n^{4}+n^{2}+1}-2n^{2}+3}{\sqrt{4n^{4}+2n^{3}+4n^{2} + 1}} && \\
\intertext{$n^{2}$ ausklammern}
&\Leftrightarrow &\frac{\sqrt{9+\frac{1}{n^{2}}+\frac{1}{n^{4}}}-2+\frac{3}{n^{2}}}{\sqrt{4+\frac{2}{n}+\frac{4}{n^{2}} + \frac{1}{n^{4}}}} &\rightarrow & \frac{7}{2}
\end{alignat*}
\section{} %2
\subsection{} %a
\subsubsection{} %i
\begin{alignat*}{3}
a_{0} &=& 1 &&&\\
a_{1} &=& \frac{2}{5} &&& \\
a_{2} &=& \left(\frac{2}{5}\right)^{2} = \frac{4}{25} &&& \\
a_{3} &=& \left(\frac{2}{5}\right)^{3} = \frac{8}{125} &&& \\
a_{4} &=& \left(\frac{2}{5}\right)^{4} = \frac{16}{625} &&& \\
s_{0} &=& a_{0} &=&& 1 \\
s_{1} &=& a_{0} + a_{1} = 1 + \frac{2}{5} = \frac{7}{5} &=&& 1.4 \\
s_{2} &=& a_{0} + a_{1} + a_{2} = \frac{7}{5} + \frac{4}{25} = \frac{39}{25} &=&& 1.56 \\
s_{3} &=& a_{0} + a_{1} + a_{2} + a_{3} = \frac{39}{25} + \frac{8}{125} = \frac{203}{125} &=&& 1.624 \\
s_{4}&=& a_{0} + a_{1} + a_{2} + a_{3} + a_{4} = \frac{203}{125} + \frac{16}{625} = \frac{1031}{625} &=&& 1.6496
\end{alignat*}\\
Bestimmung des Grenzwertes mithilfe der Geometrischen Summenformel:\\
\begin{alignat*}{2}
\underset{n \rightarrow \infty}{\text{lim}} \left( \sum\limits_{i=0}^{n} \left(\frac{2}{5}\right)^{i} \right) &=& \underset{n \rightarrow \infty}{\text{lim}} \left(\frac{1-\left(\frac{2}{5}\right)^{n+1}}{1-\left(\frac{2}{5}\right)} \right) \\
&=& \frac{1}{\frac{3}{5}} = \frac{5}{3}
\end{alignat*}
\subsubsection{} %ii
\begin{alignat*}{3}
a_{0} &=& 1 &&& \\
a_{1} &=& \frac{5}{2}&&& \\
a_{2} &=& \left(\frac{5}{2}\right)^{2} = \frac{25}{4} &&&\\
a_{3} &=& \left(\frac{5}{2}\right)^{3} = \frac{125}{8} &&& \\
a_{4} &=& \left(\frac{5}{2}\right)^{4} = \frac{625}{16} &&& \\
s_{0} &=& a_{0} &=& 1 \\
s_{1} &=& a_{0} + a_{1} = 1 + \frac{5}{2} = \frac{7}{2} &=&& 3.5 \\
s_{2} &=& a_{0} + a_{1} + a_{2} = \frac{7}{2} + \frac{25}{4} = \frac{39}{4} &=&& 9.75 \\
s_{3} &=& a_{0} + a_{1} + a_{2} + a_{3} = \frac{39}{4} + \frac{125}{8} = \frac{203}{8} &=&& 25.375 \\
s_{4}&=& a_{0} + a_{1} + a_{2} + a_{3} + a_{4} = \frac{203}{8} + \frac{625}{16} = \frac{1031}{16} &=&& 64,4375
\end{alignat*}\\
Die Reihe divergiert, da geometrische Reihen immer divergieren, wenn der Betrag von q größer als $1$ ist. Dies ist mit $\frac{5}{2}$ der Fall.
\subsubsection{} %iii
\begin{alignat*}{3}
a_{0} &=& 1 &&&\\
a_{1} &=& -\frac{2}{5} &&& \\
a_{2} &=& \left(-\frac{2}{5}\right)^{2} = \frac{4}{25} &&& \\
a_{3} &=& \left(-\frac{2}{5}\right)^{3} = -\frac{8}{125} &&& \\
a_{4} &=& \left(-\frac{2}{5}\right)^{4} = \frac{16}{625} &&& \\
s_{0} &=& a_{0} &=&& 1 \\
s_{1} &=& a_{0} + a_{1} = 1 - \frac{2}{5} = \frac{3}{5} &=&& 0.6 \\
s_{2} &=& a_{0} + a_{1} + a_{2} = \frac{3}{5} + \frac{4}{25} = \frac{19}{25} &=&& 0.76 \\
s_{3} &=& a_{0} + a_{1} + a_{2} + a_{3} = \frac{19}{25} - \frac{8}{125} = \frac{87}{125} &=&& 0.696 \\
s_{4}&=& a_{0} + a_{1} + a_{2} + a_{3} + a_{4} = \frac{87}{125} + \frac{16}{625} = \frac{451}{625} &=&& 0.7216
\end{alignat*} \\
Bestimmung des Grenzwertes mithilfe der Geometrischen Summenformel:\\
\begin{alignat*}{2}
\underset{n \rightarrow \infty}{\text{lim}} \left( \sum\limits_{i=0}^{n} \left(-\frac{2}{5}\right)^{i} \right) &=& \underset{n \rightarrow \infty}{\text{lim}} \left(\frac{1-\left(-\frac{2}{5}\right)^{n+1}}{1+\left(\frac{2}{5}\right)} \right) \\
&=& \frac{1}{\frac{7}{5}} = \frac{5}{7}
\end{alignat*}
\subsection{} %b
\subsubsection{} %i
Bestimmung des Grenzwertes mithilfe der Geometrischen Summenformel:\\
\begin{alignat*}{2}
\underset{n \rightarrow \infty}{\text{lim}} \left( \sum\limits_{i=0}^{n} \left(-\frac{3}{10}\right)^{i} \right) &=& \underset{n \rightarrow \infty}{\text{lim}} \left(\frac{1-\left(-\frac{3}{10}\right)^{n+1}}{1+\left(\frac{3}{10}\right)} \right) \\
&=& \frac{1}{\frac{13}{10}} = \frac{10}{13}
\end{alignat*}
Die Reihe konvergiert gegen den Wert $\frac{10}{13} \approx 0.769$.
\subsubsection{} %ii
\begin{alignat*}{3}
&& \underset{n \rightarrow \infty}{\text{lim}} \left( \sum\limits_{i=0}^{n} x^{i} \right) &=& \frac{5}{8} \\
\Rightarrow && \frac{1}{1-x} &=& \frac{5}{8} \\
\overset{\cdot (1-x)}{\Leftrightarrow} && 1 &=& \frac{5}{8} - \frac{5}{8}x \\
\overset{-\frac{5}{8}}{\Leftrightarrow} && \frac{3}{8} &=& -\frac{5}{8}x \\
\overset{\cdot -\frac{8}{5}}{\Leftrightarrow} && -\frac{3}{5} &=& x
\end{alignat*}
$x$ ist gleich $-\frac{3}{5}$.
\section{} %3
\subsection{}
\subsubsection{} %(i)
Die Reihe konvergiert, da der Betrag von $q = \frac{7}{9}$ kleiner als $1$ ist.\\
Berechnung des Grenzwertes mithilfe der Geometrischen Summenformel:\\
\begin{alignat*}{2}
\underset{n \rightarrow \infty}{\text{lim}} \left( \sum\limits_{i=0}^{n} \left(\frac{7}{9}\right)^{i} \right) &=& \underset{n \rightarrow \infty}{\text{lim}} \left(\frac{1-\left(\frac{7}{9}\right)^{n+1}}{1-\left(\frac{7}{9}\right)} \right) \\
&=& \frac{1}{\frac{2}{9}} = \frac{9}{2}
\end{alignat*}
\subsubsection{} %(ii)
Die Reihe konvergiert, da der Betrag von $q = -\frac{7}{9}$ kleiner als $1$ ist.\\
Berechnung des Grenzwertes mithilfe der Geometrischen Summenformel:\\
\begin{alignat*}{2}
\sum\limits_{i=1}^{\infty} \left(-\frac{7}{9}\right)^{i} &=& \sum\limits_{i=0}^{\infty} \left(-\frac{7}{9}\right)^{i} - \left(-\frac{7}{9}\right)^{0} \\
&=& \underset{n \rightarrow \infty}{\text{lim}} \left( \sum\limits_{i=0}^{n} \left(-\frac{7}{9}\right)^{i} - \left(-\frac{7}{9}\right)^{0} \right) \\
&=& \underset{n \rightarrow \infty}{\text{lim}} \left( \sum\limits_{i=0}^{n} \left(-\frac{7}{9}\right)^{i} \right) - 1 \\
&=& \frac{1}{1 + \frac{7}{9}} - 1 = \frac{1}{\frac{16}{9}} \\
&=& \frac{9}{16} - 1 = -\frac{7}{16}
\end{alignat*}
Der Grenzwert ist $-\frac{7}{16}$.
\subsubsection{} %(iii)
\begin{alignat*}{2}
\sum\limits_{i=2}^{\infty} (-1)^{i} \cdot \left( \frac{7}{9} \right)^{i+1} &=& \sum\limits_{i=2}^{\infty} (-1)^{i} \cdot \left( \frac{7}{9} \right)^{i} \cdot \left( \frac{7}{9} \right) \\
&=& \sum\limits_{i=2}^{\infty} \left( -\frac{7}{9} \right)^{i} \cdot \left( \frac{7}{9} \right) \\
&=& \left( \frac{7}{9} \right) \cdot \sum\limits_{i=2}^{\infty} \left( -\frac{7}{9} \right)^{i} \\
&=& \underset{n \rightarrow \infty}{\text{lim}} \left( \frac{7}{9} \cdot \sum\limits_{i=2}^{n} \left( -\frac{7}{9} \right)^{i} \right) \\
&=& \underset{n \rightarrow \infty}{\text{lim}} \left( \frac{7}{9} \cdot \left( \sum\limits_{i=0}^{n} \left( -\frac{7}{9} \right)^{i} - \left( -\frac{7}{9} \right)^{0} - \left( -\frac{7}{9} \right)^{1} \right) \right) \\
&=& \underset{n \rightarrow \infty}{\text{lim}} \left( \frac{7}{9} \cdot \left( \sum\limits_{i=0}^{n} \left( -\frac{7}{9} \right)^{i} - 1 + \frac{7}{9} \right) \right) \\
&=& \underset{n \rightarrow \infty}{\text{lim}} \left( \frac{7}{9} \cdot \left( \frac{1- \left(-\frac{7}{9} \right)^{n+1}}{1 - \left(-\frac{7}{9} \right)} - 1 + \frac{7}{9} \right) \right) \\
&=& \underset{n \rightarrow \infty}{\text{lim}} \left( \frac{7}{9} \cdot \left( \frac{1- \left(-\frac{7}{9} \right)^{n+1}}{1 + \frac{7}{9}} - 1 + \frac{7}{9} \right) \right)\\
&\rightarrow & \frac{7}{9} \cdot \left( \frac{1}{1 + \frac{7}{9}} - 1 + \frac{7}{9} \right) \\
&=& \frac{7}{9} \cdot \left( \frac{9}{16} - 1 + \frac{7}{9} \right) \\
&=& \frac{7}{9} \cdot \left( -\frac{7}{16} + \frac{7}{9} \right) \\
&=& \frac{7}{9} \cdot \frac{49}{144} \\
&=& \frac{343}{1296} \approx 0.26
\end{alignat*}\\
Der Grenzwert beträgt $\frac{343}{1296}$.
\subsubsection{} %(iv)
\begin{alignat*}{3}
\sum\limits_{i=2}^{\infty} \frac{1}{(i+1)i} &=& \sum\limits_{i=2}^{\infty} \left( \frac{1}{i} - \frac{1}{i+1} \right) \\
&=& \sum\limits_{i=2}^{\infty} \frac{1}{i} - \sum\limits_{i=2}^{\infty} \frac{1}{i+1} \\
&=& \frac{1}{2} + \frac{1}{3} + \frac{1}{4} + ... + \frac{1}{n} - \frac{1}{3} - \frac{1}{4} - ... - \frac{1}{n+1} \\
&\Rightarrow & \underset{n \rightarrow \infty}{\text{lim}} \left(\frac{1}{2} - \frac{1}{n+1} \right) = \frac{1}{2}
\end{alignat*}\\
Der Grenzwert ist $\frac{1}{2}$.
\section{} %4
\subsection{}
\subsubsection{} %(i)
\begin{alignat*}{2}
\underset{n \rightarrow \infty}{\text{lim}} \left(1 + \frac{1}{n} \right)^{n+3} &=& \underset{n \rightarrow \infty}{\text{lim}} \left(1 + \frac{1}{n} \right)^{n} \cdot \underset{n \rightarrow \infty}{\text{lim}} \left(1 + \frac{1}{n} \right)^{3} \\
&=& e \cdot 1 = e
\end{alignat*}\\
\subsubsection{} %(ii)
\begin{alignat*}{2}
\underset{n \rightarrow \infty}{\text{lim}} \left(1 + \frac{1}{n} \right)^{3n} &=& \underset{n \rightarrow \infty}{\text{lim}} \left(1 + \frac{1}{n} \right)^{n} \cdot \underset{n \rightarrow \infty}{\text{lim}} \left(1 + \frac{1}{n} \right)^{n} \cdot \underset{n \rightarrow \infty}{\text{lim}} \left(1 + \frac{1}{n} \right)^{n} \\
&=& e \cdot e \cdot e \\
&=& e^{3}
\end{alignat*}\\
\subsubsection{} %(iii)
\begin{alignat*}{2}
\underset{n \rightarrow \infty}{\text{lim}} \left(1 + \frac{1}{n} \right)^{3} &=& \underset{n \rightarrow \infty}{\text{lim}} \left(1 + \frac{1}{n} \right) \cdot \underset{n \rightarrow \infty}{\text{lim}} \left(1 + \frac{1}{n} \right) \cdot \underset{n \rightarrow \infty}{\text{lim}} \left(1 + \frac{1}{n} \right) \\
&=& 1 \cdot 1 \cdot 1 = 1
\end{alignat*}
\subsubsection{} %(iv)
\begin{alignat*}{2}
\underset{n \rightarrow \infty}{\text{lim}} \left(1 + \frac{1}{3n} \right)^{3n} &=& \underset{n \rightarrow \infty}{\text{lim}} \left( \left(1 + \frac{1}{3n} \right)^{3} \right)^{n} \\
&=& \underset{n \rightarrow \infty}{\text{lim}} \left( \left(1 + \frac{1}{3n} \right)^{2} \cdot \left(1 + \frac{1}{3n} \right) \right)^{n} \\
&=& \underset{n \rightarrow \infty}{\text{lim}} \left( \left(1 + \frac{2}{3n} + \frac{1}{9n^{2}} \right) \cdot \left(1 + \frac{1}{3n} \right) \right)^{n} \\
&=& \underset{n \rightarrow \infty}{\text{lim}} \left( 1 + \frac{1}{3n} + \frac{2}{3n} + \frac{2}{9n^{2}} + \frac{1}{9n^{2}} + \frac{1}{27n^{3}} \right)^{n} \\
&=& \underset{n \rightarrow \infty}{\text{lim}} \left( 1 + \frac{3}{3n} + \frac{3}{9n^{2}} + \frac{1}{27n^{3}} \right)^{n} \\
&=& \underset{n \rightarrow \infty}{\text{lim}} \left( 1 + \frac{1}{n} + \frac{1}{3n^{2}} + \frac{1}{27n^{3}} \right)^{n} \\
&\rightarrow & 1
\end{alignat*}
\end{document}

View File

@ -0,0 +1,105 @@
\documentclass[10pt,a4paper,oneside,ngerman,numbers=noenddot]{scrartcl}
\usepackage[T1]{fontenc}
\usepackage[utf8]{inputenc}
\usepackage[ngerman]{babel}
\usepackage{amsmath}
\usepackage{amsfonts}
\usepackage{amssymb}
\usepackage{paralist}
\usepackage{gauss}
\usepackage[locale=DE,exponent-product=\cdot,detect-all]{siunitx}
\usepackage{tikz}
\usetikzlibrary{matrix,fadings,calc,positioning,decorations.pathreplacing,arrows,decorations.markings}
\usepackage{polynom}
\polyset{style=C, div=:,vars=x}
\pagenumbering{arabic}
\def\thesection{\arabic{section})}
\def\thesubsection{\alph{subsection})}
\def\thesubsubsection{(\roman{subsubsection})}
\makeatletter
\renewcommand*\env@matrix[1][*\c@MaxMatrixCols c]{%
\hskip -\arraycolsep
\let\@ifnextchar\new@ifnextchar
\array{#1}}
\makeatother
\begin{document}
\author{Jim Martens (6420323)}
\title{Hausaufgaben zum 25. April}
\maketitle
\section{} %1
\subsection{} %a
\vspace{11cm}
Die Unstetigkeitsstellen sind $x=2$ und $x=6$.
\subsection{} %b
\vspace{4cm}
Die Funktion $g(x)$ ist periodisch. Betrachtet man die Periode für $x=0$ bis $x=1$, so ist ersichtlich, dass sowohl $x=0$ als auch $x=1$ Unstetigkeitsstellen sind. Nähert man sich an die beiden Stellen von links an, so stimmt der Grenzwert nicht mit dem Funktionswert überein. Nähert man sich von rechts an, stimmt er überein.\\
Stetigkeit erfordert jedoch, dass der Grenzwert existiert und gleich dem Funktionswert ist, unabhängig von der Folge mit der man sich annähert.
Daher ist $g(x)$ in diesen beiden Stellen unstetig. Aufgrund der Periodizität der Funktion ist $g(x)$ an allen Stellen $x \in \mathbb{Z}$ unstetig.\\
\\
Betrachtet man hingegen eine andere Stelle in der Periode, so stimmen Grenzwert und Funktionswert überein, unabhängig davon ob man sich von rechts oder links annähert. Daher ist $g(x)$ in allen Stellen der Periode mit $x \neq 0$ und $x \neq 1$ stetig. Aufgrund der Periodizität der Funktion ist $g(x)$ an allen Stellen $x \not\in \mathbb{Z}$ stetig.
\section{} %2
\subsection{} %a
\begin{alignat*}{2}
\underset{n \rightarrow \infty}{\text{lim}} a_{n} &=& \underset{n \rightarrow \infty}{\text{lim}} \frac{\sqrt{3n^{2}-2n+5}-\sqrt{n}}{\sqrt{n^{2}-n+1}+4n} \\
\intertext{Ausklammern von $n^{2}$ unterhalb der Wurzeln}
&=& \underset{n \rightarrow \infty}{\text{lim}} \frac{\sqrt{n^{2} (3 - \frac{2}{n} + \frac{5}{n^{2}})} - \sqrt{n^{2} \cdot \frac{1}{n}}}{\sqrt{n^{2} (1 -\frac{1}{n} + \frac{1}{n^{2}})} + 4n} \\
\intertext{Wurzelgesetze anwenden}
&=& \underset{n \rightarrow \infty}{\text{lim}} \frac{\sqrt{n^{2}} \cdot \sqrt{3 - \frac{2}{n} + \frac{5}{n^{2}}} - \sqrt{n^{2}} \cdot \sqrt{\frac{1}{n}}}{\sqrt{n^{2}} \cdot \sqrt{1 -\frac{1}{n} + \frac{1}{n^{2}}} + 4n} \\
\intertext{Wurzel auflösen}
&=& \underset{n \rightarrow \infty}{\text{lim}} \frac{n \cdot \sqrt{3 - \frac{2}{n} + \frac{5}{n^{2}}} - n \cdot \sqrt{\frac{1}{n}}}{n \cdot \sqrt{1 -\frac{1}{n} + \frac{1}{n^{2}}} + 4n} \\
\intertext{$n$ ausklammern}
&=& \underset{n \rightarrow \infty}{\text{lim}} \frac{n \cdot (\sqrt{3 - \frac{2}{n} + \frac{5}{n^{2}}} - \sqrt{\frac{1}{n}})}{n \cdot (\sqrt{1 -\frac{1}{n} + \frac{1}{n^{2}}} + 4)} \\
\intertext{$n$ kürzen}
&=& \underset{n \rightarrow \infty}{\text{lim}} \frac{\sqrt{3 - \frac{2}{n} + \frac{5}{n^{2}}} - \sqrt{\frac{1}{n}}}{\sqrt{1 -\frac{1}{n} + \frac{1}{n^{2}}} + 4} \\
\intertext{lim mit Wurzelfunktion vertauschen, da Wurzelfunktion stetig}
&=& \frac{\sqrt{\underset{n \rightarrow \infty}{\text{lim}} (3 - \frac{2}{n} + \frac{5}{n^{2}})} - \sqrt{\underset{n \rightarrow \infty}{\text{lim}}(\frac{1}{n})}}{\sqrt{\underset{n \rightarrow \infty}{\text{lim}} (1 -\frac{1}{n} + \frac{1}{n^{2}})} + 4} \\
\intertext{limes anwenden und Nullfolgen entfernen}
&=& \frac{\sqrt{3} - \sqrt{0}}{\sqrt{1} + 4} \\
\intertext{Zusammenfassen}
&=& \frac{\sqrt{3}}{5}
\end{alignat*}
\subsection{} %b
\begin{alignat*}{2}
&& \underset{n \rightarrow \infty}{\text{lim}} \left( \text{cos} \left( \frac{\sqrt{10n^{2}-n}-n}{2n+3} \right) \right) \\
\intertext{Ausklammern von $n^{2}$ unter der Wurzel}
&=& \underset{n \rightarrow \infty}{\text{lim}} \left( \text{cos} \left( \frac{\sqrt{n^{2} (10-\frac{1}{n})}-n}{2n+3} \right) \right) \\
\intertext{Wurzelgesetze anwenden}
&=& \underset{n \rightarrow \infty}{\text{lim}} \left( \text{cos} \left( \frac{\sqrt{n^{2}} \cdot \sqrt{10-\frac{1}{n}}-n}{2n+3} \right) \right) \\
\intertext{Wurzel auflösen}
&=& \underset{n \rightarrow \infty}{\text{lim}} \left( \text{cos} \left( \frac{n \cdot \sqrt{10-\frac{1}{n}}-n}{2n+3} \right) \right) \\
\intertext{Ausklammern von $n$ in Zähler und Nenner}
&=& \underset{n \rightarrow \infty}{\text{lim}} \left( \text{cos} \left( \frac{n \cdot (\sqrt{10 - \frac{1}{n}} - 1)}{n \cdot (2 + \frac{3}{n})} \right) \right) \\
\intertext{Kürzen von $n$}
&=& \underset{n \rightarrow \infty}{\text{lim}} \left( \text{cos} \left( \frac{\sqrt{10 - \frac{1}{n}} - 1}{2 + \frac{3}{n}} \right) \right) \\
\intertext{cos mit lim vertauschen, da Cosinusfunktion stetig}
&=& \text{cos} \left( \underset{n \rightarrow \infty}{\text{lim}} \left( \frac{\sqrt{10 - \frac{1}{n}} - 1}{2 + \frac{3}{n}} \right) \right) \\
\intertext{lim in Wurzel ziehen, da Wurzelfunktion stetig}
&=& \text{cos} \left( \frac{\sqrt{\underset{n \rightarrow \infty}{\text{lim}} (10 - \frac{1}{n})} - \underset{n \rightarrow \infty}{\text{lim}} (1) }{\underset{n \rightarrow \infty}{\text{lim}} (2 + \frac{3}{n})} \right) \\
&=& \text{cos} \left( \frac{\sqrt{10} - 1}{2} \right)\\
&\approx & 0.47
\end{alignat*}
\section{} %3
$g \circ f$ kann auch so geschrieben werden $g(f(x))$. Vereinfacht gesagt, liefert $g$ den Funktionswert an der Stelle, die dem Funktionswert von $f$ an der Stelle $x$ entspricht.\\
Das berücksichtigend wissen wir, dass $f$ an der Stelle $x_{0}$ stetig ist. Der Funktionswert für diese Stelle ist $f(x_{0}) = y_{0}$. Wir wissen ferner, dass $g$ an der Stelle $y_{0}$ stetig ist.\\
\\
Da der Funktionswert von $f$ an der Stelle $x_{0}$ der Stelle entspricht, an der $g$ bekanntermaßen stetig ist, werden hier zwei stetige Funktionen nacheinander ausgeführt. Und die Nacheinanderausführung von zwei stetigen Funktionen ist selbst wiederum stetig.\\
Es ist somit ersichtlich, dass $g(f(x_{0}))$ den soeben beschrieben Fall darstellt und damit klarstellt, dass $g \circ f$ ebenfalls an der Stelle $x_{0}$ stetig ist.
\section{} %4
$g(x)$:\\
\begin{alignat*}{2}
\underset{x \rightarrow 0}{\text{lim}} g(x) &\Rightarrow & \underset{x \rightarrow 0}{\text{lim}} (x^{2} \cdot 1 ) \geq \underset{x \rightarrow 0}{\text{lim}} \left(x^{2} \cdot \text{sin} \left(\frac{1}{x}\right) \right) \geq \underset{x \rightarrow 0}{\text{lim}} (x^{2} \cdot -1) \\
&\Rightarrow & 0 \geq \underset{x \rightarrow 0}{\text{lim}} \left(x^{2} \cdot \text{sin} \left(\frac{1}{x} \right) \right) \geq 0
\end{alignat*}
Daraus folgt, dass $g(x)$ für alle $x \in \mathbb{R}$ stetig ist. \\
$h(x)$:\\
\begin{alignat*}{2}
\underset{n \rightarrow \infty}{\text{lim}} h(x_{n}) &=& \underset{n \rightarrow \infty}{\text{lim}} \text{sin} \left( \frac{1}{x_{n}} \right) \\
\intertext{$x_{n}$ sei $\frac{1}{2\pi n}, n \in \mathbb{N}$}
&=& \underset{n \rightarrow \infty}{\text{lim}} \text{sin} \left(\frac{1}{\frac{1}{2 \pi n}}\right) \\
&=& \underset{n \rightarrow \infty}{\text{lim}} \text{sin} (2 \pi n) \\
&=& \text{sin} \left( \underset{n \rightarrow \infty}{\text{lim}} (2 \pi n) \right) = 0
\end{alignat*}
$h(x)$ ist stetig für alle $x = x_{n} = \frac{1}{2\pi n}$. Die Funktion ist nicht stetig für andere $x$.
\end{document}

View File

@ -0,0 +1,284 @@
\documentclass[10pt,a4paper,oneside,ngerman,numbers=noenddot]{scrartcl}
\usepackage[T1]{fontenc}
\usepackage[utf8]{inputenc}
\usepackage[ngerman]{babel}
\usepackage{amsmath}
\usepackage{amsfonts}
\usepackage{amssymb}
\usepackage{paralist}
\usepackage{gauss}
\usepackage{pgfplots}
\usepackage[locale=DE,exponent-product=\cdot,detect-all]{siunitx}
\usepackage{tikz}
\usetikzlibrary{matrix,fadings,calc,positioning,decorations.pathreplacing,arrows,decorations.markings}
\usepackage{polynom}
\polyset{style=C, div=:,vars=x}
\pagenumbering{arabic}
\def\thesection{\arabic{section})}
\def\thesubsection{\alph{subsection})}
\def\thesubsubsection{(\roman{subsubsection})}
\makeatletter
\renewcommand*\env@matrix[1][*\c@MaxMatrixCols c]{%
\hskip -\arraycolsep
\let\@ifnextchar\new@ifnextchar
\array{#1}}
\makeatother
\begin{document}
\author{Jim Martens (6420323)}
\title{Hausaufgaben zum 2. Mai}
\maketitle
\section{} %1
\subsection{} %a
\subsubsection{} %i
\begin{alignat*}{2}
f(x) &=& 7x^{5} + 3x^{3} + x + 1 \\
f'(x) &=& 35x^{4} + 9x^{2} + 1
\end{alignat*}
\subsubsection{} %ii
\begin{alignat*}{2}
f(x) &=& (3x^{7} - 4x^{3} + x^{2} - 3x + 1)^{8} \\
f'(x) &=& 8 \cdot (3x^{7} - 4x^{3} + x^{2} - 3x + 1)^{7} \cdot (21x^{6} - 12x^{2} + 2x - 3)
\end{alignat*}
\subsubsection{} %iii
\begin{alignat*}{2}
f(x) &=& (3x^{4} + 2x) \cdot \sqrt{x^{2} + 1} \\
f'(x) &=& (12x^{3} + 2) \cdot \sqrt{x^{2} + 1} + (3x^{4} + 2x) \cdot \frac{2x}{2 \cdot \sqrt{x^{2} + 1}}
\end{alignat*}
\subsubsection{} %iv
\begin{alignat*}{2}
f(x) &=& (x^{3} + 1) \cdot \ln (x^{4} + 3x^{2} + 1) \\
f'(x) &=& 3x \cdot \ln (x^{4} + 3x^{2} + 1) + (x^{3} + 1) \cdot \frac{4x^{3} + 6x}{x^{4} + 3x^{2} + 1}
\end{alignat*}
\subsubsection{} %v
\begin{alignat*}{2}
f(x) &=& e^{x^{3} + x^{2} + 1} \cdot \sqrt{x} \\
f'(x) &=& e^{x^{3} + x^{2} + 1} \cdot (3x^{2} + 2x) \cdot \sqrt{x} + e^{x^{3} + x^{2} + 1} \cdot \frac{1}{2 \cdot \sqrt{x}}
\end{alignat*}
\subsubsection{} %vi
\begin{alignat*}{2}
f(x) &=& \sqrt{x^{4} + 1} \cdot \ln x \\
f'(x) &=& \frac{4x^{3}}{2 \cdot \sqrt{x^{4} + 1}} \cdot \ln x + \sqrt{x^{4} + 1} \cdot \frac{1}{x}
\end{alignat*}
\subsection{} %b
\begin{alignat*}{2}
q(x) &=& \frac{5x^{2} + 1}{x - 3} \\
q'(x) &=& \frac{10x \cdot (x-3) - (5x^{2} + 1) \cdot 1}{(x-3)^{2}} \\
\intertext{Ausmultiplizieren}
&=& \frac{10x^{2} - 30x - (5x^{2} + 1)}{x^{2} - 6x + 9}\\
\intertext{Zusammenfassen}
&=& \frac{5x^{2} - 30x - 1}{x^{2} - 6x + 9}\\
q''(x) &=& \frac{(10x-30) \cdot (x^{2} - 6x + 9) - (5x^{2} - 30x - 1) \cdot (2x - 6)}{(x^{2} - 6x + 9)^{2}} \\
\intertext{Ausmultiplizieren}
&=& \frac{10x^{3} - 60x^{2} + 90x - 30x^{2} + 180x - 270 - (10x^{3} - 30x^{2} - 60x^{2} + 180x -x^{2} - 9)}{x^{4} - 6x^{3} + 9x^{2} - 6x^{3} + 36x^{2} - 54x + 9x^{2} - 54x + 81} \\
\intertext{Zusammenfassen}
&=& \frac{10x^{3} - 90x^{2} + 270x - 270 -10x^{3} + 91x^{2} - 180x + 9}{x^{4} - 12x^{3} + 54x^{2} - 108x + 81} \\
\intertext{Zusammenfassen}
&=& \frac{x^{2} + 90x - 261}{x^{4} - 12x^{3} + 54x^{2} - 108x + 81} \\
q'''(x) &=& \frac{(2x + 90) \cdot (x^{4} - 12x^{3} + 54x^{2} - 108x + 81) - (x^{2} + 90x - 261) \cdot (4x^{3} - 36x^{2} + 108x - 108)}{(x^{4} - 12x^{3} + 54x^{2} - 108x + 81)^{2}} \\
\intertext{Ausmultiplizieren und Zusammenfassen}
&=& \frac{-2x^{5} - 258x^{4} + 3204x^{3} - 14364x^{2} +28350x - 20898}{x^{8} - 24x^{7} + 252x^{6} -1404x^{5} +5670x^{4} - 13716x^{3} + 20412x^{2} -8748x - 2187}
\end{alignat*}
\section{} %2
\begin{alignat*}{2}
f(x) &=& \left| 3 - \frac{1}{2}x \right| \\
\intertext{Sei $x_{n} = 6 + \frac{1}{n} : n \in \mathbb{N}$}
\underset{n \rightarrow \infty}{\text{lim}} \frac{f(x_{n}) - f(x_{0})}{x_{n} - x_{0}} &=& \underset{n \rightarrow \infty}{\text{lim}} \frac{\left| 3 - \frac{1}{2} \cdot (6 + \frac{1}{n}) \right| - \left| 3 - \frac{1}{2} \cdot 6 \right|}{6 + \frac{1}{n} - 6} \\
&=& \underset{n \rightarrow \infty}{\text{lim}} \frac{\left| 3 - 3 - \frac{1}{2n} \right| - \left| 3 - 3 \right|}{\frac{1}{n}} \\
&=& \underset{n \rightarrow \infty}{\text{lim}} \frac{\left|-\frac{1}{2n} \right|}{\frac{1}{n}} \\
&=& \underset{n \rightarrow \infty}{\text{lim}} \frac{\frac{1}{2n}}{\frac{1}{n}} \\
&=& \underset{n \rightarrow \infty}{\text{lim}} \frac{n}{2n} = \frac{1}{2} \\
\intertext{Sei $x_{n} = 6 - \frac{1}{n} : n \in \mathbb{N}$}
\underset{n \rightarrow \infty}{\text{lim}} \frac{f(x_{n}) - f(x_{0})}{x_{n} - x_{0}} &=& \underset{n \rightarrow \infty}{\text{lim}} \frac{\left| 3 - \frac{1}{2} \cdot (6 - \frac{1}{n}) \right| - \left| 3 - \frac{1}{2} \cdot 6 \right|}{6 - \frac{1}{n} - 6} \\
&=& \underset{n \rightarrow \infty}{\text{lim}} \frac{\left| 3 - 3 + \frac{1}{2n} \right| - \left| 3 - 3 \right|}{-\frac{1}{n}} \\
&=& \underset{n \rightarrow \infty}{\text{lim}} \frac{\left|\frac{1}{2n} \right|}{-\frac{1}{n}} \\
&=& \underset{n \rightarrow \infty}{\text{lim}} \frac{\frac{1}{2n}}{-\frac{1}{n}} \\
&=& \underset{n \rightarrow \infty}{\text{lim}} -\frac{n}{2n} = -\frac{1}{2}
\end{alignat*}
Der Grenzwert existiert an der Stelle $x _{0} = 6$ nicht. Daher ist die Funktion $f$ an der Stelle $x_{0} = 6$ nicht differenzierbar.
\begin{tikzpicture}[>=stealth]
\begin{axis}[
ymin=-10,ymax=10,
x=1em,
y=1em,
axis x line=middle,
axis y line=middle,
axis line style=->,
xlabel={$x$},
ylabel={$y$},
]
\addplot[no marks, black, -] expression[domain=-10:6,samples=100]{3 - (1/2)*x} node[pos=0.65,anchor=north]{};
\addplot[no marks, black, -] expression[domain=6:10,samples=100]{-1*(3 - (1/2)*x)} node[pos=0.65,anchor=north]{};
\draw (16em, 10em) circle (2pt);
\end{axis}
\end{tikzpicture}
\section{} %3
\subsection{} %a
\begin{alignat*}{2}
f(x) &=& (x^{4}+1)^{x+2} \\
&=& e^{ln\left((x^{4}+1)^{x+2}\right)} \\
&=& e^{(x+2) \cdot \ln(x^{4}+1)} \\
f'(x) &=& e^{(x+2) \cdot \ln(x^{4}+1)} \cdot \left((x+2) \cdot \ln(x^{4}+1)\right)' \\
&=& e^{(x+2) \cdot \ln(x^{4}+1)} \cdot \left((x+2)' \cdot \ln(x^{4}+1) + (x+2) \cdot \ln(x^{4}+1)'\right) \\
&=& e^{(x+2) \cdot \ln(x^{4}+1)} \cdot \left(1 \cdot \ln(x^{4}+1) + (x+2) \cdot \frac{1}{x^{4}+1} \cdot (x^{4}+1)'\right) \\
&=& (x^{4}+1)^{x+2} \cdot \left(\ln(x^{4}+1) + (x+2) \cdot \frac{4x^{3}}{x^{4}+1}\right)
\end{alignat*}
\subsection{} %b
\begin{alignat*}{2}
f(x) &=& x^{\frac{1}{2}} \\
&=& e^{\ln \left(x^{\frac{1}{2}} \right)} \\
&=& e^{\frac{1}{2} \cdot \ln (x)} \\
f'(x) &=& e^{\frac{1}{2} \cdot \ln (x)} \cdot \left(\frac{1}{2} \cdot \ln (x) \right)' \\
&=& e^{\frac{1}{2} \cdot \ln (x)} \cdot \frac{1}{2} \cdot \frac{1}{x} \cdot (x)' \\
&=& e^{\frac{1}{2} \cdot \ln (x)} \cdot \frac{1}{2x} \\
&=& x^{\frac{1}{2}} \cdot \frac{1}{2x} \\
&=& \frac{1}{2} \cdot \frac{x^{\frac{1}{2}}}{x} \\
&=& \frac{1}{2} \cdot x^{-\frac{1}{2}} \\
g(x) &=& \left( \frac{1}{2}\right)^{x} \\
&=& e^{\ln \left( \left( \frac{1}{2}\right)^{x} \right)} \\
&=& e^{x \cdot \ln \left( \frac{1}{2}\right)} \\
g'(x) &=& e^{x \cdot \ln \left( \frac{1}{2}\right)} \cdot \left(x \cdot \ln \left( \frac{1}{2}\right) \right)' \\
&=& e^{x \cdot \ln \left( \frac{1}{2}\right)} \cdot (x)' \cdot \ln \left( \frac{1}{2}\right) \\
&=& e^{x \cdot \ln \left( \frac{1}{2}\right)} \cdot 1 \cdot \ln \left( \frac{1}{2}\right) \\
&=& \left( \frac{1}{2}\right)^{x} \cdot \ln \left( \frac{1}{2}\right)
\end{alignat*}
\subsection{} %c
\subsubsection{} %i
\begin{alignat*}{2}
g(x) &=& (x^{2}+1)^{4x+1} \\
&=& e^{\ln \left( (x^{2}+1)^{4x+1}\right)} \\
&=& e^{(4x+1) \cdot \ln (x^{2}+1)} \\
g'(x) &=& e^{(4x+1) \cdot \ln (x^{2}+1)} \cdot \left((4x+1) \cdot \ln (x^{2}+1) \right)' \\
&=& e^{(4x+1) \cdot \ln (x^{2}+1)} \cdot \left((4x+1)' \cdot \ln (x^{2}+1) + (4x+1) \cdot \ln (x^{2}+1)' \right) \\
&=& e^{(4x+1) \cdot \ln (x^{2}+1)} \cdot \left(4 \cdot \ln (x^{2}+1) + (4x+1) \cdot \frac{1}{x^{2}+1} \cdot (x^{2}+1)' \right) \\
&=& e^{(4x+1) \cdot \ln (x^{2}+1)} \cdot \left(4 \cdot \ln (x^{2}+1) + (4x+1) \cdot \frac{2x}{x^{2}+1} \right) \\
&=& (x^{2}+1)^{4x+1} \cdot \left(4 \cdot \ln (x^{2}+1) + \frac{8x^{2} + 2x}{x^{2}+1} \right)
\end{alignat*}
\subsubsection{} %ii
\begin{alignat*}{2}
h(x) = (x-3)^{3x^{4}+5} \\
&=& e^{\ln \left((x-3)^{3x^{4}+5} \right)} \\
&=& e^{(3x^{4}+5) \cdot \ln (x-3)} \\
h'(x) &=& e^{(3x^{4}+5) \cdot \ln (x-3)} \cdot \left((3x^{4}+5) \cdot \ln (x-3) \right)' \\
&=& e^{(3x^{4}+5) \cdot \ln (x-3)} \cdot \left((3x^{4}+5)' \cdot \ln (x-3) + (3x^{4}+5) \cdot \ln (x-3)' \right) \\
&=& e^{(3x^{4}+5) \cdot \ln (x-3)} \cdot \left(12x^{3} \cdot \ln (x-3) + (3x^{4}+5) \cdot \frac{1}{x-3} \cdot (x-3)' \right) \\
&=& e^{(3x^{4}+5) \cdot \ln (x-3)} \cdot \left(12x^{3} \cdot \ln (x-3) + (3x^{4}+5) \cdot \frac{1}{x-3} \right) \\
&=& (x-3)^{3x^{4}+5} \cdot \left(12x^{3} \cdot \ln (x-3) + \frac{3x^{4}+5}{x-3} \right)
\end{alignat*}
\section{} %4
\subsection{} %a
\begin{alignat*}{2}
g(p) &=& 10^{5} \cdot \left( \frac{1}{p} - \frac{3}{p^{2}} \right) \\
&=& 10^{5} \cdot \left( p^{-1} - 3p^{-2} \right) \\
g'(p) &=& 10^{5} \cdot \left( p^{-1} - 3p^{-2} \right)' \\
&=& 10^{5} \cdot \left((p^{-1})' - (3p^{-2})' \right) \\
&=& 10^{5} \cdot \left(-p^{-2} - (-6p^{-3}) \right) \\
&=& 10^{5} \cdot \left(-p^{-2} + 6p^{-3} \right) \\
&=& 10^{5} \cdot \left(-\frac{1}{p^{2}} + \frac{6}{p^{3}} \right) \\
g''(p) &=& 10^{5} \cdot \left(-p^{-2} + 6p^{-3} \right)' \\
&=& 10^{5} \cdot \left((-p^{-2})' + (6p^{-3})' \right) \\
&=& 10^{5} \cdot \left(2p^{-3} - 18p^{-4} \right) \\
&=& 10^{5} \cdot \left(\frac{2}{p^{3}} - \frac{18}{p^{4}} \right)
\end{alignat*}
\begin{enumerate}
\item Bestimmung der Nullstellen der 1. Ableitung:
\begin{alignat*}{2}
-\frac{1}{p^{2}} + \frac{6}{p^{3}} &=& 0 \\
-\frac{1}{p^{2}} &=& - \frac{6}{p^{3}} \\
\frac{1}{p^{2}} &=& \frac{6}{p^{3}} \\
\frac{p^{3}}{p^{2}} &=& 6 \\
p &=& 6 \\
\intertext{Einsetzen in $g'(p)$}
0 &=& 10^{5} \cdot \left(-\frac{1}{6^{2}} + \frac{6}{6^{3}} \right) \\
&=& 10^{5} \cdot \left(-\frac{1}{36} + \frac{6}{216} \right) \\
&=& 10^{5} \cdot \left(-\frac{1}{36} + \frac{1}{36} \right) \\
&=& 10^{5} \cdot 0 = 0
\end{alignat*} \\
\item Einsetzen von $p=6$ in $g''(p)$:
\begin{alignat*}{2}
g''(6) &=& 10^{5} \cdot \left(\frac{2}{6^{3}} - \frac{18}{6^{4}} \right) \\
&=& 10^{5} \cdot \left(\frac{2}{216} - \frac{18}{1296} \right) \\
&=& 10^{5} \cdot \left(\frac{2}{216} - \frac{3}{216} \right) \\
&=& 10^{5} \cdot \left(- \frac{1}{216} \right) = -\frac{10^{5}}{216}
\end{alignat*}
Daraus ergibt sich, dass $p=6$ ein lokales Maximum der Funktion $g(p)$ ist. Es bleibt noch festzustellen, dass es auch das globale Maximum ist. \\
\item Feststellung des globalen Maximums:\\
Da es nur eine Nullstelle für die erste Ableitung gibt, kann es insofern nur einen Extrempunkt geben. Da der Funktionswert der 2. Ableitung für diese Stelle negativ ist, liegt an der Stelle ein Maximum. Der Funktionswert von $g(p)$ für die Stelle wird wie folgt berechnet:
\begin{alignat*}{2}
g(p) &=& 10^{5} \cdot \left(\frac{1}{p} - \frac{3}{p^{2}} \right) \\
g(6) &=& 10^{5} \cdot \left( \frac{1}{6} - \frac{3}{3^{2}} \right) \\
&=& 10^{5} \cdot \left( \frac{1}{6} - \frac{3}{9} \right) \\
&=& 10^{5} \cdot \left( \frac{1}{6} - \frac{2}{6} \right) \\
&=& 10^{5} \cdot \left(- \frac{1}{6} \right) \\
&=& -\frac{10^{5}}{6} = \frac{5000}{3} \\
&=& \frac{4998}{3} + \frac{2}{3} \\
&=& 1666 + \frac{2}{3}
\end{alignat*} \\\\\\
Zuletzt muss noch geprüft werden, ob die Grenzen des Definitionsbereiches einen höheren Wert aufweisen.
\begin{alignat*}{2}
g(p) &=& 10^{5} \cdot \left(\frac{1}{p} - \frac{3}{p^{2}} \right) \\
g(3) &=& 10^{5} \cdot \left(\frac{1}{3} - \frac{3}{3^{2}} \right) \\
&=& 10^{5} \cdot \left(\frac{1}{3} - \frac{3}{9} \right) \\
&=& 10^{5} \cdot \left(\frac{1}{3} - \frac{1}{3} \right) \\
&=& 10^{5} \cdot 0 = 0 \\
g(100) &=& 10^{5} \cdot \left(\frac{1}{100} - \frac{3}{100^{2}} \right) \\
&=& 10^{5} \cdot \left(\frac{1}{100} - \frac{3}{10000} \right) \\
&=& 10^{5} \cdot \left(\frac{100}{10000} - \frac{3}{10000} \right) \\
&=& 10^{5} \cdot \frac{97}{10000} \\
&=& 970
\end{alignat*}
Die Funktion hat nur einen Extrempunkt und die beiden Definitionsgrenzen weisen einen niedrigeren Funktionswert auf, als der zuvor bestimmte lokale Extrempunkt mit $p = 6$. Somit ist der einzige lokale Extrempunkt auch der globale Extrempunkt. Da es sich bei dem Extrempunkt um ein Maximum handelt, ist er somit das globale Maximum.
\end{enumerate}
\subsection{} %b
\subsubsection{} %i
Bestimmung der Nullstellen von $f, f'$ und $f''$:\\
\begin{alignat*}{2}
f(x) &=& -2x^{3} - x + 25 \\
f'(x) &=& -6x^{2} - 1 \\
f''(x) &=& -18x \\
f'''(x) &=& -18 \\
f(2) &=& -2 \cdot 2^{3} - 2 + 25 \\
&=& -2 \cdot 8 +23 \\
&=& -16 + 23 = 7 \\
f(3) &=& -2 \cdot 3^{3} - 3 + 25 \\
&=& -2 \cdot 27 + 22 \\
&=& -54 + 22 \\
&=& -32 \\
f'(0) &=& -6 \cdot 0^{2} - 1 \\
&=& 0 -1 = -1 \\
f''(0) &=& -18 \cdot 0 = 0
\end{alignat*}
Die Nullstelle von $f(x)$ liegt zwischen $x=2$ und $x=3$. $f'(x)$ hat keine Nullstelle. Der höchste erreichbare Wert ist $-1$.
$f''(x)$ hat eine Nullstelle, welche nachweislich bei $x=0$ liegt.
$f'''(x)$ hat offensichtlich keine Nullstelle.\\
\\
$f(x) > 0$ für $x \leq 2$ und $f(x) < 0$ für $x \geq 3$. $f'(x) < 0$ für alle $x \in \mathbb{R} \wedge x \geq -5 \wedge x \leq 5$. $f''(x) < 0$ für $x < 0$ und $f''(x) > 0$ für $x > 0$. $f'''(x) > 0$ für alle $x \in \mathbb{R} \wedge x \geq -5 \wedge x \leq 5$. \\
Es gibt keine Maxima oder Minima. Allerdings gibt es einen Wendepunkt bei $x=0$, da $f''(0) = 0$ und $f'''(0) \neq 0$. \\
Da es keine Nullstellen der ersten Ableitung gibt und $f(2) > f(3)$ gilt, ist $f(x)$ auf dem gesamten Intervall streng monoton fallend. Der höchste Punkt ist demnach am Beginn des Definitionsbereiches, der niedrigste am Ende.
Daraus ergibt sich:\\
\begin{alignat*}{2}
f(-5) &=& -2 \cdot (-5)^{3} - (-5) + 25 \\
&=& -2 \cdot (-125) + 30 \\
&=& 250 + 30 = 280 \\
f(5) &=& -2 \cdot 5^{3} - 5 + 25 \\
&=& -2 \cdot 125 + 20 \\
&=& -250 + 20 = -230
\end{alignat*}
$f(x)$ nimmt das globale Maximum bei $x=-5$ und das globale Minimum bei $x=5$ an.
\subsubsection{} %ii
\begin{alignat*}{2}
g(x) &=& x^{3} - 6x^{2} + 3x + 8 \\
g'(x) &=& 3x^{2} - 12x + 3 \\
g''(x) &=& 6x - 12 \\
g'''(x) &=& 6
\end{alignat*}
\subsubsection{} %iii
\begin{alignat*}{2}
h(x) &=& e^{2x-3} - e^{x+2} \\
h'(x) &=& e^{2x-3} \cdot 2 - e^{x+2} \\
h''(x) &=& e^{2x-3} \cdot 4 - e^{x+2} \\
h'''(x) &=& e^{2x-3} \cdot 8 - e^{x+2}
\end{alignat*}
\end{document}

View File

@ -0,0 +1,349 @@
\documentclass[10pt,a4paper,oneside,ngerman,numbers=noenddot]{scrartcl}
\usepackage[T1]{fontenc}
\usepackage[utf8]{inputenc}
\usepackage[ngerman]{babel}
\usepackage{amsmath}
\usepackage{amsfonts}
\usepackage{amssymb}
\usepackage{paralist}
\usepackage{gauss}
\usepackage{pgfplots}
\usepackage[locale=DE,exponent-product=\cdot,detect-all]{siunitx}
\usepackage{tikz}
\usetikzlibrary{matrix,fadings,calc,positioning,decorations.pathreplacing,arrows,decorations.markings}
\usepackage{polynom}
\polyset{style=C, div=:,vars=x}
\pagenumbering{arabic}
\def\thesection{\arabic{section})}
\def\thesubsection{\alph{subsection})}
\def\thesubsubsection{(\roman{subsubsection})}
\makeatletter
\renewcommand*\env@matrix[1][*\c@MaxMatrixCols c]{%
\hskip -\arraycolsep
\let\@ifnextchar\new@ifnextchar
\array{#1}}
\makeatother
\begin{document}
\author{Jim Martens (6420323)}
\title{Hausaufgaben zum 16. Mai}
\maketitle
\section{} %1
\subsubsection{} %i
\begin{alignat*}{2}
f(x) &=& \frac{1}{\sqrt[4]{x^{5}}} \cdot \sqrt[3]{\sqrt{x^{7}}} \\
\intertext{Wurzeln umformen}
&=& \frac{1}{(x^{5})^{\frac{1}{4}}} \cdot (\sqrt{x^{7}})^{\frac{1}{3}} \\
\intertext{Wurzeln umformen}
&=& \frac{1}{(x^{5})^{\frac{1}{4}}} \cdot ((x^{7})^{\frac{1}{2}})^{\frac{1}{3}} \\
\intertext{Zusammenfassen}
&=& \frac{1}{x^{\frac{5}{4}}} \cdot x^{\frac{7}{6}} \\
\intertext{Umformen}
&=& x^{-\frac{5}{4}} \cdot x^{\frac{7}{6}} \\
\intertext{Zusammenfassen}
&=& x^{\frac{7}{6} - \frac{5}{4}} \\
\intertext{Auf selben Nenner bringen}
\intertext{Zusammenfassen}
&=& x^{\frac{14 - 15}{12}} \\
&=& x^{-\frac{1}{12}} \\
f'(x) &=& -\frac{1}{12}x^{-\frac{13}{12}}
\end{alignat*}
\subsubsection{} %ii
\begin{alignat*}{2}
f(x) &=& \sin (x^{2}) \\
f'(x) &=& \cos (x^{2}) \cdot 2x
\end{alignat*}
\subsubsection{} %iii
\begin{alignat*}{2}
f(x) &=& \sin ^{2} x \\
&=& \sin x \cdot \sin x \\
f'(x) &=& \cos x \cdot \sin x + \sin x \cdot \cos x
\end{alignat*}
\subsubsection{} %iv
\begin{alignat*}{2}
f(x) &=& \sin x \cdot \cos x \\
f'(x) &=& \cos x \cdot \cos x + \sin x \cdot (- \sin x) \\
&=& \cos x \cdot \cos x - \sin x \cdot + \sin x)
\end{alignat*}
\subsubsection{} %v
\begin{alignat*}{2}
f(x) &=& \arcsin (\sqrt{x}) \\
f'(x) &=& \frac{1}{\sqrt{1 - \sqrt{x}}} \cdot (\sqrt{x})' \\
&=& \frac{1}{\sqrt{1 - \sqrt{x}}} \cdot \frac{1}{2 \sqrt{x}}
\end{alignat*}
\subsubsection{} %vi
\begin{alignat*}{2}
f(x) &=& (x^{3}-1)^{\arctan x} \\
&=& e^{\ln \left((x^{3}-1)^{\arctan x} \right)} \\
&=& e^{\left(\arctan x \right) \cdot \ln \left(x^{3}-1 \right)} \\
f'(x) &=& e^{\left(\arctan x \right) \cdot \ln \left(x^{3}-1 \right)} \cdot \left( \left(\arctan x \right) \cdot \ln \left(x^{3}-1 \right)\right)' \\
&=& e^{\left(\arctan x \right) \cdot \ln \left(x^{3}-1 \right)} \cdot \left( \left(\arctan x \right)' \cdot \ln \left(x^{3}-1 \right) + \left(\arctan x \right) \cdot \ln \left(x^{3}-1 \right)' \right) \\
&=& e^{\left(\arctan x \right) \cdot \ln \left(x^{3}-1 \right)} \cdot \left( \frac{1}{\sqrt{1 - x}} \cdot \ln \left(x^{3}-1 \right) + \left(\arctan x \right) \cdot \frac{1}{x^{3}-1} \cdot (x^{3}-1)' \right) \\
&=& e^{\left(\arctan x \right) \cdot \ln \left(x^{3}-1 \right)} \cdot \left( \frac{1}{\sqrt{1 - x}} \cdot \ln \left(x^{3}-1 \right) + \left(\arctan x \right) \cdot \frac{3x^{2}}{x^{3}-1}\right) \\
&=& (x^{3}-1)^{\arctan x} \cdot \left( \frac{1}{\sqrt{1 - x}} \cdot \ln \left(x^{3}-1 \right) + \left(\arctan x \right) \cdot \frac{3x^{2}}{x^{3}-1}\right)
\end{alignat*}
\section{} %2
\begin{alignat*}{2}
f(x) &=& \frac{2x}{1+x^{2}} \\
f'(x) &=& \frac{2 \cdot (1+x^{2}) - 2x \cdot 2x}{\left(1+x^{2} \right)^{2}} \\
&=& \frac{2 \cdot (1+x^{2}) - 4x^{2}}{\left(1+x^{2} \right)^{2}} \\
f''(x) &=& \frac{\left(2 \cdot (1+x^{2}) - 4x^{2} \right)' \cdot \left(1+x^{2} \right)^{2} - ((2 \cdot (1+x^{2}) - 4x^{2}) \cdot \left(\left(1+x^{2} \right)^{2} \right)'}{\left(1+x^{2} \right)^{4}} \\
&=& \frac{(4x-8x) \cdot \left(1+x^{2} \right)^{2} - (2 \cdot (1+x^{2}) - 4x^{2}) \cdot 2 \cdot (1+x^{2}) \cdot 2x}{\left(1+x^{2} \right)^{4}} \\
\intertext{$(1+x^{2})$ ausklammern und kürzen}
&=& \frac{-4x \cdot \left(1+x^{2} \right) - (2 \cdot (1+x^{2}) - 4x^{2}) \cdot 2 \cdot 2x}{\left(1+x^{2} \right)^{3}} \\
&=& \frac{4x \cdot \left( -\left(1+x^{2} \right) - (2 \cdot (1+x^{2}) - 4x^{2})\right)}{\left(1+x^{2} \right)^{3}} \\
&=& -\frac{4x \cdot \left(1+x^{2} + (2 \cdot (1+x^{2}) - 4x^{2}\right)}{\left(1+x^{2} \right)^{3}} \\
\intertext{Zusammenfassen}
&=& -\frac{4x \cdot \left(3-x^{2}\right)}{\left(1+x^{2} \right)^{3}}
\end{alignat*}
\begin{enumerate}
\item $x \in \mathbb{R}$
\item
\begin{alignat*}{2}
f(x) &=& 0 \\
\intertext{$f(x)$ mit Zähler ersetzen, da der über Nullstelle bestimmt}
2x &=& 0 \\
\intertext{geteilt durch $2$}
x &=& 0 \\
f'(x) &=& 0 \\
\intertext{$f'(x)$ mit Zähler ersetzen, da der über Nullstelle bestimmt}
2 - 2x^{2} &=& 0 \\
\intertext{$2$ subtrahieren}
-2x^{2} &=& -2 \\
\intertext{geteilt durch $-2$}
x^{2} &=& 1 \\
\intertext{Wurzel ziehen}
x_{1} &=& 1 \\
x_{2} &=& -1 \\
f''(x) &=& 0 \\
\intertext{$f''(x)$ mit Zähler ersetzen, da der über Nullstelle bestimmt}
-4x \cdot \left(3-x^{2}\right) &=& 0 \\
&\Rightarrow & x_{1} = 0, x_{2} = \sqrt{3}, x_{3} = - \sqrt{3}
\end{alignat*}
\item Es gibt keine Randpunkte des Definitionsbereiches.
\item
\begin{alignat*}{2}
f(-1) &=& \frac{2\cdot (-1)}{1 + (-1)^{2}} \\
&=& \frac{-2}{2} = -1 \\
f(1) &=& \frac{2 \cdot 1}{1 + 1^{2}} \\
&=& \frac{2}{2} = 1 \\
f'(-2) &=& \frac{2 - 2 \cdot (-2)^{2}}{\left(1+ (-2)^{2} \right)^{2}} \\
&=& \frac{2 - 2 \cdot 4}{\left(1+ 4 \right)^{2}} \\
&=& \frac{-6}{\left(5 \right)^{2}} \\
&=& \frac{-6}{25} \\
f'(0) &=& \frac{2 - 2 \cdot 0^{2}}{\left(1+ 0^{2} \right)^{2}} \\
&=& \frac{2}{1} = 2 \\
f'(2) &=& \frac{2 - 2 \cdot 2^{2}}{\left(1+ 2^{2} \right)^{2}} \\
&=& \frac{2 - 8}{\left(5 \right)^{2}} \\
&=& \frac{-6}{25}
\end{alignat*}\\
\begin{alignat*}{2}
f''(-2) &=& -\frac{4 \cdot (-2) \cdot \left(3- (-2)^{2}\right)}{\left(1+ (-2)^{2} \right)^{3}} \\
&=& -\frac{-8 \cdot \left(3- 4 \right)}{\left(1+ 4 \right)^{3}} \\
&=& -\frac{-8 \cdot -1}{5^{3}} \\
&=& -\frac{8}{125} \\
f''(-1) &=& -\frac{4 \cdot (-1) \cdot \left(3- (-1)^{2}\right)}{\left(1+ (-1)^{2} \right)^{3}} \\
&=& -\frac{-4 \cdot \left(3- 1\right)}{\left(1+ 1 \right)^{3}} \\
&=& -\frac{-4 \cdot 2}{2^{3}} \\
&=& -\frac{-8}{8} \\
&=& 1 \\
f''(1) &=& -\frac{4 \cdot 1 \cdot \left(3- 1^{2}\right)}{\left(1+ 1^{2} \right)^{3}} \\
&=& -\frac{4 \cdot 2}{2^{3}} \\
&=& -\frac{8}{8} \\
&=& -1 \\
f''(2) &=& -\frac{4 \cdot 2 \cdot \left(3- 2^{2}\right)}{\left(1+ 2^{2} \right)^{3}} \\
&=& -\frac{8 \cdot \left(3- 4\right)}{\left(1+ 4 \right)^{3}} \\
&=& -\frac{8 \cdot -1}{5^{3}} \\
&=& -\frac{-8}{125} \\
&=& \frac{8}{125}
\end{alignat*}\\
$f(x)$ ist negativ für $x < 0$ und positiv für $x > 0$. $f$ ist streng monoton fallend für $x < -1$, streng monoton steigend für $-1 < x < 1$ und streng monoton fallend für $x > 1$. $f$ ist streng konkav für $x < -\sqrt{3}$, streng konvex für $- \sqrt{3} < x < 0$, streng konkav für $0 < x < \sqrt{3}$ und streng konvex für $x > \sqrt{3}$.
\label{enum:1}
\item Aus \ref{enum:1}\text{.} ergibt sich: $f$ hat ein Minimum an $x=-1$, ein Maximum an $x=1$, Wendepunkte an $x=-\sqrt{3}, x=0$ und $x=\sqrt{3}$.
\item Mithilfe von Satz 22 zeige ich, dass $f$ für $x \rightarrow \infty$ eine Asymptote $g(x) =ax +b$ besitzt und berechne diese:\\
\begin{alignat*}{2}
a = \lim_{x \rightarrow \infty} \left( \frac{f(x)}{x} \right) &=& \lim_{x \rightarrow \infty} \left(\frac{2x}{x+x^{3}} \right) \\
&=& \lim_{x \rightarrow \infty} \left(\frac{(2) \cdot x}{(\frac{1}{x^{2}}+1) \cdot x^{3}} \right) \\
&=& \lim_{x \rightarrow \infty} \left(\frac{2}{(\frac{1}{x^{2}}+1) \cdot x^{2}} \right) \\
\intertext{Der Nenner geht gegen unendlich, der Zähler ist konstant, daher geht der Ausdruck gegen Null.}
&=& 0\\
\\
b = \lim_{x \rightarrow \infty} \left( f(x) - ax \right) &=& \lim_{x \rightarrow \infty} \left( \frac{2x}{1+x^{2}} - 0 \right) \\
&=& \lim_{x \rightarrow \infty} \left( \frac{(2) \cdot x}{(\frac{1}{x^{2}}+1) \cdot x^{2}} \right) \\
\lim_{x \rightarrow \infty} \left( \frac{2}{(\frac{1}{x^{2}}+1) \cdot x} \right) \\
\intertext{Der Nenner geht gegen unendlich, der Zähler ist konstant, daher geht der Ausdruck gegen Null.}
&=& 0
\end{alignat*}\\
Also besitzt $f$ für $x \rightarrow \infty$ die Asymptote $g(x)=0$. Eine ähnliche Überlegung ergibt, dass $f$ für $x \rightarrow -\infty$ ebenfalls die Asymptote $g(x)=0$ besitzt.\\
Es gibt einen Schnittpunkt von $f$ mit der Asymptote bei $x=0$, da\\
\begin{alignat*}{2}
\frac{2x}{1+x^{2}} &=& 0 \\
\intertext{Mit $1+x^{2}$ multiplizieren geht, da der Term nie gleich 0 sein kann.}
2x &=& 0 \\
x &=& 0
\end{alignat*}\\
gilt.
\item
\begin{alignat*}{2}
f(0) &=& \frac{2 \cdot 0}{1 + 0^{2}} \\
&=& \frac{0}{1} = 0\\
f(-1) &=& \frac{2 \cdot -1}{1 + (-1)^{2}} \\
&=& \frac{-2}{2} = -1 \\
f(1) &=& \frac{2 \cdot 1}{1 + 1^{2}} \\
&=& \frac{2}{2} = 1 \\
f(-\sqrt{3}) &=& \frac{2 \cdot (-\sqrt{3})}{1 + (-\sqrt{3})^{2}} \\
&=& \frac{-2\sqrt{3}}{1 + 3} \\
&=& \frac{-2\sqrt{3}}{4} \\
&=& -\frac{\sqrt{3}}{2} \\
f(\sqrt{3}) &=& \frac{2 \cdot \sqrt{3}}{1 + \sqrt{3}^{2}} \\
&=& \frac{2\sqrt{3}}{1 + 3} \\
&=& \frac{2\sqrt{3}}{4} \\
&=& \frac{\sqrt{3}}{2}
\end{alignat*}
\begin{tikzpicture}[>=stealth]
\begin{axis}[
ymin=-5,ymax=5,
x=1em,
y=1em,
axis x line=middle,
axis y line=middle,
axis line style=->,
xlabel={$x$},
ylabel={$y$},
]
\addplot[no marks, black, -] expression[domain=-10:10,samples=100]{(2*x)/(1+(x*x))} node[pos=0.65,anchor=north]{};
\end{axis}
\end{tikzpicture}
\end{enumerate}
\section{} %3
\begin{alignat*}{2}
f(x) &=& 4x^{3} - 10x +5 \\
f'(x) &=& 12x^{2} - 10 \\
\intertext{Die Berechnung der Näherungswerte erfolgt durch folgende Formel:}
x_{n+1} &=& x_{n} - \frac{f(x_{n})}{f'(x_{n})} \\
\intertext{Exemplarische Berechnung von $f(x)$ für $x=2$:}
f(2) &=& 4 \cdot 2^{3} - 10 \cdot 2 +5 \\
&=& 4 \cdot 8 - 20 + 5 \\
&=& 32 -15 \\
&=& 17 \\
\intertext{Exemplarische Berechnung von $f'(x)$ für $x=2$:}
f'(2) &=& 12 \cdot 2^{2} -10 \\
&=& 48 - 10 \\
&=& 38 \\
\intertext{Berechnung der Näherungswerte:}
x_{1} &=& 2 - \frac{17}{38} \\
&=& \frac{76 -17}{38} \\
&=& \frac{59}{38} \\
x_{2} &=& \frac{59}{38} - \frac{f(\frac{59}{38})}{f'(\frac{59}{38})} \\
&\approx & 1.317784436 \\
x_{3} &\approx & 1.317784436 - \frac{f(1.317784436)}{f'(1.317784436)} \\
&\approx & 1.227756731 \\
x_{4} &\approx & 1.227756731 - \frac{f(1.227756731)}{f'(1.227756731)} \\
&\approx & 1.212272195 \\
x_{5} &\approx & 1.212272195 - \frac{f(1.212272195)}{f'(1.212272195)} \\
&\approx & 1.211811475 \\
x_{6} &\approx & 1.211811475 - \frac{f(1.211811475)}{f'(1.211811475)} \\
&\approx & 1.21181107 \\
x_{7} &=& 1.21181107 - \frac{f(1.21181107)}{f'(1.21181107)} \\
&=& 1.21181107
\end{alignat*}
\section{} %4
Die Länge der Schnur berechnet sie wie folgt:\\
\begin{alignat*}{2}
L &=& 2x + y \\
\intertext{Nach y umgestellt ergibt sich}
y &=& L - 2x \\
\intertext{Eingesetzt in die Formel für den Flächeninhalt von Rechtecken ergibt sich:}
A(x) &=& x \cdot (L-2x) \\
&=& -2x^{2} + Lx \\
A'(x) &=& -4x + L \\
A''(x) &=& -4 \\
\intertext{Gleichsetzen von $A'(x)$ mit Null}
0 &=& -4x + L \\
4x &=& L \\
x &=& \frac{L}{4} \\
\intertext{Einsetzen in zweite Ableitung}
A''(\frac{L}{4}) &=& -4 \Rightarrow \text{$< 0$, daher Maximum}
\end{alignat*}
Daraus folgt, dass die beiden gleichlangen Seiten des Rechtecks, die von der Schnur begrenzt werden, jeweils ein Viertel der Schnur ausmachen. Die dritte von der Schnur begrenzte Seite ist folglich halb so lang wie die Schnur.
\section{} %5
\subsection{} %a
Die Oberfläche ($A$) und das Volumen ($V$) der Dose sei durch folgende Formeln gegeben:\\
\begin{alignat*}{2}
A &=& 2\pi r^{2} + 2\pi rh \\
V &=& 1000 cm^{3} = \pi r^{2}h \\
\intertext{Umstellen von $V$ nach $h$}
h &=& \frac{1000 cm^{3}}{\pi r^{2}} \\
\intertext{Einsetzen in $A$}
A(r) &=& 2 \pi r^{2} + 2 \pi r \cdot \frac{1000 cm^{3}}{\pi r^{2}} \\
\intertext{Kürzen}
&=& 2 \pi r^{2} + 2 \cdot \frac{1000 cm^{3}}{r} \\
\intertext{Zusammenfassen}
&=& 2 \pi r^{2} + 2000 cm^{3} \cdot r^{-1} \\
A'(r) &=& 4 \pi r - 2000 cm^{3} \cdot r^{-2} \\
A''(r) &=& 4 \pi + 4000 cm^{3} \cdot r^{-3} \\
\intertext{Erste Ableitung mit Null gleichsetzen}
0 &=& 4 \pi r - 2000 cm^{3} \cdot r^{-2} \\
2000 cm^{3} \cdot r^{-2} &=& 4 \pi r \\
2000 cm^{3} &=& 4 \pi r^{3} \\
\frac{2000 cm^{3}}{4 \pi} &=& r^{3} \\
\frac{500 cm^{3}}{\pi} &=& r^{3} \\
\intertext{Dritte Wurzel ziehen}
\sqrt[3]{\frac{500 cm^{3}}{\pi}} &=& r \\
\intertext{Einsetzen in zweite Ableitung}
A''\left(\sqrt[3]{\frac{500 cm^{3}}{\pi}} \right) &=& 4 \pi + 4000 cm^{3} \cdot \left(\sqrt[3]{\frac{500 cm^{3}}{\pi}}\right)^{-3} \\
\intertext{Umformen}
&=& 4 \pi + 4000 cm^{3} \cdot \frac{1}{\left(\sqrt[3]{\frac{500 cm^{3}}{\pi}} \right)^{3}} \\
\intertext{Auflösen der Wurzel}
&=& 4 \pi + 4000 cm^{3} \cdot \frac{1}{\frac{500 cm^{3}}{\pi}} \\
\intertext{Auflösen Doppelbruch}
&=& 4 \pi + 4000 cm^{3} \cdot \frac{\pi}{500 cm^{3}} \\
\intertext{Kürzen}
&=& 4 \pi + 8 \cdot \pi \\
&=& 12 \pi \Rightarrow \text{$>0$, daher Minimum} \\
\intertext{Einsetzen des Radius in Formel für $h$}
h &=& \frac{1000 cm^{3}}{\pi \cdot \left(\sqrt[3]{\frac{500 cm^{3}}{\pi}} \right)^{2}} \\
\intertext{Umformen}
&=& \frac{1000 cm^{3}}{\pi \cdot \left(\sqrt[3]{500 cm^{3}\pi^{-1}} \right)^{2}} \\
\intertext{Umformen}
&=& \frac{1000 cm^{3}}{\pi \cdot \left(\left(500 cm^{3}\pi^{-1}\right)^{\frac{1}{3}} \right)^{2}} \\
\intertext{innere Klammer auflösen}
&=& \frac{1000 cm^{3}}{\pi \cdot \left((500 cm^{3})^{\frac{1}{3}} \cdot \pi^{-\frac{1}{3}} \right)^{2}} \\
\intertext{Klammer auflösen}
&=& \frac{1000 cm^{3}}{\pi \cdot (500 cm^{3})^{\frac{2}{3}} \cdot \pi^{-\frac{2}{3}}} \\
\intertext{Zusammenfassen und Bruch auseinanderziehen}
&=& \frac{1000 cm^{3}}{(500 cm^{3})^{\frac{2}{3}} } \cdot \frac{1}{\pi^{\frac{1}{3}}}
\end{alignat*}
\subsection{} %b
Die Oberfläche $A$ wird in diesem Fall ähnlich berechnet. Allerdings muss die Grundfläche nur einmal eingerechnet werden:\\
\begin{alignat*}{2}
A &=& \pi r^{2} + 2 \pi r h \\
\intertext{Die Berechnung von r ist analog. Lediglich der Faktor vor dem $\pi r^{2}$ ist anders. Daraus folgt für $A(r)$:}
A(r) &=& \pi r^{2} + 2000 cm^{3} \cdot r^{-1} \\
A'(r) &=& 2 \pi r - 2000 cm^{3} \cdot r^{-2} \\
\intertext{Aufgrund der Änderung ergibt sich nach umstellen nach r folgendes:}
\frac{2000 cm^{3}}{2 \pi} &=& r^{3} \\
\intertext{Durch Kürzen ergibt sich:}
\frac{1000 cm^{3}}{\pi} &=& r^{3} \\
\intertext{Ziehen der dritten Wurzel:}
\sqrt[3]{\frac{1000 cm^{3}}{\pi}} &=& r \\
\intertext{Beim Einsetzen in die zweite Ableitung verändert sich nicht viel:}
A''\left(\sqrt[3]{\frac{1000 cm^{3}}{\pi}} \right) = 2 \pi + 4000 cm^{3} \cdot \left(\sqrt[3]{\frac{1000 cm^{3}}{\pi}} \right)^{-3} \\
&=& 2 \pi + 4000 cm^{3} \cdot \frac{1}{ \left(\sqrt[3]{\frac{1000 cm^{3}}{\pi}} \right)^{3}} \\
&=& 2 \pi + 4000 cm^{3} \cdot \frac{1}{ \frac{1000 cm^{3}}{\pi}} \\
&=& 2 \pi + 4000 cm^{3} \cdot \frac{\pi}{ 1000 cm^{3}} \\
&=& 2 \pi + 4 \pi \\
&=& 6 \pi \Rightarrow \text{$>0$, daher Minimum}\\
\intertext{Durch Einsetzen in Gleichung von $h$ ergibt sich nun:}
h &=& \frac{1000 cm^{3}}{\pi \cdot \left(\sqrt[3]{\frac{1000 cm^{3}}{\pi}} \right)^{2}} \\
&=& \frac{1000 cm^{3}}{\pi \cdot \left(\left(\frac{1000 cm^{3}}{\pi}\right)^{\frac{1}{3}} \right)^{2}} \\
&=& \frac{1000 cm^{3}}{\pi \cdot \left(\left(1000 cm^{3} \cdot \pi^{-1} \right)^{\frac{1}{3}} \right)^{2}} \\
&=& \frac{1000 cm^{3}}{\pi \cdot \left((1000 cm^{3})^{\frac{1}{3}} \cdot \pi^{-\frac{1}{3}}\right)^{2}} \\
&=& \frac{1000 cm^{3}}{\pi \cdot (1000 cm^{3})^{\frac{2}{3}} \cdot \pi^{-\frac{2}{3}}} \\
&=& \frac{1000 cm^{3}}{(1000 cm^{3})^{\frac{2}{3}} \cdot \pi^{\frac{1}{3}}} \\
&=& \frac{1000 cm^{3}}{(1000 cm^{3})^{\frac{2}{3}}} \cdot \frac{1}{\pi^{\frac{1}{3}}} \\
\intertext{Kürzen}
&=& (1000 cm^{3})^{-\frac{1}{3}} \cdot \frac{1}{\pi^{\frac{1}{3}}}
\end{alignat*}
\end{document}

View File

@ -0,0 +1,392 @@
\documentclass[10pt,a4paper,oneside,ngerman,numbers=noenddot]{scrartcl}
\usepackage[T1]{fontenc}
\usepackage[utf8]{inputenc}
\usepackage[ngerman]{babel}
\usepackage{amsmath}
\usepackage{amsfonts}
\usepackage{amssymb}
\usepackage{paralist}
\usepackage{gauss}
\usepackage{pgfplots}
\usepackage[locale=DE,exponent-product=\cdot,detect-all]{siunitx}
\usepackage{tikz}
\usetikzlibrary{matrix,fadings,calc,positioning,decorations.pathreplacing,arrows,decorations.markings}
\usepackage{polynom}
\polyset{style=C, div=:,vars=x}
\pagenumbering{arabic}
\def\thesection{\arabic{section})}
\def\thesubsection{\alph{subsection})}
\def\thesubsubsection{(\roman{subsubsection})}
\makeatletter
\renewcommand*\env@matrix[1][*\c@MaxMatrixCols c]{%
\hskip -\arraycolsep
\let\@ifnextchar\new@ifnextchar
\array{#1}}
\makeatother
\begin{document}
\author{Jim Martens (6420323)}
\title{Hausaufgaben zum 30. Mai}
\maketitle
\section{} %1
\subsection{} %a
Berechnen von $\lim\limits_{n \rightarrow \infty} O_{n}$. Es gilt:\\
\begin{alignat*}{2}
O_{n} &=&& \frac{b-a}{n} \sum\limits_{i=1}^{n} f(x_{i}) = \frac{3}{n} \sum\limits_{i=1}^{n} f\left( \frac{3i}{n}\right) = \frac{3}{n} \sum\limits_{i=1}^{n} \left( \frac{3i}{n}\right)^{3} = \frac{3}{n^{4}} \sum\limits_{i=1}^{n} 27 \cdot i^{3}\\
&=&& \frac{81}{n^{4}} \sum\limits_{i=1}^{n} i^{3} = \frac{81}{n^{4}} \cdot \frac{n^{2}(n+1)^{2}}{4} = \frac{81 \cdot n^{2}(n+1)^{2}}{4 \cdot n^{4}} = \frac{81 \cdot (n+1)^{2}}{4 \cdot n^{2}} \\
&=&& \frac{81 \cdot (n^{2} + 2n + 1)}{4 \cdot n^{2}} = \frac{81n^{2} + 162n + 81}{4n^{2}}
\end{alignat*}\\
Also gilt $\lim\limits_{n \rightarrow \infty} O_{n} = \lim\limits_{n \rightarrow \infty} \left(\frac{81n^{2} + 162n + 81}{4n^{2}} \right) = \frac{81}{4}$. Der gesuchte Flächeninhalt hat den Wert $\frac{81}{4}$.
\subsection{} %b
\begin{alignat*}{2}
\int\limits_{a}^{b} f(x) \,dx = \int\limits_{0}^{3} x^{3} \,dx &=& \left[\frac{1}{4}x^{4}\right]_{0}^{3} \\
&=& \frac{1}{4} \cdot 3^{4} - \frac{1}{4} \cdot 0^{4} = \frac{1}{4} \cdot 81 - 0 = \frac{81}{4}
\end{alignat*}
\section{} %2
\subsubsection{} %i
\begin{alignat*}{2}
\int\limits_{1}^{3} (x^{2} -x-6) \,dx &=& \left[\frac{1}{3}x^{3} - \frac{1}{2}x^{2} - 6x \right]_{1}^{3} \\
&=& \left(\frac{1}{3} \cdot 3^{3} - \frac{1}{2} \cdot 3^{2} - 6 \cdot 3\right) - \left(\frac{1}{3} \cdot 1^{3} - \frac{1}{2} \cdot 1^{2} - 6 \cdot 1\right) \\
&=& \left(9 - \frac{9}{2} - 18\right) - \left(\frac{1}{3} - \frac{1}{2} - 6\right) = \left(\frac{18 - 9 - 36}{2}\right) - \left(\frac{2 - 3 - 36}{6}\right) \\
&=& \left(\frac{-27}{2}\right) - \left(\frac{-37}{6}\right) = \frac{-27}{2} + \frac{37}{6} = \frac{-81 + 37}{6} \\
&=& -\frac{44}{6} = -\frac{22}{3}
\end{alignat*}
\begin{tikzpicture}[>=stealth]
\begin{axis}[
ymin=-7,ymax=1,
x=1em,
y=1em,
axis x line=middle,
axis y line=middle,
axis line style=->,
xlabel={$x$},
ylabel={$y$},
xmin=0,xmax=4
]
\addplot[no marks, black, -] expression[domain=1:3,samples=100]{x*x -x -6} node[pos=0.65,anchor=north]{};
\node at (axis cs: 2.75,-3) {f};
\node at (axis cs: 1.75,-2.5) {A};
\draw[>=stealth] (axis cs:1,0) -- (axis cs:1,-6) node [pos=0.65,anchor=north]{};
\end{axis}
\end{tikzpicture}
\subsubsection{} %ii
\begin{alignat*}{2}
\int\limits_{1}^{3} x^{\frac{1}{3}} \,dx &=& \left[\frac{3}{4}x^{\frac{4}{3}} \right]_{1}^{3} \\
&=& \left(\frac{3}{4} \cdot 3^{\frac{4}{3}}\right) - \left(\frac{3}{4} \cdot 1^{\frac{4}{3}}\right) = 3^{\frac{4}{3}} \cdot \frac{3}{4} - \frac{3}{4} \\
&=& \left(3^{\frac{4}{3}} - 1\right) \cdot \frac{3}{4}
\end{alignat*}
\begin{tikzpicture}[>=stealth]
\begin{axis}[
ymin=0,ymax=3,
x=1cm,
y=1cm,
axis x line=middle,
axis y line=middle,
axis line style=->,
xlabel={$x$},
ylabel={$y$},
xmin=0,xmax=4
]
\addplot[no marks, black, -] expression[domain=1:3,samples=100]{x^(1/3)} node[pos=0.65,anchor=north]{};
\node at (axis cs: 2,1.5) {f};
\node at (axis cs: 2,0.5) {A};
\draw[>=stealth] (axis cs:1,0) -- (axis cs:1,1) node [pos=0.65,anchor=north]{};
\draw[>=stealth] (axis cs:3,0) -- (axis cs:3,1.44224957) node [pos=0.65,anchor=north]{};
\end{axis}
\end{tikzpicture}
\subsubsection{} %iii
\begin{alignat*}{2}
\int\limits_{1}^{3} \frac{1}{1+x^{2}} \,dx &=& \left[\arctan x \right]_{1}^{3} \\
&=& \arctan 3 - \arctan 1
\end{alignat*}
\begin{tikzpicture}[>=stealth]
\begin{axis}[
ymin=0,ymax=2,
x=1cm,
y=1cm,
axis x line=middle,
axis y line=middle,
axis line style=->,
xlabel={$x$},
ylabel={$y$},
xmin=0,xmax=4
]
\addplot[no marks, black, -] expression[domain=1:3,samples=100]{1/(1+x*x)} node[pos=0.65,anchor=north]{};
\node at (axis cs: 2,0.5) {f};
\node at (axis cs: 1.2,0.2) {A};
\draw[>=stealth] (axis cs:1,0) -- (axis cs:1,0.5) node [pos=0.65,anchor=north]{};
\draw[>=stealth] (axis cs:3,0) -- (axis cs:3,0.1) node [pos=0.65,anchor=north]{};
\end{axis}
\end{tikzpicture}
\subsubsection{} %iv
\begin{alignat*}{2}
\int\limits_{1}^{3} \ln x \,dx &=& \left[x \cdot \ln x - x \right]_{1}^{3} \\
&=& \left(3 \cdot \ln 3 - 3 \right) - \left(1 \cdot \ln 1 - 1 \right) = 3 \cdot \ln 3 - 3 + 1 \\
&=& 3 \cdot \ln 3 - 2
\end{alignat*}
\begin{tikzpicture}[>=stealth]
\begin{axis}[
ymin=0,ymax=2,
x=1cm,
y=1cm,
axis x line=middle,
axis y line=middle,
axis line style=->,
xlabel={$x$},
ylabel={$y$},
xmin=0,xmax=4
]
\addplot[no marks, black, -] expression[domain=1:3,samples=100]{ln x} node[pos=0.65,anchor=north]{};
\node at (axis cs: 2,1) {f};
\node at (axis cs: 2.2,0.4) {A};
%\draw[>=stealth] (axis cs:1,0) -- (axis cs:1,0.5) node [pos=0.65,anchor=north]{};
\draw[>=stealth] (axis cs:3,0) -- (axis cs:3,1.098612289) node [pos=0.65,anchor=north]{};
\end{axis}
\end{tikzpicture}
\subsubsection{} %v
\begin{alignat*}{2}
\int\limits_{1}^{3} e^{-x} \,dx &=& \left[-e^{-x} \right]_{1}^{3} \\
&=& \left(-e^{-3} \right) - \left( -e^{-1}\right) = -e^{-3} + e^{-1}
\end{alignat*}
\begin{tikzpicture}[>=stealth]
\begin{axis}[
ymin=0,ymax=2,
x=1cm,
y=1cm,
axis x line=middle,
axis y line=middle,
axis line style=->,
xlabel={$x$},
ylabel={$y$},
xmin=0,xmax=4
]
\addplot[no marks, black, -] expression[domain=1:3,samples=100]{e^(-x)} node[pos=0.65,anchor=north]{};
\node at (axis cs: 2,0.5) {f};
\node at (axis cs: 1.15,0.12) {A};
\draw[>=stealth] (axis cs:1,0) -- (axis cs:1,0.367879441) node [pos=0.65,anchor=north]{};
\draw[>=stealth] (axis cs:3,0) -- (axis cs:3,0.049787068) node [pos=0.65,anchor=north]{};
\end{axis}
\end{tikzpicture}
\section{} %3
\setcounter{subsubsection}{0}
\subsubsection{} %i
\begin{alignat*}{2}
\int (x^{4} + 2x^{3} -x +5) \,dx &=& \frac{1}{5}x^{5} + \frac{1}{4}x^{4} - \frac{1}{2}x^{2} + 5x
\end{alignat*}
\subsubsection{} %ii
\begin{alignat*}{2}
\int \frac{1}{\sqrt{x^{3}}} \,dx &=& \int \frac{1}{x^{\frac{3}{2}}} \,dx = \int x^{\frac{2}{3}} \,dx \\
&=& \frac{3}{5}x^{\frac{5}{3}}
\end{alignat*}
\subsubsection{} %iii
\begin{alignat*}{2}
\int x \cdot \sin(3x) \,dx &=& \frac{-\cos(3x)}{3} \cdot x + \frac{\sin(3x)}{9} \\
\intertext{Probe:}
\left(\frac{-\cos(3x)}{3} \cdot x + \frac{\sin(3x)}{9}\right)' &=& \left(\frac{-\cos(3x)}{3}\right)' \cdot x + \frac{-\cos(3x)}{3} \cdot 1 + \left(\frac{\sin(3x)}{9}\right)' \\
&=& \frac{\sin(3x) \cdot 3}{3} \cdot x + \frac{-\cos(3x)}{3} + \frac{\cos(3x) \cdot 3}{9} \\
&=& \sin(3x) \cdot x + \frac{-\cos(3x)}{3} + \frac{\cos(3x)}{3} \\
&=& \sin(3x) \cdot x
\end{alignat*}
\subsubsection{} %iv
\begin{alignat*}{2}
\int x^{3} \cdot \ln x \,dx &=& \frac{1}{4}x^{4} \cdot \ln x - \int \frac{1}{4}x^{4} \cdot \frac{1}{x}\,dx \\
&=& \frac{1}{4}x^{4} \cdot \ln x - \int \frac{1}{4}x^{3}\,dx \\
&=& \frac{1}{4}x^{4} \cdot \ln x - \frac{1}{16}x^{4} \\
\intertext{Probe:}
\left(\frac{1}{4}x^{4} \cdot \ln x - \frac{1}{4}x^{4}\right)' &=& (\frac{1}{4}x^{4} \cdot \ln x)' - \left(\frac{1}{16}x^{4}\right)' \\
&=& x^{3} \cdot \ln x + \frac{1}{4}x^{4} \cdot \frac{1}{x} - \frac{1}{4}x^{3} \\
&=& x^{3} \cdot \ln x + \frac{1}{4}x^{3} - \frac{1}{4}x^{3}\\
&=& x^{3} \cdot \ln x
\end{alignat*}
\subsubsection{} %v
\begin{alignat*}{2}
\int x^{2}e^{x} \,dx &=& x^{2}e^{x} - \int 2x \cdot e^{x}\,dx \\
&=& x^{2}e^{x} - 2 \cdot \int x \cdot e^{x}\,dx \\
&=& x^{2}e^{x} - 2 \cdot \left(x \cdot e^{x} - \int 1 \cdot e^{x}\,dx \right) \\
&=& x^{2}e^{x} - 2 \cdot (x \cdot e^{x} - e^{x}) \\
&=& x^{2}e^{x} - 2x \cdot e^{x} +2e^{x} \\
\intertext{Probe:}
(x^{2}e^{x} - 2x \cdot e^{x} +2e^{x})' &=& (x^{2}e^{x})' - (2x \cdot e^{x})' + 2e^{x} \\
&=& (2x \cdot e^{x} + x^{2}e^{x}) - (2e^{x} + 2x \cdot e^{x}) + 2e^{x} \\
&=& 2x \cdot e^{x} + x^{2}e^{x} - 2e^{x} - 2x \cdot e^{x} + 2e^{x} \\
&=& x^{2}e^{x}
\end{alignat*}
\section{} %4
\setcounter{subsubsection}{0}
\subsubsection{} %i
\begin{alignat*}{2}
t &=& \sqrt{2x+5} \\
\frac{dt}{dx} &=& \frac{2}{2 \cdot \sqrt{2x+5}} \\
\sqrt{2x+5}\,dt &=& dx \\
dx &=& t\,dt\\
\int \cos(\sqrt{2x+5}) \,dx &=& \int \cos(t) \cdot t\,dt \\
&=& \int \cos(t) \cdot t\,dt \\
&=& \sin(t) \cdot t - \int \sin(t) \cdot 1 \,dt \\
&=& \sin(t) \cdot t + \cos(t) \\
\int \cos(\sqrt{2x+5}) \,dx &=& \sqrt{2x+5} \cdot \sin(\sqrt{2x+5}) + \cos(\sqrt{2x+5})
\end{alignat*}
\begin{alignat*}{3}
\intertext{Probe:}
\left(\sqrt{2x+5} \cdot \sin(\sqrt{2x+5}) + \cos(\sqrt{2x+5})\right)' &=& (\sqrt{2x+5} \cdot \sin(\sqrt{2x+5}))' + (\cos(\sqrt{2x+5}))' \\
\intertext{Ableiten}
&=\begin{split}\frac{2 \cdot \sin(\sqrt{2x+5})}{2 \cdot \sqrt{2x+5}}\\ + \sqrt{2x+5} \cdot \frac{\cos(\sqrt{2x+5}) \cdot 2}{2 \cdot \sqrt{2x+5}} \\ + (-\sin(\sqrt{2x+5}) \cdot (\sqrt{2x+5})')\end{split}\\
\intertext{Zusammenfassen und Ableiten}
&= \begin{split}\frac{\sin(\sqrt{2x+5})}{\sqrt{2x+5}} + \cos(\sqrt{2x+5})\\ - \frac{\sin(\sqrt{2x+5}) \cdot 2}{2 \cdot \sqrt{2x+5}}\end{split} \\
\intertext{Zusammenfassen}
&= \begin{split}\frac{\sin(\sqrt{2x+5})}{\sqrt{2x+5}} + \cos(\sqrt{2x+5})\\ - \frac{\sin(\sqrt{2x+5})}{\sqrt{2x+5}}\end{split} \\
\intertext{Zusammenfassen}
&=& \cos(\sqrt{2x+5})
\end{alignat*}
\subsubsection{} %ii
\begin{alignat*}{2}
t &=& \sqrt[3]{x} \\
\frac{dt}{dx} &=& \frac{1}{3 \cdot \left(\sqrt[3]{x}\right)^{2}} \\
dx &=& 3 \cdot \left(\sqrt[3]{x}\right)^{2}\,dt \\
dx &=& 3 \cdot t^{2}\,dt\\
\int \sin(\sqrt[3]{x})\,dx &=& \int \sin(t) \cdot 3t^{2}\,dt\\
&=& 3 \int \sin(t) \cdot t^{2}\,dt \\
&=& 3\left(-\cos(t) \cdot t^{2} - \int -\cos(t) \cdot 2t \right) \\
3\left(-\cos(t) \cdot t^{2} + 2\int \cos(t) \cdot t \right) \\
&=& 3\left(-\cos(t) \cdot t^{2} + 2\left( \sin(t) \cdot t - \int \sin(t) \cdot 1 \right)\right) \\
&=& 3\left(-\cos(t) \cdot t^{2} + 2\left( \sin(t) \cdot t + \cos(t) \right)\right) \\
&=& 3\left(-\cos(t) \cdot t^{2} + \sin(t) \cdot 2t + 2 \cdot \cos(t)\right) \\
&=& -3t^{2} \cdot \cos(t) + 6t \cdot \sin(t) + 6 \cdot \cos(t) \\
\int \sin(\sqrt[3]{x})\,dx &=& -3 \cdot \sqrt[3]{x}^{2} \cdot \cos(\sqrt[3]{x}) + 6 \cdot \sqrt[3]{x} \cdot \sin(\sqrt[3]{x}) + 6 \cdot \cos(\sqrt[3]{x}) \\
&=& -3 \cdot x^{\frac{2}{3}} \cdot \cos(x^{\frac{1}{3}}) + 6 \cdot x^{\frac{1}{3}} \cdot \sin(x^{\frac{1}{3}}) + 6 \cdot \cos(x^{\frac{1}{3}})
\end{alignat*}
\begin{alignat*}{2}
\intertext{Probe:}
&& \left(-3 \cdot x^{\frac{2}{3}} \cdot \cos(x^{\frac{1}{3}}) + 6 \cdot x^{\frac{1}{3}} \cdot \sin(x^{\frac{1}{3}}) + 6 \cdot \cos(x^{\frac{1}{3}})\right)' \\
&=& \left(-3 \cdot x^{\frac{2}{3}} \cdot \cos(x^{\frac{1}{3}})\right)' + 6\left(\left(x^{\frac{1}{3}} \cdot \sin(x^{\frac{1}{3}})\right)' + \left(\cos(x^{\frac{1}{3}})\right)'\right) \\
&=& -3\left(\frac{2}{3}x^{-\frac{1}{3}} \cdot \cos(x^{\frac{1}{3}}) + x^{\frac{2}{3}} \cdot (-\sin(x^{\frac{1}{3}})) \cdot \frac{1}{3}x^{-\frac{2}{3}}\right)+ 6\left(\left(x^{\frac{1}{3}} \cdot \sin(x^{\frac{1}{3}})\right)' + \left(\cos(x^{\frac{1}{3}})\right)'\right) \\
&=& -3\left(\frac{2}{3}x^{-\frac{1}{3}} \cdot \cos(x^{\frac{1}{3}}) - \frac{1}{3} \cdot \sin(x^{\frac{1}{3}})\right)+ 6\left(\left(x^{\frac{1}{3}} \cdot \sin(x^{\frac{1}{3}})\right)' + \left(\cos(x^{\frac{1}{3}})\right)'\right) \\
\intertext{Ableiten}
&= \begin{split}-3\left(\frac{2}{3}x^{-\frac{1}{3}} \cdot \cos(x^{\frac{1}{3}}) - \frac{1}{3} \cdot \sin(x^{\frac{1}{3}})\right)\\ + 6\left(\left(\frac{1}{3}x^{-\frac{2}{3}} \cdot \sin(x^{\frac{1}{3}}) + x^{\frac{1}{3}} \cdot \cos(x^{\frac{1}{3}}) \cdot \frac{1}{3}x^{-\frac{2}{3}} \right) + \left(\cos(x^{\frac{1}{3}})\right)'\right) \end{split} \\
\intertext{Zusammenfassen}
&= \begin{split}-3\left(\frac{2}{3}x^{-\frac{1}{3}} \cdot \cos(x^{\frac{1}{3}}) - \frac{1}{3} \cdot \sin(x^{\frac{1}{3}})\right)\\ + 6\left(\frac{1}{3}x^{-\frac{2}{3}} \cdot \sin(x^{\frac{1}{3}}) + \frac{1}{3}x^{-\frac{1}{3}} \cdot \cos(x^{\frac{1}{3}}) + \left(\cos(x^{\frac{1}{3}})\right)'\right) \end{split} \\
\intertext{Ableiten}
&= \begin{split}-3\left(\frac{2}{3}x^{-\frac{1}{3}} \cdot \cos(x^{\frac{1}{3}}) - \frac{1}{3} \cdot \sin(x^{\frac{1}{3}})\right)\\ + 6\left(\frac{1}{3}x^{-\frac{2}{3}} \cdot \sin(x^{\frac{1}{3}}) + \frac{1}{3}x^{-\frac{1}{3}} \cdot \cos(x^{\frac{1}{3}}) - \sin(x^{\frac{1}{3}}) \cdot \frac{1}{3}x^{-\frac{2}{3}} \right) \end{split} \\
\intertext{Zusammenfassen}
&= \begin{split}-3\left(\frac{2}{3}x^{-\frac{1}{3}} \cdot \cos(x^{\frac{1}{3}}) - \frac{1}{3} \cdot \sin(x^{\frac{1}{3}})\right)\\ + 6\left(\frac{1}{3}x^{-\frac{1}{3}} \cdot \cos(x^{\frac{1}{3}})\right) \end{split} \\
\intertext{Zusammenfassen}
&=& -2x^{-\frac{1}{3}} \cdot \cos(x^{\frac{1}{3}}) + \sin(x^{\frac{1}{3}}) + 2x^{-\frac{1}{3}} \cdot \cos(x^{\frac{1}{3}}) \\
\intertext{Zusammenfassen}
&=& \sin(x^{\frac{1}{3}})
\end{alignat*}
\subsubsection{} %iii
\begin{alignat*}{2}
t &=& \sqrt{\frac{2}{7}x+3} \\
\frac{dt}{dx} &=& \frac{2}{14 \cdot \sqrt{\frac{2}{7}x+3}} \\
\frac{dt}{dx} &=& \frac{1}{7 \cdot \sqrt{\frac{2}{7}x+3}} \\
dx &=& 7 \cdot \sqrt{\frac{2}{7}x+3}\, dt \\
&=& 7t\\
\int e^{\sqrt{\frac{2}{7}x+3}}\,dx &=& \int e^{t} \cdot 7t\,dt \\
&=& e^{t} \cdot 7t - \int e^{t} \cdot 7\,dt \\
&=& e^{t} \cdot 7t - 7\int e^{t}\,dt \\
&=& e^{t} \cdot 7t - 7 \cdot e^{t} \\
&=& e^{t}(7t - 7) \\
&=& e^{\sqrt{\frac{2}{7}x+3}}\left(7 \cdot \sqrt{\frac{2}{7}x+3} - 7\right)
\end{alignat*}
\begin{alignat*}{2}
\intertext{Probe:}
&& \left(e^{\sqrt{\frac{2}{7}x+3}}\left(7 \cdot \sqrt{\frac{2}{7}x+3} - 7\right) \right)' \\
&=& \left(e^{\sqrt{\frac{2}{7}x+3}}\right)' \cdot \left(7 \cdot \sqrt{\frac{2}{7}x+3} - 7\right) + \left(e^{\sqrt{\frac{2}{7}x+3}}\right) \cdot \left(7 \cdot \sqrt{\frac{2}{7}x+3} - 7\right)' \\
&=& e^{\sqrt{\frac{2}{7}x+3}} \cdot \frac{2}{14 \cdot \sqrt{\frac{2}{7}x+3}} \cdot \left(7 \cdot \sqrt{\frac{2}{7}x+3} - 7\right) + \left(e^{\sqrt{\frac{2}{7}x+3}}\right) \cdot \left(7 \cdot \left(\frac{2}{7}x+3\right)^{\frac{1}{2}} - 7\right)' \\
&=& e^{\sqrt{\frac{2}{7}x+3}} \cdot \frac{1}{7 \cdot \sqrt{\frac{2}{7}x+3}} \cdot \left(7 \cdot \sqrt{\frac{2}{7}x+3} - 7\right) + \left(e^{\sqrt{\frac{2}{7}x+3}}\right) \cdot \left(7 \cdot \left(\frac{2}{7}x+3\right)^{\frac{1}{2}}\right)'\\
&=& e^{\sqrt{\frac{2}{7}x+3}} \cdot \frac{1}{7 \cdot \sqrt{\frac{2}{7}x+3}} \cdot \left(7 \cdot \sqrt{\frac{2}{7}x+3} - 7\right) + \left(e^{\sqrt{\frac{2}{7}x+3}}\right) \cdot \left(\frac{7}{2} \cdot \left(\frac{2}{7}x+3\right)^{-\frac{1}{2}} \cdot \frac{2}{7}\right) \\
\intertext{Zusammenfassen}
&=& e^{\sqrt{\frac{2}{7}x+3}} \cdot \left(1 - \frac{1}{\sqrt{\frac{2}{7}x+3}}\right) + \left(e^{\sqrt{\frac{2}{7}x+3}}\right) \cdot \left(\frac{2}{7}x+3\right)^{-\frac{1}{2}} \\
\intertext{Ausklammern}
&=& e^{\sqrt{\frac{2}{7}x+3}} \left(\left(1 - \frac{1}{\sqrt{\frac{2}{7}x+3}}\right) + \frac{1}{\sqrt{\frac{2}{7}x+3}} \right) \\
\intertext{Klammern auflösen}
&=& e^{\sqrt{\frac{2}{7}x+3}} \left(1 - \frac{1}{\sqrt{\frac{2}{7}x+3}} + \frac{1}{\sqrt{\frac{2}{7}x+3}}\right) \\
\intertext{Zusammenfassen}
&=& e^{\sqrt{\frac{2}{7}x+3}}
\end{alignat*}
\subsubsection{} %iv
\begin{alignat*}{2}
t &=& \ln(2x+1) \\
e^{t} &=& 2x+1 \\
x &=& \frac{e^{t}-1}{2} \\
\frac{dx}{dt} &=& \frac{1}{2}\left(e^{t}\right) \\
dx &=& \frac{1}{2} \cdot e^{t}\,dt \\
\int \left(\ln (2x+1)\right)^{2}\,dx &=& \int t^{2} \cdot \frac{1}{2} \cdot e^{t}\,dt \\
&=& \frac{1}{2} \int t^{2} \cdot e^{t} \\
&=& \frac{1}{2}\left(t^{2} \cdot e^{t} - \int 2t \cdot e^{t} \right) \\
&=& \frac{1}{2}\left(t^{2} \cdot e^{t} - 2\int t \cdot e^{t} \right) \\
&=& \frac{1}{2}\left(t^{2} \cdot e^{t} - 2\left(t \cdot e^{t} - \int 1 \cdot e^{t} \right)\right) \\
&=& \frac{1}{2}\left(t^{2} \cdot e^{t} - 2t \cdot e^{t} + 2 \cdot e^{t} \right) \\
&=& \frac{1}{2}t^{2} \cdot e^{t} - t \cdot e^{t} + e^{t} \\
&=& \frac{1}{2} \left(\ln(2x+1)\right)^{2} \cdot e^{\ln(2x+1)} - \ln(2x+1) \cdot e^{\ln(2x+1)} + e^{\ln(2x+1)} \\
&=& \left(\frac{1}{2} \left(\ln(2x+1)\right)^{2} - \ln(2x+1) + 1\right)e^{\ln(2x+1)} \\
&=& \left(\frac{1}{2} \left(\ln(2x+1)\right)^{2} - \ln(2x+1) + 1\right) \cdot (2x+1)
\end{alignat*}
\begin{alignat*}{2}
\intertext{Probe:}
&& \left(\left(\frac{1}{2} \left(\ln(2x+1)\right)^{2} - \ln(2x+1) + 1\right)\cdot (2x+1)\right)' \\
&=& \left(\frac{1}{2} \left(\ln(2x+1)\right)^{2} - \ln(2x+1) + 1\right)' \cdot (2x+1) + \left(\frac{1}{2}\left(\ln(2x+1)\right)^{2} - \ln(2x+1) + 1\right) \cdot (2x+1)' \\
&=& \left(\frac{2 \cdot \ln(2x+1)}{2x+1} - \frac{2}{2x+1}\right) \cdot (2x+1) + \left(\frac{1}{2}\left(\ln(2x+1)\right)^{2} - \ln(2x+1) + 1\right) \cdot 2 \\
&=& \left(\frac{2 \cdot \ln(2x+1) - 2}{2x+1}\right) \cdot (2x+1) + \left(\frac{1}{2}\left(\ln(2x+1)\right)^{2} - \ln(2x+1) + 1\right) \cdot 2 \\
\intertext{Zusammenfassen}
&=& \frac{2 \cdot \ln(2x+1) \cdot (2x+1) - 2 \cdot (2x+1)}{2x+1} + \left(\frac{1}{2}\left(\ln(2x+1)\right)^{2} - \ln(2x+1) + 1\right) \cdot 2 \\
\intertext{Zusammenfassen}
&=& 2 \cdot \ln(2x+1) - 2 + \left(\frac{1}{2}\left(\ln(2x+1)\right)^{2} - \ln(2x+1) + 1\right) \cdot 2 \\
\intertext{Zusammenfassen}
&=& 2 \cdot \ln(2x+1) - 2 + \left(\ln(2x+1)\right)^{2} - 2 \cdot \ln(2x+1) + 2 \\
\intertext{Zusammenfassen}
&=& \left(\ln(2x+1)\right)^{2}
\end{alignat*}
\section{} %5
\setcounter{subsubsection}{0}
\subsubsection{} %i
\begin{alignat*}{2}
f(x) &=& x^{3} - 12x^{2} + 36x + 1\\
f'(x) &=& 3x^{2} -24x + 36 \\
f''(x) &=& 6x - 24 \\
f'''(x) &=& 6 \\
\intertext{Berechnung Nullstellen erste Ableitung}
f'(x) &=& 0 \\
0 &=& 3x^{2} - 24x + 36 \\
0 &=& x^{2} - 8x + 12 \\
\intertext{pq-Formel}
x_{1,2} &=& 4 \pm \sqrt{4^{2} -12} \\
&=& 4 \pm \sqrt{16 - 12} \\
&=& 4 \pm \sqrt{4} \\
x_{1} &=& 4 + 2 = 6 \\
x_{2} &=& 4-2 = 2 \\
\intertext{Einsetzen in zweite Ableitung}
f''(2) &=& 6 \cdot 2 - 24 \\
&=& -12 \Rightarrow \text{ Maximum} \\
f''(6) &=& 6 \cdot 6 - 24 \\
&=& 12 \Rightarrow \text{ Minimum} \\
\intertext{Einsetzen in Funktion}
f(2) &=& 2^{3} - 12 \cdot 2^{2} + 36 \cdot 2 + 1 \\
&=& 8 - 48 + 72 + 1 \\
&=& 33 \\
f(6) &=& 6^{3} - 12 \cdot 6^{2} + 36 \cdot 6 + 1 \\
&=& 216 - 12 \cdot 36 + 216 + 1 \\
&=& 216 - 432 + 216 + 1 \\
&=& 1 \\
\intertext{Überprüfen der Intervallgrenzen - nur $f(0)$ noch nötig, da $f(6)$ bereits berechnet}
f(0) &=& 0^{3} - 12 \cdot 0^{2} + 36 \cdot 0 + 1 \\
&=& 1
\end{alignat*}
Die Tageshöchsttemperatur ist demnach $33\,^{\circ} \mathrm{C}$.
\subsubsection{} %ii
Wie in i) berechnet, ist die Tagestiefsttemperatur $1\,^{\circ} \mathrm{C}$.
\subsubsection{} %iii
Der Mittelwert wird klassisch so berechnet: Addieren aller Einzelwerte und Teilen der Summe durch deren Anzahl. In diesem Fall entspricht der Flächeninhalt zwischen dem Integral und der x-Achse der Summe aller einzelnen Temperaturen. Nun muss das Ergebnis noch mal $\frac{1}{6}$ genommen werden.\\
\begin{alignat*}{2}
\frac{1}{6} \cdot \int\limits_{0}^{6} (x^{3} - 12x^{2} + 36x +1) \,dx &=& \frac{1}{6} \cdot \left[\frac{1}{4}x^{4} - 4x^{3} + 18x^{2} + x \right]_{0}^{6} \\
&=& \frac{1}{6} \cdot \left(\left(\frac{1}{4} \cdot 6^{4} - 4 \cdot 6^{3} + 18 \cdot 6^{2} + 6 \right) - \left(\frac{1}{4} \cdot 0^{4} - 4 \cdot 0^{3} + 18 \cdot 0^{2} + 0 \right)\right) \\
&=& \frac{1}{6} \cdot \left(\frac{1}{4} \cdot 1296 - 4 \cdot 216 + 18 \cdot 36 + 6 \right)\\
&=& \frac{1}{6} \cdot \left(324 - 864 + 648 + 6 \right) \\
&=& \frac{1}{6} \cdot 114 = 19
\end{alignat*}\\
Die Durchschnittstemperatur des Tages ist demnach $19\,^{\circ} \mathrm{C}$.
\end{document}

View File

@ -0,0 +1,525 @@
\documentclass[10pt,a4paper,oneside,ngerman,numbers=noenddot]{scrartcl}
\usepackage[T1]{fontenc}
\usepackage[utf8]{inputenc}
\usepackage[ngerman]{babel}
\usepackage{amsmath}
\usepackage{amsfonts}
\usepackage{amssymb}
\usepackage{paralist}
\usepackage{gauss}
\usepackage{pgfplots}
\usepackage[locale=DE,exponent-product=\cdot,detect-all]{siunitx}
\usepackage{tikz}
\usetikzlibrary{matrix,fadings,calc,positioning,decorations.pathreplacing,arrows,decorations.markings}
\usepackage{polynom}
\polyset{style=C, div=:,vars=x}
\pgfplotsset{compat=1.8}
\pagenumbering{arabic}
\def\thesection{\arabic{section})}
\def\thesubsection{\alph{subsection})}
\def\thesubsubsection{(\roman{subsubsection})}
\makeatletter
\renewcommand*\env@matrix[1][*\c@MaxMatrixCols c]{%
\hskip -\arraycolsep
\let\@ifnextchar\new@ifnextchar
\array{#1}}
\makeatother
\begin{document}
\author{Jim Martens (6420323)}
\title{Hausaufgaben zum 6. Juni}
\maketitle
\section{} %1
\subsubsection{} %i
\begin{alignat*}{2}
\int \frac{x + 1}{x^{2} - x +6}\,dx &=& \int \frac{x}{x^{2} - x + 6}\,dx + \int \frac{1}{x^{2} -x + 6}\,dx \\
&=& \frac{1}{2}\int \frac{2x - 1 + 1}{x^{2} - x + 6}\,dx + \int \frac{1}{x^{2} -x + 6}\,dx \\
&=& \frac{1}{2}\int \frac{2x - 1}{x^{2} - x + 6}\,dx + \frac{1}{2}\int \frac{1}{x^{2} - x + 6}\,dx + \int \frac{1}{x^{2} -x + 6}\,dx \\
&=& \frac{1}{2} \cdot \ln |x^{2} - x + 6| + \frac{1}{2}\int \frac{1}{x^{2} - x + 6}\,dx + \int \frac{1}{x^{2} -x + 6}\,dx
\end{alignat*}
\begin{alignat*}{2}
x^{2} - x + 6 &=& x^{2} - x + \frac{1}{4} + 6 - \frac{1}{4} \\
&=& \left( x - \frac{1}{2}\right)^{2} + 6 - \frac{1}{4}
\end{alignat*}
\begin{alignat*}{2}
\intertext{Setze $c = 6 - \frac{1}{4}$. Substitution: $t = x - \frac{1}{2}$, $\frac{dt}{dx} = 1$ und $dx = dt$.}
&& \frac{1}{2} \cdot \ln |x^{2} - x + 6| + \frac{1}{2}\int \frac{1}{x^{2} - x + 6}\,dx + \int \frac{1}{x^{2} -x + 6}\,dx\\
&=& \frac{1}{2} \cdot \ln |x^{2} - x + 6| + \frac{1}{2}\int \frac{1}{\left( x - \frac{1}{2}\right)^{2} + c}\,dx + \int \frac{1}{\left( x - \frac{1}{2}\right)^{2} + c}\,dx \\
&=& \frac{1}{2} \cdot \ln |x^{2} - x + 6| + \frac{1}{2}\int \frac{1}{t^{2} + c}\,dt + \int \frac{1}{t^{2} + c}\,dt
\end{alignat*}
\begin{alignat*}{2}
\intertext{Bestimmung Nullstellen des Nennerpolynoms}
0 &=& x^{2} - x + 6 \\
x_{1,2} &=& \frac{1}{2} \pm \sqrt{\left(\frac{1}{2}\right)^{2} - 6} \\
&=& \frac{1}{2} \pm \sqrt{- \frac{23}{4}}
\end{alignat*}
\begin{alignat*}{2}
\intertext{Eine negative Wurzel kann nicht gelöst werden. Demnach gibt es keine Nullstellen. Setzen von $r = \sqrt{c}$. Dann ist $c = r^{2}$.}
&& \frac{1}{2} \cdot \ln |x^{2} - x + 6| + \frac{1}{2}\int \frac{1}{x^{2} - x + 6}\,dx + \int \frac{1}{x^{2} -x + 6}\,dx\\
&=& \frac{1}{2} \cdot \ln |x^{2} - x + 6| + \frac{1}{2}\int \frac{1}{t^{2} + r^{2}}\,dt + \int \frac{1}{t^{2} + r^{2}}\,dt \\
&=& \frac{1}{2} \cdot \ln |x^{2} - x + 6| + \frac{1}{2} \cdot \frac{1}{r} \cdot \arctan \left(\frac{t}{r} \right) + \frac{1}{r} \cdot \arctan \left(\frac{t}{r} \right) \\
&=& \frac{1}{2} \cdot \ln |x^{2} - x + 6| + \frac{1}{2} \cdot \frac{1}{\sqrt{c}} \cdot \arctan \left(\frac{t}{\sqrt{c}} \right) + \frac{1}{\sqrt{c}} \cdot \arctan \left(\frac{t}{\sqrt{c}} \right) \\
\intertext{Resubsitution}
&& \frac{1}{2} \cdot \ln |x^{2} - x + 6| + \frac{1}{2}\int \frac{1}{x^{2} - x + 6}\,dx + \int \frac{1}{x^{2} -x + 6}\,dx \\
&=& \frac{1}{2} \cdot \ln |x^{2} - x + 6| + \frac{1}{2} \cdot \frac{1}{\sqrt{c}} \cdot \arctan \left(\frac{x - \frac{1}{2}}{\sqrt{c}} \right) + \frac{1}{\sqrt{c}} \cdot \arctan \left(\frac{x - \frac{1}{2}}{\sqrt{c}} \right)
\end{alignat*}
\begin{alignat*}{2}
\intertext{Probe:}
&& \left(\frac{1}{2} \cdot \ln |x^{2} - x + 6| + \frac{1}{2} \cdot \frac{1}{\sqrt{c}} \cdot \arctan \left(\frac{x - \frac{1}{2}}{\sqrt{c}} \right) + \frac{1}{\sqrt{c}} \cdot \arctan \left(\frac{x - \frac{1}{2}}{\sqrt{c}} \right)\right)' \\
&=& \left(\frac{1}{2} \cdot \ln |x^{2} - x + 6|\right)' + \left(\frac{1}{2} \cdot \frac{1}{\sqrt{c}} \cdot \arctan \left(\frac{x - \frac{1}{2}}{\sqrt{c}} \right)\right)' + \left(\frac{1}{\sqrt{c}} \cdot \arctan \left(\frac{x - \frac{1}{2}}{\sqrt{c}} \right)\right)' \\
&=& \frac{1}{2} \cdot \frac{2x - 1}{x^{2} - x + 6} + \frac{1}{2} \cdot \frac{1}{\sqrt{c}} \cdot \frac{1}{\left(\frac{x - \frac{1}{2}}{\sqrt{c}} \right)^{2} + 1} \cdot \left(\frac{x - \frac{1}{2}}{\sqrt{c}} \right)' + \frac{1}{\sqrt{c}} \cdot \frac{1}{\left(\frac{x - \frac{1}{2}}{\sqrt{c}} \right)^{2} + 1} \cdot \left(\frac{x - \frac{1}{2}}{\sqrt{c}} \right)' \\
&=& \frac{1}{2} \cdot \frac{2x - 1}{x^{2} - x + 6} + \frac{1}{2} \cdot \frac{1}{\sqrt{c}} \cdot \frac{1}{\sqrt{c}} \cdot \frac{1}{\frac{\left(x - \frac{1}{2}\right)^{2}}{c} + 1} \cdot \left(x - \frac{1}{2}\right)' + \frac{1}{\sqrt{c}} \cdot \frac{1}{\sqrt{c}} \cdot \frac{1}{\frac{\left(x - \frac{1}{2}\right)^{2}}{c} + 1} \cdot \left(x - \frac{1}{2}\right)' \\
&=& \frac{1}{2} \cdot \frac{2x - 1}{x^{2} - x + 6} + \frac{1}{2} \cdot \frac{1}{c} \cdot \frac{1}{\frac{\left(x - \frac{1}{2}\right)^{2}}{c} + 1} + \frac{1}{c} \cdot \frac{1}{\frac{\left(x - \frac{1}{2}\right)^{2}}{c} + 1} \\
&=& \frac{1}{2} \cdot \frac{2x - 1}{x^{2} - x + 6} + \frac{1}{2} \cdot \frac{1}{c\left(\frac{\left(x - \frac{1}{2}\right)^{2}}{c} + 1\right)} + \frac{1}{c\left(\frac{\left(x - \frac{1}{2}\right)^{2}}{c} + 1\right)} \\
&=& \frac{1}{2} \cdot \frac{2x - 1}{x^{2} - x + 6} + \frac{1}{2} \cdot \frac{1}{\left(x - \frac{1}{2}\right)^{2} + c} + \frac{1}{\left(x - \frac{1}{2}\right)^{2} + c} \\
&=& \frac{1}{2} \cdot \frac{2x - 1}{x^{2} - x + 6} + \frac{1}{2} \cdot \frac{1}{x^{2} - x + \frac{1}{4} + 6 - \frac{1}{4}} + \frac{1}{x^{2} - x + \frac{1}{4} + 6 - \frac{1}{4}} \\
&=& \frac{1}{2} \cdot \frac{2x - 1}{x^{2} - x + 6} + \frac{1}{2} \cdot \frac{1}{x^{2} - x + 6} + \frac{1}{x^{2} - x + 6} \\
&=& \frac{1}{2} \cdot \frac{2x - 1 + 1}{x^{2} - x + 6} + \frac{1}{x^{2} - x + 6} \\
&=& \frac{1}{2} \cdot \frac{2x}{x^{2} - x + 6} + \frac{1}{x^{2} - x + 6} \\
&=& \frac{x}{x^{2} - x + 6} + \frac{1}{x^{2} - x + 6} \\
&=& \frac{x + 1}{x^{2} - x + 6}
\end{alignat*}
\subsubsection{} %ii
\begin{alignat*}{2}
0 &=& x^{2} - 4x + 4 \\
x_{1,2} &=& 2 \pm \sqrt{2^{2} - 4} \\
&=& 2 \pm \sqrt{0} \\
x_{1} = x_{2} &=& 2 \\
x^{2} - 4x + 4 &=& (x-2)(x-2) = (x-2)^{2}
\end{alignat*}
\begin{alignat*}{2}
\frac{2x + 1}{x^{2} -4x +4} = \frac{2x + 1}{(x-2)^{2}} &=& \frac{A}{(x-2)^{2}} + \frac{B}{x-2} \\
2x + 1 &=& \frac{A(x-2)^{2}}{(x-2)^{2}} + \frac{B(x-2)^{2}}{x-2} \\
&=& A + B(x-2) \\
&=& A + Bx -2B \\
&=& Bx - 2B + A
\end{alignat*}
\begin{alignat*}{2}
\intertext{Vergleichen der Koeffizienten}
B &=& 2 \\
-2B + A &=& 1 \\
-2 \cdot 2 + A &=& 1 \\
-4 + A &=& 1 \\
A &=& 5
\end{alignat*}
\begin{alignat*}{2}
\intertext{Partialbruchzerlegung:}
\frac{2x+1}{x^{2}-4x+4} &=& \frac{5}{(x-2)^{2}} + \frac{2}{x-2}
\end{alignat*}
\begin{alignat*}{2}
\intertext{Integrieren}
&& \int \frac{2x+1}{x^{2}-4x+4}\,dx \\
&=& \int \left(\frac{5}{(x-2)^{2}} + \frac{2}{x-2}\right)\,dx \\
&=& \int \frac{5}{(x-2)^{2}}\,dx + \int \frac{2}{x-2}\,dx \\
&=& 5 \int (x-2)^{-2}\,dx + 2 \int \frac{1}{x-2}\,dx \\
&=& -5 (x-2)^{-1} + 2 \cdot \ln |x-2|
\end{alignat*}
\begin{alignat*}{2}
\intertext{Probe}
&& \left(-5 (x-2)^{-1} + 2 \cdot \ln |x-2|\right)' \\
&=& \left(-5 (x-2)^{-1}\right)' + \left(2 \cdot \ln |x-2|\right)' \\
&=& 5(x-2)^{-2} + \frac{2}{x-2} \\
&=& \frac{5}{(x-2)^{2}} + \frac{2}{x-2} \\
&=& \frac{5 + 2(x-2)}{(x-2)^{2}} \\
&=& \frac{5 + 2x -4)}{(x-2)^{2}} \\
&=& \frac{2x + 1}{x^{2} -4x +4}
\end{alignat*}
\subsubsection{} %iii
\begin{alignat*}{2}
\int \frac{4x + 1}{x^{2} + 4x +8}\,dx &=& \int \frac{4x}{x^{2} + 4x + 8}\,dx + \int \frac{1}{x^{2} + 4x + 8}\,dx \\
&=& 2\int \frac{2x +4 - 4}{x^{2} + 4x + 8}\,dx + \int \frac{1}{x^{2} + 4x + 8}\,dx \\
&=& 2\int \frac{2x + 4}{x^{2} + 4x + 8}\,dx - 8\int \frac{1}{x^{2} + 4x + 8}\,dx + \int \frac{1}{x^{2} + 4x + 8}\,dx \\
&=& 2 \cdot \ln |x^{2} + 4x + 8| - 8\int \frac{1}{x^{2} + 4x + 8}\,dx + \int \frac{1}{x^{2} + 4x + 8}\,dx
\end{alignat*}
\begin{alignat*}{2}
x^{2} + 4x + 8 &=& x^{2} + 4x + \frac{16}{4} + 8 - \frac{16}{4} \\
&=& (x +2)^{2} + 4
\end{alignat*}
\begin{alignat*}{2}
\intertext{Setze $c = 4$. Substitution: $t = x + 2$, $\frac{dt}{dx} = 1$ und $dx = dt$.}
&& 2 \cdot \ln |x^{2} + 4x + 8| - 8\int \frac{1}{x^{2} + 4x + 8}\,dx + \int \frac{1}{x^{2} + 4x + 8}\,dx\\
&=& 2 \cdot \ln |x^{2} + 4x + 8| -8\int \frac{1}{(x+2)^{2} + c}\,dx + \int \frac{1}{(x+2)^{2} + c}\,dx \\
&=& 2 \cdot \ln |x^{2} + 4x + 8| -8\int \frac{1}{t^{2} + c}\,dt + \int \frac{1}{t^{2} + c}\,dt
\end{alignat*}
\begin{alignat*}{2}
\intertext{Bestimmung Nullstellen des Nennerpolynoms}
0 &=& x^{2} + 4x + 8 \\
x_{1,2} &=& -2 \pm \sqrt{(2)^{2} - 8} \\
&=& -2 \pm \sqrt{-4}
\end{alignat*}
\begin{alignat*}{2}
\intertext{Eine negative Wurzel kann nicht gelöst werden. Demnach gibt es keine Nullstellen. Setzen von $r = \sqrt{c}$. Dann ist $c = r^{2}$.}
&& 2 \cdot \ln |x^{2} + 4x + 8| - 8\int \frac{1}{x^{2} + 4x + 8}\,dx + \int \frac{1}{x^{2} + 4x + 8}\,dx\\
&=& 2 \cdot \ln |x^{2} + 4x + 8| -8\int \frac{1}{t^{2} + r^{2}}\,dt + \int \frac{1}{t^{2} + r^{2}}\,dt \\
&=& 2 \cdot \ln |x^{2} + 4x + 8| -8 \cdot \frac{1}{r} \cdot \arctan \left(\frac{t}{r} \right) + \frac{1}{r} \cdot \arctan \left(\frac{t}{r} \right) \\
&=& 2 \cdot \ln |x^{2} + 4x + 8| -8 \cdot \frac{1}{\sqrt{c}} \cdot \arctan \left(\frac{t}{\sqrt{c}} \right) + \frac{1}{\sqrt{c}} \cdot \arctan \left(\frac{t}{\sqrt{c}} \right) \\
\intertext{Resubsitution}
&& 2 \cdot \ln |x^{2} + 4x + 8| - 8\int \frac{1}{x^{2} + 4x + 8}\,dx + \int \frac{1}{x^{2} + 4x + 8}\,dx \\
&=& 2 \cdot \ln |x^{2} + 4x + 8| -8 \cdot \frac{1}{\sqrt{c}} \cdot \arctan \left(\frac{x+2}{\sqrt{c}} \right) + \frac{1}{\sqrt{c}} \cdot \arctan \left(\frac{x+2}{\sqrt{c}} \right)
\end{alignat*}
\begin{alignat*}{2}
\intertext{Probe:}
&& \left(2 \cdot \ln |x^{2} + 4x + 8| -8 \cdot \frac{1}{\sqrt{c}} \cdot \arctan \left(\frac{x+2}{\sqrt{c}} \right) + \frac{1}{\sqrt{c}} \cdot \arctan \left(\frac{x+2}{\sqrt{c}} \right)\right)' \\
&=& \left(2 \cdot \ln |x^{2} + 4x + 8|\right)' + \left(-6 \cdot \frac{1}{\sqrt{c}} \cdot \arctan \left(\frac{x+2}{\sqrt{c}} \right)\right)' + \left(\frac{1}{\sqrt{c}} \cdot \arctan \left(\frac{x+2}{\sqrt{c}} \right)\right)' \\
&=& 2 \cdot \frac{2x + 4}{x^{2} + 4x + 8} - 8 \cdot \frac{1}{\sqrt{c}} \cdot \frac{1}{\left(\frac{x + 2}{\sqrt{c}} \right)^{2} + 1} \cdot \left(\frac{x + 2}{\sqrt{c}} \right)' + \frac{1}{\sqrt{c}} \cdot \frac{1}{\left(\frac{x + 2}{\sqrt{c}} \right)^{2} + 1} \cdot \left(\frac{x + 2}{\sqrt{c}} \right)' \\
&=& 2 \cdot \frac{2x + 4}{x^{2} + 4x + 8} - 8 \cdot \frac{1}{\sqrt{c}} \cdot \frac{1}{\sqrt{c}} \cdot \frac{1}{\frac{(x + 2)^{2}}{c} + 1} \cdot (x + 2)' + \frac{1}{\sqrt{c}} \cdot \frac{1}{\sqrt{c}} \cdot \frac{1}{\frac{(x + 2)^{2}}{c} + 1} \cdot (x + 2)' \\
&=& 2 \cdot \frac{2x + 4}{x^{2} + 4x + 8} - 8 \cdot \frac{1}{c} \cdot \frac{1}{\frac{(x + 2)^{2}}{c} + 1} + \frac{1}{c} \cdot \frac{1}{\frac{(x + 2)^{2}}{c} + 1} \\
&=& 2 \cdot \frac{2x + 4}{x^{2} + 4x + 8} - 8 \cdot \frac{1}{c\left(\frac{(x + 2)^{2}}{c} + 1\right)} + \frac{1}{c\left(\frac{(x + 2)^{2}}{c} + 1\right)} \\
&=& 2 \cdot \frac{2x + 4}{x^{2} + 4x + 8} - 8 \cdot \frac{1}{(x + 2)^{2} + c} + \frac{1}{(x + 2)^{2} + c} \\
&=& 2 \cdot \frac{2x + 4}{x^{2} + 4x + 8} - 8 \cdot \frac{1}{x^{2} + 4x + 4 + 4} + \frac{1}{x^{2} + 4x + 4 + 4} \\
&=& 2 \cdot \frac{2x + 4}{x^{2} + 4x + 8} - 2\cdot \frac{4}{x^{2} + 4x + 8} + \frac{1}{x^{2} + 4x + 8} \\
&=& 2 \cdot \frac{2x + 4 - 4}{x^{2} + 4x + 8} + \frac{1}{x^{2} + 4x + 8} \\
&=& 2 \cdot \frac{2x}{x^{2} + 4x + 8} + \frac{1}{x^{2} + 4x + 8} \\
&=& \frac{4x + 1}{x^{2} + 4x + 8}
\end{alignat*}
\section{} %2
\subsection{} %a
\begin{alignat*}{2}
f(x) &=& e^{-x} \\
f'(x) &=& -e^{-x} \\
f''(x) &=& e^{-x} \\
f'''(x) &=& -e^{-x} \\
\intertext{Wendepunkte bestimmen}
f''(x) &=& 0 \\
e^{-x} &=& 0 \\
-x &=& \ln 0 \Rightarrow \text{$\ln 0$ ist nicht definiert, daher kann es keine Wendepunkte für $e^{-x}$ geben.}
\end{alignat*}
\begin{alignat*}{2}
g(x) &=& \frac{1}{1+x} \\
g'(x) &=& -(x+1)^{-2} \\
g''(x) &=& 2(x+1)^{-3} \\
g'''(x) &=& -6(x+1)^{-4} \\
\intertext{Wendepunkte bestimmen}
g''(x) &=& 0 \\
2(x+1)^{-3} &=& 0 \\
\frac{2}{(x+1)^{3}} &=& 0 \Rightarrow \text{Die Funktion wird niemals $0$. Daher kann es keine Wendepunkte geben.}
\end{alignat*}
\begin{alignat*}{2}
h(x) &=& \frac{1}{1+x^{2}} \\
h'(x) &=& -(1+x^{2})^{-2} \cdot 2x \\
h''(x) &=& 2(1+x^{2})^{-3} \cdot 2x \cdot 2x -(1+x^{2})^{-2} \cdot 2 \\
&=& 2(1+x^{2})^{-3} \cdot 4x^{2} - 2(1+x^{2})^{-2} \\
h'''(x) &=& -6(1+x^{2})^{-4} \cdot 2x \cdot 4x^{2} + 2(1+x^{2})^{-3} \cdot 8x + 4(1+x^{2})^{-3} \cdot 2x \\
&=& -6(1+x^{2})^{-4} \cdot 8x^{3} + 2(1+x^{2})^{-3} \cdot 8x + 4(1+x^{2})^{-3} \cdot 2x \\
\intertext{Wendepunkte bestimmen}
h''(x) &=& 0 \\
2(1+x^{2})^{-3} \cdot 4x^{2} - 2(1+x^{2})^{-2} &=& 0 \\
\intertext{Es ergibt sich das Ergebnis:}
x = \frac{1}{\sqrt{3}}
\end{alignat*}
\begin{alignat*}{2}
\intertext{Probe}
2(1+\left(\frac{1}{\sqrt{3}}\right)^{2})^{-3} \cdot 4\left(\frac{1}{\sqrt{3}}\right)^{2} - 2(1+\left(\frac{1}{\sqrt{3}}\right)^{2})^{-2} &=& 0 \\
2(1+\frac{1}{3})^{-3} \cdot 4 \cdot \frac{1}{3} - 2(1+\frac{1}{3})^{-2} &=& 0 \\
\frac{2 \cdot \frac{4}{3}}{\left(\frac{4}{3}\right)^{3}} - \frac{2}{\left(\frac{4}{3}\right)^{2}} &=& 0 \\
\frac{2 \cdot \frac{4}{3}}{\frac{64}{27}} - \frac{2}{\frac{16}{9}} &=& 0 \\
2 \cdot \frac{4}{3} \cdot \frac{27}{64} - 2 \cdot \frac{9}{16} &=& 0 \\
\frac{4}{3} \cdot \frac{54}{64} - \frac{18}{16} &=& 0 \\
\frac{4}{3} \cdot \frac{27}{32} - \frac{9}{8} &=& 0 \\
\intertext{27 mit 3 kürzen und 32 mit 4 kürzen}
1 \cdot \frac{9}{8} - \frac{9}{8} &=& 0 \\
\frac{9 - 9}{8} &=& 0 \\
0 &=& 0
\end{alignat*}
\begin{tikzpicture}[>=stealth]
\begin{axis}[
ymin=0,ymax=5,
x=1cm,
y=1cm,
axis x line=middle,
axis y line=middle,
axis line style=->,
xlabel={$x$},
ylabel={$y$},
xmin=0,xmax=5
]
\addplot[no marks, black, -] expression[domain=0:5,samples=100]{e^(-x)} node[pos=0.65,anchor=north]{};
\node at (axis cs: 1,0.8) {f};
\end{axis}
\end{tikzpicture}
\begin{tikzpicture}[>=stealth]
\begin{axis}[
ymin=0,ymax=5,
x=1cm,
y=1cm,
axis x line=middle,
axis y line=middle,
axis line style=->,
xlabel={$x$},
ylabel={$y$},
xmin=0,xmax=5
]
\addplot[no marks, black, -] expression[domain=0:5,samples=100]{1/(1+x)} node[pos=0.65,anchor=north]{};
\node at (axis cs: 1,0.8) {g};
\end{axis}
\end{tikzpicture}\\
\begin{tikzpicture}[>=stealth]
\begin{axis}[
ymin=0,ymax=5,
x=1cm,
y=1cm,
axis x line=middle,
axis y line=middle,
axis line style=->,
xlabel={$x$},
ylabel={$y$},
xmin=0,xmax=5
]
\addplot[no marks, black, -] expression[domain=0:5,samples=100]{1/(1+x^2)} node[pos=0.65,anchor=north]{};
\node at (axis cs: 2,0.8) {h};
\draw (axis cs:0.577350269,0.75) circle (2pt);
\end{axis}
\end{tikzpicture}
\subsection{} %b
\begin{alignat*}{2}
\intertext{Integrieren von f}
\int\limits_{0}^{b} e^{-x}\,dx &=& \left[ -e^{-x}\right]_{0}^{b} \\
&=& -e^{-b} + e^{0} \\
&=& -\frac{1}{e^{b}} + 1 \rightarrow 1 \text{ für } b \rightarrow \infty \\
&\Longrightarrow & \lim\limits_{b \rightarrow \infty} \int\limits_{0}^{b} e^{-x}\,dx = 1
\end{alignat*}
\begin{alignat*}{2}
\intertext{Integrieren von g}
\int\limits_{0}^{b} \frac{1}{1+x}\,dx &=& \left[\ln |1+x|\right]_{0}^{b} \\
&=& \ln |1 + b| - \ln |1 + 0| \\
&=& \ln |1 + b| \rightarrow \infty \text{ für } b \rightarrow \infty \\
&\Longrightarrow & \lim\limits_{b \rightarrow \infty} \int\limits_{0}^{b} \frac{1}{1+x}\,dx = \infty
\end{alignat*}
\begin{alignat*}{2}
\intertext{Integrieren von h}
\int\limits_{0}^{b} \frac{1}{1+x^{2}} \,dx &=& \left[ \arctan x \right]_{0}^{b} \\
&=& \arctan b - \arctan 0 \\
&=& \arctan b \rightarrow \frac{\pi}{2} \text{ für } b \rightarrow \infty \\
&\Longrightarrow & \lim\limits_{b \rightarrow \infty} \int\limits_{0}^{b} \frac{1}{1+x^{2}}\,dx = \frac{\pi}{2}
\end{alignat*}
\subsection{} %c
\begin{tikzpicture}[>=stealth]
\begin{axis}[
ymin=0,ymax=2,
x=2cm,
y=2cm,
axis x line=middle,
axis y line=middle,
axis line style=->,
xlabel={$x$},
ylabel={$y$},
xmin=-1,xmax=1
]
\addplot[no marks, black, -] expression[domain=-1:1,samples=100]{1/sqrt(1-x^2)} node[pos=0.65,anchor=north]{};
\node at (axis cs: 2,0.8) {f};
\end{axis}
\end{tikzpicture}
\begin{alignat*}{2}
\intertext{Integrieren von f}
\int\limits_{-1}^{1} \frac{1}{\sqrt{1-x^{2}}}\,dx &=& \left[\arcsin x\right]_{-1}^{1} \\
&=& \arcsin 1 - \arcsin (-1) \\
&=& \frac{\pi}{2} + \frac{\pi}{2}\\
&=& \pi
\end{alignat*}
\section{} %3
\begin{alignat*}{2}
\intertext{$n=4$}
\int\limits_{0}^{1} \sin x \,dx &\approx & \frac{1}{8}\left(\sin(0) + 2\sin\left(\frac{1}{4}\right) + 2\sin\left(\frac{1}{2}\right) + 2\sin\left(\frac{3}{4}\right) + \sin(1)\right) \\
&\approx & 0.4573009376
\end{alignat*}
\begin{alignat*}{2}
\intertext{$n=5$}
\int\limits_{0}^{1} \sin x \,dx &\approx & \frac{1}{10}\left(\sin(0) + 2\sin\left(\frac{1}{5}\right) +
2\sin\left(\frac{2}{5}\right) + 2\sin\left(\frac{3}{5}\right) + 2\sin\left(\frac{4}{5}\right) + \sin(1)\right) \\
&\approx & 0.4581643460
\end{alignat*}
\begin{alignat*}{2}
\intertext{$n=10$}
\int\limits_{0}^{1} \sin x \,dx &\approx \begin{split} \frac{1}{20}\left(\sin(0) + 2\sin\left(\frac{1}{10}\right) +
2\sin\left(\frac{1}{5}\right) + 2\sin\left(\frac{3}{10}\right) + 2\sin\left(\frac{2}{5}\right) +
2\sin\left(\frac{1}{2}\right)\right.\\ +
\left. 2\sin\left(\frac{3}{5}\right) + 2\sin\left(\frac{7}{10}\right) + 2\sin\left(\frac{4}{5}\right) +
2\sin\left(\frac{9}{10}\right) + \sin(1)\right)\end{split} \\
&\approx & 0.4593145489
\end{alignat*}
\section{} %4
\subsection{} %a
\begin{alignat*}{2}
f(1) &=& 10 \cdot e^{-\frac{2}{5}} \\
&\approx & 6.7032 \\
f(2) &=& 10 \cdot 2 \cdot e^{-frac{2}{5}\cdot 2} \\
&=& 20 \cdot e^{-\frac{4}{5}} \\
&\approx & 8.9866 \\
f(6) &=& 10 \cdot 6 \cdot e^{-\frac{2}{5}\cdot 6}\\
&=& 60 \cdot e^{-\frac{12}{5}} \\
&\approx & 5.4431 \\
f(12) &=& 10 \cdot 12 \cdot e^{-\frac{2}{5}\cdot 12} \\
&=& 120 \cdot e^{-\frac{24}{5}} \\
&\approx & 0.9876 \\
f(24) &=& 10 \cdot 24 \cdot e^{-\frac{2}{5}\cdot 24} \\
&=& 240 \cdot e^{-\frac{48}{5}} \\
&=& 0.0163
\end{alignat*}
\subsection{} %b
\begin{alignat*}{2}
f'(t) &=& 10 \cdot e^{-\frac{2}{5}t} - 10t \cdot e^{-\frac{2}{5}t} \cdot \frac{2}{5} \\
&=& 10 \cdot e^{-\frac{2}{5}t} - \frac{20}{5}t \cdot e^{-\frac{2}{5}t} \\
&=& 10 \cdot e^{-\frac{2}{5}t} - 4t \cdot e^{-\frac{2}{5}t} \\
&=& (10 - 4t) \cdot e^{-\frac{2}{5}t} \\
&=& \frac{10-4t}{e^{\frac{2}{5}t}} \\
f''(x) &=& \left((10 - 4t) \cdot e^{-\frac{2}{5}t}\right)' \\
&=& (10 - 4t)' \cdot e^{-\frac{2}{5}t} + (10 - 4t) \cdot \left(e^{-\frac{2}{5}t}\right)' \\
&=& -4 \cdot e^{-\frac{2}{5}t} + (10 - 4t) \cdot e^{-\frac{2}{5}t} \cdot \left(-\frac{2}{5}\right) \\
\intertext{Berechnung der Nullstelle(n) von $f'(x)$}
f'(x) &=& 0 \\
\intertext{$f'(x)$ mit Zähler ersetzen, da der über Nullstelle bestimmt}
10-4t &=& 0 \\
10 &=& 4t \\
\frac{5}{2} &=& t \\
\intertext{Einsetzen in $f(x)$}
f\left(\frac{5}{2}\right) &=& 10 \cdot \frac{5}{2} \cdot e^{-\frac{2}{5} \cdot \frac{5}{2}} \\
&=& 25 \cdot e^{-1} \\
&\approx & 9.1970
\end{alignat*}
\subsection{} %c
\begin{alignat*}{2}
\int 10t \cdot e^{-\frac{2}{5}t}\,dx &=& -\frac{5}{2}\cdot e^{-\frac{2}{5}t} \cdot 10t - \int -\frac{5}{2} \cdot e^{-\frac{2}{5}t} \cdot 10\,dx \\
&=& -\frac{5}{2}\cdot e^{-\frac{2}{5}t} \cdot 10t + 25 \int e^{-\frac{2}{5}t}\,dx \\
&=& -\frac{5}{2}\cdot e^{-\frac{2}{5}t} \cdot 10t - \frac{5}{2} \cdot e^{-\frac{2}{5}t} \cdot 25 \\
&=& -\frac{5}{2}\cdot e^{-\frac{2}{5}t} \cdot \left( 10t + 25\right) \\
\frac{1}{6} \int\limits_{0}^{6} 10t \cdot e^{-\frac{2}{5}t}\,dx &=& \frac{1}{6} \cdot \left[-\frac{5}{2}\cdot e^{-\frac{2}{5}t} \cdot (10t + 25)\right]_{0}^{6} \\
&=& \frac{1}{6}\left(-\frac{5}{2}\cdot e^{-\frac{2}{5} \cdot 6} \cdot ( 10 \cdot 6 + 25) - \left(-\frac{5}{2}\cdot e^{-\frac{2}{5} \cdot 0} \cdot ( 10 \cdot 0 + 25)\right)\right) \\
&=& \frac{1}{6}\left(-\frac{5}{2}\cdot e^{-\frac{12}{5}} \cdot 85 + \frac{5}{2}\cdot e^{0} \cdot 25\right) \\
&=& \frac{1}{6}\left(-\frac{425}{2}\cdot e^{-\frac{12}{5}} + \frac{125}{2}\right) \\
&\approx & 7.2037
\end{alignat*}
\subsection{} %d
\begin{alignat*}{2}
\intertext{Die Stammfunktion wurde bereits in c) berechnet. Daher setze ich direkt die Werte entsprechend ein.}
\frac{1}{6} \int\limits_{6}^{12} 10t \cdot e^{-\frac{2}{5}t}\,dx &=& \frac{1}{6} \cdot \left[-\frac{5}{2}\cdot e^{-\frac{2}{5}t} \cdot (10t + 25)\right]_{6}^{12} \\
&=& \frac{1}{6}\left(-\frac{5}{2}\cdot e^{-\frac{2}{5} \cdot 12} \cdot ( 10 \cdot 12 + 25) - \left(-\frac{5}{2}\cdot e^{-\frac{2}{5} \cdot 6} \cdot ( 10 \cdot 6 + 25)\right)\right) \\
&=& \frac{1}{6}\left(-\frac{5}{2}\cdot e^{-\frac{24}{5}} \cdot 145 + \frac{5}{2}\cdot e^{-\frac{12}{5}} \cdot 85\right) \\
&=& \frac{1}{6}\left(-\frac{725}{2}\cdot e^{-\frac{24}{5}} + \frac{425}{2}\cdot e^{-\frac{12}{5}}\right) \\
&\approx & 2.7157
\end{alignat*}
\subsection{} %e
\begin{alignat*}{2}
f'(t) &=& \frac{10-4t}{e^{\frac{2}{5}t}} \\
f''(t) &=& -4 \cdot e^{-\frac{2}{5}t} + (10 - 4t) \cdot e^{-\frac{2}{5}t} \cdot \left(-\frac{2}{5}\right) \\
&=& e^{-\frac{2}{5}t} \cdot \left(-4 + (10 - 4t) \cdot \left(-\frac{2}{5}\right) \right) \\
&=& e^{-\frac{2}{5}t} \cdot \left(-4 - 4 + \frac{8}{5}t \right) \\
&=& e^{-\frac{2}{5}t} \cdot \left(-8 + \frac{8}{5}t \right) \\
&=& \frac{-8 + \frac{8}{5}t}{ e^{-\frac{2}{5}t}} \\
f'''(t) &=& \left(e^{-\frac{2}{5}t} \cdot \left(-8 + \frac{8}{5}t \right)\right)' \\
&=& e^{-\frac{2}{5}t} \cdot \left(-\frac{2}{5} \right) \cdot \left(-8 + \frac{8}{5}t \right) + e^{-\frac{2}{5}t} \cdot \left(-8 + \frac{8}{5}t \right)' \\
&=& e^{-\frac{2}{5}t} \cdot \left(\frac{16}{5} - \frac{16}{25}t \right) + e^{-\frac{2}{5}t} \cdot \left(\frac{8}{5}\right) \\
&=& e^{-\frac{2}{5}t} \cdot \left(\frac{16}{5} - \frac{16}{25}t + \frac{8}{5}\right) \\
&=& e^{-\frac{2}{5}t} \cdot \left(\frac{24}{5} - \frac{16}{25}t\right) \\
\intertext{Nullstelle(n) von $f''(x)$ berechnen}
f''(x) &=& 0 \\
\intertext{$f''(x)$ mit dem Zähler ersetzen, da der für die Nullstelle zuständig ist}
-8 + \frac{8}{5}t &=& 0 \\
\frac{8}{5} &=& 8 \\
t &=& 5 \\
f'''(5) &=& e^{-\frac{2}{5} \cdot 5} \cdot \left(\frac{24}{5} - \frac{16}{25} \cdot 5\right) \\
&=& e^{-2} \cdot \left(\frac{24}{5} - \frac{16}{5}\right) \\
&=& e^{-2} \cdot \frac{8}{5} > 0 \Rightarrow \text{Minimum} \\
f(5) &=& 10 \cdot 5 \cdot e^{-\frac{2}{5} \cdot 5} \\
&=& 50 \cdot e^{-2} \\
&\approx & 6.7668
\end{alignat*}
\begin{tikzpicture}[>=stealth]
\begin{axis}[
ymin=0,ymax=10,
x=1em,
y=1em,
axis x line=middle,
axis y line=middle,
axis line style=->,
xlabel={$t$},
ylabel={$f(t)$},
xmin=0,xmax=24
]
\addplot[no marks, black, -] expression[domain=0:24,samples=100]{10*x*e^(-(2/5)*x)} node[pos=0.65,anchor=north]{};
\draw (axis cs:5,6.766764162) circle (2pt);
\end{axis}
\end{tikzpicture}
\section{} %5
\subsection{} %a
\begin{alignat*}{2}
h(x) &=& (x^{2}+1)^{\cos(x)} \\
&=& e^{\ln \left((x^{2}+1)^{\cos(x)}\right)}\\
&=& e^{\cos(x) \cdot \ln (x^{2}+1)} \\
h'(x) &=& e^{\cos(x) \cdot \ln (x^{2}+1)} \cdot \left(\cos(x) \cdot \ln (x^{2}+1)\right)' \\
&=& e^{\cos(x) \cdot \ln (x^{2}+1)} \cdot \left((\cos(x))' \cdot \ln (x^{2}+1) + \cos(x) \cdot (\ln (x^{2}+1))'\right) \\
&=& e^{\cos(x) \cdot \ln (x^{2}+1)} \cdot \left(-\sin(x) \cdot \ln (x^{2}+1) + \cos(x) \cdot \frac{(x^{2}+1)'}{x^{2}+1}\right) \\
&=& e^{\cos(x) \cdot \ln (x^{2}+1)} \cdot \left(-\sin(x) \cdot \ln (x^{2}+1) + \cos(x) \cdot \frac{2x}{x^{2}+1}\right) \\
&=& (x^{2}+1)^{\cos(x)} \cdot \left(-\sin(x) \cdot \ln (x^{2}+1) + \cos(x) \cdot \frac{2x}{x^{2}+1}\right)
\end{alignat*}
\subsection{} %b
\begin{alignat*}{2}
t &=& \sqrt{\frac{x}{4} + 3} \\
t^{2} &=& \frac{x}{4} + 3 \\
t^{2} - 3 &=& \frac{x}{4} \\
4t^{2} - 12 &=& x \\
8t &=& \frac{dx}{dt} \\
8t\,dt &=& dx
\end{alignat*}
\begin{alignat*}{2}
\int \sin \left(\sqrt{\frac{x}{4} + 3}\right) &\Rightarrow & \int \sin (t) \cdot 8t\,dt \\
\int \sin (t) \cdot 8t\,dt &=& -\cos(t) \cdot 8t - \int (-\cos (t) \cdot 8)\,dt \\
&=& -\cos(t) \cdot 8t + 8\int \cos(t)\,dt \\
&=& -\cos(t) \cdot 8t + 8 \cdot \sin(t) \\
\int \sin \left(\sqrt{\frac{x}{4} + 3}\right) &=& -\cos \left(\sqrt{\frac{x}{4} + 3} \right) \cdot 8 \cdot \left(\sqrt{\frac{x}{4} + 3} \right) + 8 \cdot \sin \left(\sqrt{\frac{x}{4} + 3}\right)
\end{alignat*}
\subsection{} %c
\begin{alignat*}{2}
g(x) = x^{2}-x-6 &=& 0 \\
x_{1,2} &=& \frac{1}{2} \pm \sqrt{\frac{1}{4} + 6 } \\
&=& \frac{1}{2} \pm \sqrt{\frac{25}{4}} \\
&=& \frac{1}{2} \pm \frac{5}{2} \\
x_{1} &=& \frac{6}{2} = 3 \\
x_{2} &=& -\frac{4}{2} = -2 \\
g(x) = x^{2}-x-6 &=& (x-3)(x+2)
\end{alignat*}
\begin{alignat*}{2}
\frac{3x+2}{x^{2}-x-6} = \frac{1}{(x-3)(x+2)} &=& \frac{A}{x-3} + \frac{B}{x+2} \\
&=& \frac{A(x+2) + B(x-3)}{(x-3)(x+2)} \\
&=& \frac{Ax + Bx + 2A - 3B}{(x-3)(x+2)} \\
&=& \frac{(A+B)x + 2A - 3B}{(x-3)(x+2)} \\
A + B &=& 3 \\
2A - 3B &=& 2 \\
A &=& 3 - B \\
2(3-B) - 3B &=& 2 \\
6 - 2B - 3B &=& 2 \\
-5B &=& -4 \\
5B &=& 4 \\
B &=& \frac{4}{5}\\
A &=& 3 - \frac{4}{5} \\
&=& \frac{11}{5} \\
\frac{3x+2}{x^{2}-x-6} &=& \frac{11}{5} \cdot \frac{1}{x-3} + \frac{4}{5} \cdot \frac{1}{x+2}
\end{alignat*}
\begin{alignat*}{2}
\int \frac{3x+2}{x^{2}-x-6}\,dx &=& \int \frac{11}{5} \cdot \frac{1}{x-3}\,dx + \int \frac{4}{5} \cdot \frac{1}{x+2}\,dx \\
&=& \frac{11}{5} \int \frac{1}{x-3}\,dx + \frac{4}{5} \int \frac{1}{x+2}\,dx \\
&=& \frac{11}{5} \cdot \ln |x-3| + \frac{4}{5} \cdot \ln |x+2|
\end{alignat*}
\subsection{} %d
\begin{alignat*}{2}
g(x) = x^{2} + 8x + 16 &=& (x+4)^{2} \\
\frac{x+1}{(x+4)^{2}} &=& \frac{A}{(x+4)^{2}} + \frac{B}{x+4} \\
&=& \frac{A + B(x+4)}{(x+4)^{2}} \\
&=& \frac{A + Bx + 4B}{(x+4)^{2}} \\
&=& \frac{Bx + A + 4B}{(x+4)^{2}} \\
\Rightarrow B &=&1 \\
A + 4B &=& 1 \\
A &=& -4B + 1 \\
A &=& -4 + 1 \\
A &=& -3 \\
\frac{x+1}{x^{2}+8x+16} &=& -\frac{3}{(x+4)^{2}} + \frac{1}{x+4}
\end{alignat*}
\begin{alignat*}{2}
\int \frac{x+1}{x^{2}+8x+16} \,dx &=& \int -\frac{3}{(x+4)^{2}}\,dx + \int \frac{1}{x+4}\,dx \\
&=& -3\int (x+4)^{-2}\,dx + \int \frac{1}{x+4}\,dx \\
&=& -3\left(-(x+4)^{-1}\right)+ \ln |x+4| \\
&=& 3(x+4)^{-1} + \ln |x+4|
\end{alignat*}
\end{document}

View File

@ -0,0 +1,297 @@
\documentclass[10pt,a4paper,oneside,ngerman,numbers=noenddot]{scrartcl}
\usepackage[T1]{fontenc}
\usepackage[utf8]{inputenc}
\usepackage[ngerman]{babel}
\usepackage{amsmath}
\usepackage{amsfonts}
\usepackage{amssymb}
\usepackage{paralist}
\usepackage{gauss}
\usepackage{pgfplots}
\usepackage[locale=DE,exponent-product=\cdot,detect-all]{siunitx}
\usepackage{tikz}
\usetikzlibrary{matrix,fadings,calc,positioning,decorations.pathreplacing,arrows,decorations.markings}
\usepackage{polynom}
\polyset{style=C, div=:,vars=x}
\pgfplotsset{compat=1.8}
\pagenumbering{arabic}
\def\thesection{\arabic{section})}
\def\thesubsection{\alph{subsection})}
\def\thesubsubsection{(\roman{subsubsection})}
\makeatletter
\renewcommand*\env@matrix[1][*\c@MaxMatrixCols c]{%
\hskip -\arraycolsep
\let\@ifnextchar\new@ifnextchar
\array{#1}}
\makeatother
\begin{document}
\author{Jim Martens (6420323)}
\title{Hausaufgaben zum 13. Juni}
\maketitle
\section{} %1
\subsection{} %a
\begin{alignat*}{2}
\intertext{Anwenden der Limes-Version des Wurzelkriteriums}
\lim\limits_{i \rightarrow \infty} \sqrt[i]{\left| \frac{i}{2^{i}}\right|} &=& \lim\limits_{i \rightarrow \infty} \frac{\sqrt[i]{\left| i \right|}}{\sqrt[i]{\left| 2^{i}\right|}} \\
&=& \lim\limits_{i \rightarrow \infty} \frac{\sqrt[i]{|i|}}{2} \\
\intertext{Anwenden, dass $\sqrt[i]{|i|} \rightarrow 1$ für $i \rightarrow \infty$ gilt}
&=& \frac{1}{2} < 1 \Rightarrow \text{ Konvergenz}
\end{alignat*}
\subsection{} %b
\begin{alignat*}{2}
\intertext{Anwenden der Limes-Version des Quotientenkriteriums}
\lim\limits_{i \rightarrow \infty} \left|\frac{\frac{(-1)^{i+1} \cdot (i+1)!}{(i+1)^{i+1}}}{\frac{(-1)^{i} \cdot i!}{i^{i}}} \right| &=& \lim\limits_{i \rightarrow \infty} \left| \frac{(-1)^{i+1} \cdot (i+1)! \cdot i^{i}}{(i+1)^{i+1} \cdot (-1)^{i} \cdot i!}\right| \\
&=& \lim\limits_{i \rightarrow \infty} \left| \frac{(-1)^{i+1} \cdot (i+1)! \cdot i^{i}}{(-1)^{i} \cdot i! \cdot (i+1)^{i+1}}\right| \\
\intertext{Kürzen}
&=& \lim\limits_{i \rightarrow \infty} \left| \frac{(-1) \cdot (i+1) \cdot i^{i}}{(i+1)^{i+1}}\right| \\
\intertext{Kürzen}
&=& \lim\limits_{i \rightarrow \infty} \left| \frac{(-1) \cdot i^{i}}{(i+1)^{i}}\right| \\
\intertext{Ausklammern von $i^{i}$ im Nenner, Nullfolgen werden durch $...$ symbolisiert}
&=& \lim\limits_{i \rightarrow \infty} \left| \frac{(-1) \cdot i^{i}}{(1 + ...) \cdot i^{i}}\right| \\
&=& \lim\limits_{i \rightarrow \infty} \left| \frac{-1}{(1 + ...)}\right| \\
&=& \left|\frac{-1}{1}\right| = 1 \Rightarrow \text{ Keine Aussage möglich}
\end{alignat*}
\subsection{} %c
\subsubsection{} %i
\begin{alignat*}{2}
\intertext{Es sei $x \in \mathbb{R}$ eine beliebig fest gewählte Zahl.}
\lim\limits_{i \rightarrow \infty} \left|\frac{(i+1)^{2}2^{i+1}x^{i+1}}{i^{2}2^{i}x^{i}} \right| &=& \lim\limits_{i \rightarrow \infty} \left|\frac{(i+1)^{2} \cdot 2x}{i^{2}} \right| \\
&=& \lim\limits_{i \rightarrow \infty} \left|\frac{(i+1)^{2}}{i^{2}} \cdot 2x \right| \\
&=& \lim\limits_{i \rightarrow \infty} \left(\left|\frac{(i+1)^{2}}{i^{2}}\right| \cdot |2x| \right) \\
\intertext{Anwenden, dass $x$ fest gewählt ist}
&=& 2|x| \cdot \lim\limits_{i \rightarrow \infty} \left|\frac{(i+1)^{2}}{i^{2}}\right| \\
&=& 2|x| \cdot \lim\limits_{i \rightarrow \infty} \left|\frac{i^{2} + 2i + 1}{i^{2}}\right| \\
\intertext{Ausklammern von $i^{2}$ im Zähler und Nenner, anschließend kürzen}
&=& 2|x| \cdot \lim\limits_{i \rightarrow \infty} \left|\frac{1 + \frac{2}{i} + \frac{1}{i^{2}}}{1}\right| \\
&=& 2|x| \cdot 1 = 2|x|
\end{alignat*}
\begin{alignat*}{2}
2|x| < 1 &\Longleftrightarrow & |x| < \frac{1}{2} \\
2|x| > 1 &\Longleftrightarrow & |x| > \frac{1}{2}
\end{alignat*}
Demzufolge liegt Konvergenz vor, falls $|x| < \frac{1}{2}$ gilt; Divergenz liegt vor, falls $|x| > \frac{1}{2}$ gilt. Also gilt $R = \frac{1}{2}$.
\subsubsection{} %ii
\begin{alignat*}{2}
\intertext{Es sei $x \in \mathbb{R}$ eine beliebig fest gewählte Zahl.}
\lim\limits_{i \rightarrow \infty} \sqrt[i]{\left| i^{2} \cdot 2^{i} \cdot x^{i} \right|} &=& \lim\limits_{i \rightarrow \infty} \left( \sqrt[i]{\left| i^{2} \right|} \cdot \sqrt[i]{\left| 2^{i} \right|} \cdot \sqrt[i]{\left| x^{i} \right|} \right) \\
&=& \lim\limits_{i \rightarrow \infty} \left( \sqrt[i]{\left| i^{2} \right|} \cdot |2| \cdot |x| \right) \\
\intertext{Anwenden, dass $x$ fest gewählt ist}
&=& 2 \cdot |x| \cdot \lim\limits_{i \rightarrow \infty} \sqrt[i]{\left| i^{2} \right|} \\
&=& 2 \cdot |x| \cdot \lim\limits_{i \rightarrow \infty} \sqrt[i]{|i|} \cdot \lim\limits_{i \rightarrow \infty} \sqrt[i]{|i|} \\
\intertext{Anwenden, dass $\sqrt[i]{|i|} \rightarrow 1$ für $i \rightarrow \infty$ gilt}
&=& 2 \cdot |x| \cdot 1 \cdot 1 = 2|x|
\end{alignat*}
\begin{alignat*}{2}
2|x| < 1 &\Longleftrightarrow & |x| < \frac{1}{2} \\
2|x| > 1 &\Longleftrightarrow & |x| > \frac{1}{2}
\end{alignat*}
Demzufolge liegt Konvergenz vor, falls $|x| < \frac{1}{2}$ gilt; Divergenz liegt vor, falls $|x| > \frac{1}{2}$ gilt. Also gilt $R = \frac{1}{2}$.
\section{} %2
\setcounter{subsubsection}{0}
\subsubsection{} %i
\begin{alignat*}{2}
\lim\limits_{i \rightarrow \infty} \left|\frac{\frac{-1}{2^{i+2}}}{\frac{-1}{2^{i+1}}} \right| &=& \lim\limits_{i \rightarrow \infty} \left|\frac{(-1) \cdot 2^{i+1}}{2^{i+2} \cdot (-1)} \right| \\
\intertext{Kürzen}
&=& \lim\limits_{i \rightarrow \infty} \left|\frac{1}{2} \right| \\
&=& \frac{1}{2} < 1 \Rightarrow \text{ Konvergenz}
\end{alignat*}
\begin{alignat*}{2}
\intertext{Berechnen des Grenzwertes}
\sum\limits_{i=1}^{\infty} \frac{-1}{2^{i+1}} &=& - \sum\limits_{i=1}^{\infty} \frac{1}{2^{i+1}} \\
&=& - \sum\limits_{i=1}^{\infty} \frac{1^{i+1}}{2^{i+1}} \\
&=& - \sum\limits_{i=1}^{\infty} \left(\frac{1}{2}\right)^{i+1} \\
&=& - \sum\limits_{i=2}^{\infty} \left(\frac{1}{2}\right)^{i} \\
&=& -\left( \sum\limits_{i=0}^{\infty} \left(\frac{1}{2}\right)^{i} - \left(\frac{1}{2}\right)^{0} - \left(\frac{1}{2}\right)^{1} \right) \\
&=& -\left( \frac{1}{1 - \frac{1}{2}} - 1 - \frac{1}{2} \right) \\
&=& -\left( 2 - 1 - \frac{1}{2} \right) \\
&=& -\left( \frac{1}{2} \right)
\end{alignat*}
\subsubsection{} %ii
\begin{alignat*}{2}
\sum\limits_{i=1}^{\infty} \frac{(-1)^{i} \cdot i}{2(i+1)} &=& \sum\limits_{i=0}^{\infty} \frac{(-1)^{i} \cdot i}{2(i+1)} - \frac{(-1)^{0} \cdot 0}{2(0+1)} \\
&=& \sum\limits_{i=0}^{\infty} \frac{(-1)^{i} \cdot i}{2(i+1)} - 0 \\
&=& \sum\limits_{i=0}^{\infty} (-1)^{i} \cdot \frac{i}{2(i+1)} \\
\intertext{Die Glieder der Reihe werden betragsmäßig immer größer und bilden daher keine Nullfolge. Deswegen divergiert die Reihe.}
a_{0} &=& 0 \\
a_{1} &=& \frac{1}{4} \\
a_{2} &=& \frac{1}{3} = \frac{8}{24} \\
a_{3} &=& \frac{3}{8} = \frac{9}{24} = \frac{15}{40}\\
a_{4} &=& \frac{2}{5} = \frac{16}{40}
\end{alignat*}
\subsubsection{} %iii
\begin{alignat*}{2}
\sum\limits_{i=1}^{\infty} \frac{1}{2(i+1)} &=& \frac{1}{2} \cdot \sum\limits_{i=2}^{\infty} \frac{1}{i} \\
&=& \frac{1}{2} \cdot \left( \sum\limits_{i=1}^{\infty} \frac{1}{i} - 1\right)
\intertext{Die Harmonische Reihe divergiert. Damit divergiert auch jede Reihe, die durch Hinzufügen, Weglassen oder Abändern endlich vieler Glieder entsteht.
Daher divergiert diese Reihe.}
\end{alignat*}
\subsubsection{} %iv
\begin{alignat*}{2}
\sum\limits_{i=1}^{\infty} \frac{(-1)^{i+1}}{2^{i}} &=& \sum\limits_{i=0}^{\infty} \frac{(-1)^{i+1}}{2^{i}} - \frac{(-1)^{0+1}}{2^{0}} \\
&=& \sum\limits_{i=0}^{\infty} \frac{(-1)^{i+1}}{2^{i}} - \frac{-1}{1} \\
&=& \sum\limits_{i=0}^{\infty} \frac{(-1)^{i+1}}{2^{i}} + 1
\end{alignat*}
\begin{alignat*}{2}
\lim\limits_{i \rightarrow \infty} \sqrt[i]{\left|\frac{(-1)^{i+1}}{2^{i}} \right|} &=& \lim\limits_{i \rightarrow \infty} \frac{\sqrt[i]{\left|(-1)^{i+1}\right|}}{\sqrt[i]{|2^{i}|}} \\
&=& \lim\limits_{i \rightarrow \infty} \frac{\sqrt[i]{|(-1)^{i}| \cdot |(-1)|}}{2} \\
&=& \lim\limits_{i \rightarrow \infty} \frac{\sqrt[i]{|(-1)^{i}|} \cdot \sqrt[i]{|-1|}}{2} \\
&=& \frac{1}{2} < 1 \Rightarrow \text{ Konvergenz}
\end{alignat*}
\begin{alignat*}{2}
\sum\limits_{i=0}^{\infty} \frac{(-1)^{i+1}}{2^{i}} + 1 &=& \sum\limits_{i=0}^{\infty} (-1) \cdot \frac{(-1)^{i}}{2^{i}} + 1 \\
&=& \sum\limits_{i=0}^{\infty} (-1) \cdot \left(-\frac{1}{2}\right)^{i} + 1 \\
&=& -\sum\limits_{i=0}^{\infty} \left(-\frac{1}{2}\right)^{i} + 1 \\
&=& -\left(\frac{1}{1 + \frac{1}{2}} \right) + 1 \\
&=& -\left(\frac{1}{\frac{3}{2}} \right) + 1 \\
&=& -\frac{2}{3} + 1 \\
&=& \frac{1}{3}
\end{alignat*}
\subsubsection{} %v
\begin{alignat*}{2}
\sum\limits_{i=0}^{\infty} \frac{(-1)^{i}}{2i+1} &=& \sum\limits_{i=0}^{\infty} (-1)^{i} \cdot \frac{1}{2i+1} \\
\intertext{Nach dem Leibnitz-Kriterium konvergiert diese Reihe, da die Reihenglieder $a_{i} = \frac{1}{2i+1}$ eine monotone Nullfolge bilden.}
s_{0} &=& 1 \\
s_{1} &=& 1 - \frac{1}{3} = \frac{2}{3}\\
s_{2} &=& 1 - \frac{1}{3} + \frac{1}{5} = \frac{13}{15} \approx 0.867 \\
s_{3} &=& \frac{13}{15} - \frac{1}{7} = \frac{91}{105} - \frac{15}{105} = \frac{76}{105} \approx 0.724
\intertext{Der Grenzwert könnte $\approx 0.7853$ sein.}
\end{alignat*}
\subsubsection{} %vi
\begin{alignat*}{2}
\sum\limits_{i=1}^{\infty} \frac{(-1)^{i}}{2i} &=& \sum\limits_{i=1}^{\infty} (-1)^{i} \cdot \frac{1}{2i} \\
\intertext{Nach dem Leibnitz-Kriterium ist diese Reihe konvergent.}
s_{0} &=& -\frac{1}{2} \\
s_{1} &=& -\frac{1}{2} + \frac{1}{4} = -\frac{1}{4} \\
s_{2} &=& -\frac{1}{4} - \frac{1}{6} = -\frac{10}{24} = -\frac{5}{12} \approx -0.4166667 \\
s_{3} &=& -\frac{5}{12} + \frac{1}{8} = -\frac{40}{96} + \frac{12}{96} = -\frac{28}{96} \approx -0.2916667
\intertext{Der Grenzwert könnte $\approx -0.3541667$ sein.}
\end{alignat*}
\section{} %3
\subsection{} %a
\begin{tikzpicture}[>=stealth]
\begin{axis}[
ymin=0,ymax=10,
x=1cm,
y=1cm,
axis x line=middle,
axis y line=middle,
axis line style=->,
xlabel={$x$},
ylabel={$f(x)$},
xmin=0,xmax=10
]
\addplot[no marks, black, -] expression[domain=0:10,samples=100]{1/x} node[pos=0.65,anchor=north]{};
\addplot[no marks, black, -] expression[domain=1:2,samples]{1} node[pos=0.65,anchor=north]{};
\draw[>=stealth] (axis cs:1,1) -- (axis cs:1,0) node [pos=0.65,anchor=north]{};
\draw[>=stealth] (axis cs:2,1) -- (axis cs:2,0) node [pos=0.65,anchor=north]{};
\addplot[no marks, black, -] expression[domain=2:3,samples]{1/2} node[pos=0.65,anchor=north]{};
\draw[>=stealth] (axis cs:3,0.5) -- (axis cs:3,0) node [pos=0.65,anchor=north]{};
\addplot[no marks, black, -] expression[domain=3:4,samples]{1/3} node[pos=0.65,anchor=north]{};
\draw[>=stealth] (axis cs:4,0.3333333333333) -- (axis cs:4,0) node [pos=0.65,anchor=north]{};
\addplot[no marks, black, -] expression[domain=4:5,samples]{1/4} node[pos=0.65,anchor=north]{};
\draw[>=stealth] (axis cs:5,0.25) -- (axis cs:5,0) node [pos=0.65,anchor=north]{};
\addplot[no marks, black, -] expression[domain=5:6,samples]{1/5} node[pos=0.65,anchor=north]{};
\draw[>=stealth] (axis cs:6,0.2) -- (axis cs:6,0) node [pos=0.65,anchor=north]{};
\addplot[no marks, black, -] expression[domain=6:7,samples]{1/6} node[pos=0.65,anchor=north]{};
\draw[>=stealth] (axis cs:7,0.1666666667) -- (axis cs:7,0) node [pos=0.65,anchor=north]{};
\addplot[no marks, black, -] expression[domain=7:8,samples]{1/7} node[pos=0.65,anchor=north]{};
\draw[>=stealth] (axis cs:8,0.142857142857) -- (axis cs:8,0) node [pos=0.65,anchor=north]{};
\addplot[no marks, black, -] expression[domain=8:9,samples]{1/8} node[pos=0.65,anchor=north]{};
\draw[>=stealth] (axis cs:9,0.125) -- (axis cs:9,0) node [pos=0.65,anchor=north]{};
\addplot[no marks, black, -] expression[domain=9:10,samples]{1/9} node[pos=0.65,anchor=north]{};
\draw[>=stealth] (axis cs:10,0.111111111) -- (axis cs:10,0) node [pos=0.65,anchor=north]{};
\node at (axis cs: 1.5,1.25) {1};
\node at (axis cs: 2.5,0.75) {$\frac{1}{2}$};
\node at (axis cs: 3.5,0.5833333) {$\frac{1}{3}$};
\node at (axis cs: 4.5,0.5) {$\frac{1}{4}$};
\node at (axis cs: 5.5,0.45) {$\frac{1}{5}$};
\node at (axis cs: 6.5,0.4166667) {$\frac{1}{6}$};
\node at (axis cs: 7.5,0.392857142857) {$\frac{1}{7}$};
\node at (axis cs: 8.5,0.375) {$\frac{1}{8}$};
\node at (axis cs: 9.5,0.36111111) {$\frac{1}{9}$};
\end{axis}
\end{tikzpicture}\\
Anhand der Skizze erkennt man:\\
\begin{alignat*}{3}
H_{n} &\geq & \int\limits_{1}^{n+1} \frac{1}{x}\,dx &=& [\ln x]_{1}^{n+1} \\
&\geq & &=& \ln (n+1) - \ln 1 \\
&\geq & &=& \ln (n+1) \\
\end{alignat*}
\subsection{} %b
Der Logarithmus $\ln$ ist divergent. Da die harmonische Reihe nach entsprechender Formel größer oder gleich dem natürlichen Logarithmus ist, wächst somit die harmonische Reihe ebenfalls über alle Schranken.
\section{} %4
\subsection{} %a
Aus Aufgabe 3 ist bekannt, dass Folgendes gilt:\\
\begin{alignat*}{2}
\ln(n+1) &\leq & H_{n} \; (n = 1,2,...)\\
\intertext{Da der natürliche Logarithmus monoton wachsend ist, gilt somit auch dies:}
\ln(n) &\leq & H_{n} \; (n = 1,2,...)
\end{alignat*}
\begin{tikzpicture}[>=stealth]
\begin{axis}[
ymin=0,ymax=10,
x=1cm,
y=1cm,
axis x line=middle,
axis y line=middle,
axis line style=->,
xlabel={$x$},
ylabel={$f(x)$},
xmin=0,xmax=9
]
\addplot[no marks, black, -] expression[domain=0:9,samples=100]{1/x} node[pos=0.65,anchor=north]{};
\addplot[no marks, black, -] expression[domain=1:2,samples=100]{1/2} node[pos=0.65,anchor=north]{};
\draw[>=stealth] (axis cs:1,0.5) -- (axis cs:1,0) node [pos=0.65,anchor=north]{};
\draw[>=stealth] (axis cs:2,0.5) -- (axis cs:2,0) node [pos=0.65,anchor=north]{};
\addplot[no marks, black, -] expression[domain=2:3,samples=100]{1/3} node[pos=0.65,anchor=north]{};
\draw[>=stealth] (axis cs:3,0.3333333333333) -- (axis cs:3,0) node [pos=0.65,anchor=north]{};
\addplot[no marks, black, -] expression[domain=3:4,samples=100]{1/4} node[pos=0.65,anchor=north]{};
\draw[>=stealth] (axis cs:4,0.25) -- (axis cs:4,0) node [pos=0.65,anchor=north]{};
\addplot[no marks, black, -] expression[domain=4:5,samples=100]{1/5} node[pos=0.65,anchor=north]{};
\draw[>=stealth] (axis cs:5,0.2) -- (axis cs:5,0) node [pos=0.65,anchor=north]{};
\addplot[no marks, black, -] expression[domain=5:6,samples=100]{1/6} node[pos=0.65,anchor=north]{};
\draw[>=stealth] (axis cs:6,0.1666666667) -- (axis cs:6,0) node [pos=0.65,anchor=north]{};
\addplot[no marks, black, -] expression[domain=6:7,samples=100]{1/7} node[pos=0.65,anchor=north]{};
\draw[>=stealth] (axis cs:7,0.142857142857) -- (axis cs:7,0) node [pos=0.65,anchor=north]{};
\addplot[no marks, black, -] expression[domain=7:8,samples=100]{1/8} node[pos=0.65,anchor=north]{};
\draw[>=stealth] (axis cs:8,0.125) -- (axis cs:8,0) node [pos=0.65,anchor=north]{};
\addplot[no marks, black, -] expression[domain=8:9,samples=100]{1/9} node[pos=0.65,anchor=north]{};
\draw[>=stealth] (axis cs:9,0.111111111) -- (axis cs:9,0) node [pos=0.65,anchor=north]{};
%\addplot[no marks, black, -] expression[domain=9:10,samples=100]{1/10} node[pos=0.65,anchor=north]{};
%\draw[>=stealth] (axis cs:10,0.1) -- (axis cs:10,0) node [pos=0.65,anchor=north]{};
\node at (axis cs: 1.5,0.75) {$\frac{1}{2}$};
\node at (axis cs: 2.5,0.5833333) {$\frac{1}{3}$};
\node at (axis cs: 3.5,0.5) {$\frac{1}{4}$};
\node at (axis cs: 4.5,0.45) {$\frac{1}{5}$};
\node at (axis cs: 5.5,0.4166667) {$\frac{1}{6}$};
\node at (axis cs: 6.5,0.392857142857) {$\frac{1}{7}$};
\node at (axis cs: 7.5,0.375) {$\frac{1}{8}$};
\node at (axis cs: 8.5,0.36111111) {$\frac{1}{9}$};
%\node at (axis cs: 9.5,0.35) {$\frac{1}{10}$};
\end{axis}
\end{tikzpicture}\\
Anhand der Skizze erkennt man:\\
\begin{alignat*}{3}
H_{n} - 1 &\leq & \int\limits_{1}^{n} \frac{1}{x}\,dx &=& [\ln x]_{1}^{n} \\
&\leq & &=& \ln (n) - \ln 1 \\
&\leq & &=& \ln (n)
\end{alignat*}
\begin{alignat*}{3}
\intertext{Daraus ergibt sich:}
H_{n} - 1 &\leq & \ln(n) &\leq & H_{n}
\end{alignat*}
\subsection{} %b
\begin{alignat*}{3}
\intertext{Aus a) ergibt sich:}
\lim\limits_{n \rightarrow \infty} \frac{H_{n}-1}{H_{n}} &\leq & \lim\limits_{n \rightarrow \infty} \frac{\ln(n)}{H_{n}} &\leq & \lim\limits_{n \rightarrow \infty} \frac{H_{n}}{H_{n}} \\
\lim\limits_{n \rightarrow \infty} \frac{H_{n}}{H_{n}} - \frac{1}{H_{n}} &\leq & \lim\limits_{n \rightarrow \infty} \frac{\ln(n)}{H_{n}} &\leq & 1 \\
1 - 0 &\leq & \lim\limits_{n \rightarrow \infty} \frac{\ln(n)}{H_{n}} &\leq & 1 \\
1 &\leq & \lim\limits_{n \rightarrow \infty} \frac{\ln(n)}{H_{n}} &\leq & 1
\intertext{Nach dem Einschließungssatz muss der Grenzwert von $\lim\limits_{n \rightarrow \infty} \frac{\ln(n)}{H_{n}}$ $1$ sein.}
\end{alignat*}
\end{document}

View File

@ -0,0 +1,185 @@
\documentclass[10pt,a4paper,oneside,ngerman,numbers=noenddot]{scrartcl}
\usepackage[T1]{fontenc}
\usepackage[utf8]{inputenc}
\usepackage[ngerman]{babel}
\usepackage{amsmath}
\usepackage{amsfonts}
\usepackage{amssymb}
\usepackage{paralist}
\usepackage{gauss}
\usepackage{pgfplots}
\usepackage[locale=DE,exponent-product=\cdot,detect-all]{siunitx}
\usepackage{tikz}
\usetikzlibrary{matrix,fadings,calc,positioning,decorations.pathreplacing,arrows,decorations.markings}
\usepackage{polynom}
\polyset{style=C, div=:,vars=x}
\pgfplotsset{compat=1.8}
\pagenumbering{arabic}
\def\thesection{\arabic{section})}
\def\thesubsection{\alph{subsection})}
\def\thesubsubsection{(\roman{subsubsection})}
\makeatletter
\renewcommand*\env@matrix[1][*\c@MaxMatrixCols c]{%
\hskip -\arraycolsep
\let\@ifnextchar\new@ifnextchar
\array{#1}}
\makeatother
\begin{document}
\author{Jim Martens (6420323)}
\title{Hausaufgaben zum 20. Juni}
\maketitle
\section{} %1
\subsection{} %a
\begin{alignat*}{2}
T_{8}(x) &=&& 1 - \frac{1}{2}x^{2} + \frac{1}{24}x^{4} - \frac{1}{720}x^{6} + \frac{1}{40320}x^{8} \\
T_{9}(x) &=&& 1 - \frac{1}{2}x^{2} + \frac{1}{24}x^{4} - \frac{1}{720}x^{6} + \frac{1}{40320}x^{8} \\
T_{10}(x) &=&& 1 - \frac{1}{2}x^{2} + \frac{1}{24}x^{4} - \frac{1}{720}x^{6} + \frac{1}{40320}x^{8} - \frac{1}{10!}x^{10}\\
T_{11}(x) &=&& 1 - \frac{1}{2}x^{2} + \frac{1}{24}x^{4} - \frac{1}{720}x^{6} + \frac{1}{40320}x^{8} - \frac{1}{10!}x^{10} \\
T_{12}(x) &=&& 1 - \frac{1}{2}x^{2} + \frac{1}{24}x^{4} - \frac{1}{720}x^{6} + \frac{1}{40320}x^{8} - \frac{1}{10!}x^{10} + \frac{1}{12!}x^{12}\\
T_{13}(x) &=&& 1 - \frac{1}{2}x^{2} + \frac{1}{24}x^{4} - \frac{1}{720}x^{6} + \frac{1}{40320}x^{8} - \frac{1}{10!}x^{10} + \frac{1}{12!}x^{12}
\end{alignat*}
\begin{alignat*}{2}
T_{9}(1) &=& 1 - \frac{1}{2} \cdot 1^{2} + \frac{1}{24} \cdot 1^{4} - \frac{1}{720} \cdot 1^{6} + \frac{1}{40320} \cdot 1^{8} \\
&\approx & 0.5403026 \\
T_{11}(1) &=& 1 - \frac{1}{2} \cdot 1^{2} + \frac{1}{24} \cdot 1^{4} - \frac{1}{720} \cdot 1^{6} + \frac{1}{40320} \cdot 1^{8} - \frac{1}{10!} \cdot 1^{10} \\
&\approx & 0.5403023 \\
T_{13}(1) &=& 1 - \frac{1}{2} \cdot 1^{2} + \frac{1}{24} \cdot 1^{4} - \frac{1}{720} \cdot 1^{6} + \frac{1}{40320} \cdot 1^{8} - \frac{1}{10!} \cdot 1^{10} + \frac{1}{12!} \cdot 1^{12} \\
&\approx & 0.5403023
\end{alignat*}
\subsection{} %b
\begin{alignat*}{2}
f(x) &=& \sqrt{1+x} = (1+x)^{\frac{1}{2}} \\
f'(x) &=& \frac{1}{2} \cdot (1+x)^{-\frac{1}{2}} \\
f''(x) &=& -\frac{1}{4} \cdot (1+x)^{-\frac{3}{2}} \\
f'''(x) &=& \frac{3}{8} \cdot (1+x)^{-\frac{5}{2}} \\
f^{(4)}(x) &=& -\frac{15}{16} \cdot (1+x)^{-\frac{7}{2}}
\end{alignat*}
\begin{alignat*}{2}
T_{0}(x) &=&& 1 \\
T_{1}(x) &=&& 1 + \frac{1}{2}x \\
T_{2}(x) &=&& 1 + \frac{1}{2}x - \frac{1}{8}x^{2}\\
T_{3}(x) &=&& 1 + \frac{1}{2}x - \frac{1}{8}x^{2} + \frac{3}{48}x^{3} \\
T_{4}(x) &=&& 1 + \frac{1}{2}x - \frac{1}{8}x^{2} + \frac{3}{48}x^{3} - \frac{15}{384}x^{4}
\end{alignat*}
\begin{alignat*}{2}
g(x) &=& \sqrt[3]{1+x} = (1+x)^{\frac{1}{3}} \\
g'(x) &=& \frac{1}{3} \cdot (1+x)^{-\frac{2}{3}} \\
g''(x) &=& -\frac{2}{9} \cdot (1+x)^{-\frac{5}{3}} \\
g'''(x) &=& \frac{10}{27} \cdot (1+x)^{-\frac{8}{3}} \\
g^{(4)}(x) &=& -\frac{80}{81} \cdot (1+x)^{-\frac{11}{3}}
\end{alignat*}
\begin{alignat*}{2}
T_{0}(x) &=&& 1 \\
T_{1}(x) &=&& 1 + \frac{1}{3}x \\
T_{2}(x) &=&& 1 + \frac{1}{3}x - \frac{1}{9}x^{2} \\
T_{3}(x) &=&& 1 + \frac{1}{3}x - \frac{1}{9}x^{2} + \frac{5}{81}x^{3} \\
T_{4}(x) &=&& 1 + \frac{1}{3}x - \frac{1}{9}x^{2} + \frac{5}{81}x^{3} - \frac{10}{243}x^{4}
\end{alignat*}
\subsection{} %c
\begin{alignat*}{2}
f(x) &=& e^{x} \cdot \sin x \\
f'(x) &=& e^{x} \cdot \cos x \\
f''(x) &=& -e^{x} \cdot \sin x \\
f'''(x) &=& -e^{x} \cdot \cos x \\
f^{(4)}(x) &=& e^{x} \cdot \sin x \\
f^{(5)}(x) &=& e^{x} \cdot \cos x
\end{alignat*}
\begin{alignat*}{2}
T_{5}(x) &=& 0 + x - 0x^{2} - \frac{1}{6}x^{3} + 0x^{4} + \frac{1}{120}x^{5} \\
&=& x - \frac{1}{6}x^{3} + \frac{1}{120}x^{5}
\end{alignat*}
\section{} %2
\subsubsection{} %i
\begin{alignat*}{2}
\lim\limits_{x \rightarrow 1} \left(\frac{x^{3}-3x^{2}+x+2}{x^{2}-5x+6} \right) &=& \frac{1-3+1+2}{1-5+6} \\
&=& \frac{1}{2}
\end{alignat*}
\subsubsection{} %ii
\begin{alignat*}{3}
\lim\limits_{x \rightarrow 2} \left(\frac{x^{3}-3x^{2}+x+2}{x^{2}-5x+6} \right) &=& \lim\limits_{x \rightarrow 2} \left(\frac{3x^{2} - 6x + 1}{2x - 5} \right) &=& \frac{3 \cdot 2^{2} - 6 \cdot 2 + 1}{2 \cdot 2 - 5} \\
&& &=& \frac{12 - 12 + 1}{4 - 5} \\
&& &=& \frac{1}{-1} \\
&& &=& -1
\end{alignat*}
\subsubsection{} %iii
\begin{alignat*}{2}
\lim\limits_{x \rightarrow 0} (1+3x)^{\frac{1}{2x}}
\end{alignat*}
\subsubsection{} %iv
\begin{alignat*}{2}
\lim\limits_{x \rightarrow 0} \left(\frac{1}{e^{x} - 1} - \frac{1}{\sin x} \right)
\end{alignat*}
\section{} %3
\subsection{} %a
\begin{alignat*}{2}
f(x) &=& 3^{x} \\
f'(x) &=& 3^{x} \cdot \ln 3 \\
t'(x) = f'(2) &=& 9 \cdot \ln 3 \\
&\approx & 9.88751 \\
\intertext{Die Steigung der Tangente beträgt $9 \cdot \ln 3$ oder rund $9.88751$.}
t(x) &=& \ln (3) \cdot 9x + b \\
\intertext{Bestimmen des Schnittpunkts mit der y-Achse}
b &=& t(x) - \ln (3) \cdot 9x \\
&=& t(2) - \ln (3) \cdot 18 \\
&=& 9 - \ln (3) \cdot 18 \\
&\approx & -10.77502 \\
t(x) &=& \ln 3 \cdot 9x + 9 - \ln (3) \cdot 18 \\
\intertext{Bestimmen des Schnittpunkts mit der x-Achse}
0 &=& \ln (3) \cdot 9x + 9 - \ln (3) \cdot 18 \\
\ln (3) \cdot 18 - 9 &=& \ln (3) \cdot 9x \\
\frac{\ln (3) \cdot 18 - 9}{\ln 3} &=& 9x \\
\frac{\ln (3) \cdot 18 - 9}{\ln (3) \cdot 9} &=& x \\
x &\approx & 1.08976
\end{alignat*}
\subsection{} %b
\begin{alignat*}{2}
f(x) &=& \sqrt[7]{x+2} \\
&=& (x+2)^{\frac{1}{7}} \\
f'(x) &=& \frac{1}{7} \cdot (x+2)^{-\frac{6}{7}} \\
f''(x) &=& -\frac{6}{49} \cdot (x+2)^{-\frac{13}{7}}
\end{alignat*}
\begin{alignat*}{2}
T_{0} &=&& 2^{\frac{1}{7}} \\
T_{1} &=&& 2^{\frac{1}{7}} + \frac{1}{7} \cdot 2^{-\frac{6}{7}}x \\
T_{2} &=&& 2^{\frac{1}{7}} + \frac{1}{7} \cdot 2^{-\frac{6}{7}}x - \frac{3}{49} \cdot 2^{-\frac{13}{7}}
\end{alignat*}
\subsection{} %c
\begin{alignat*}{2}
x_{n} &=& \frac{1}{2\pi n} \\
\lim\limits_{n\rightarrow \infty} f(x_{n}) = \lim\limits_{n\rightarrow \infty} \cos \left(\frac{1}{x_{n}} \right) &=& \lim\limits_{n\rightarrow \infty} \cos \left(\frac{1}{\frac{1}{2 \pi n}} \right) \\
&=& \lim\limits_{n\rightarrow \infty} \cos (2 \pi n) = 1
\end{alignat*} \\
Der mögliche Grenzwert 1 stimmt nicht mit dem Funktionswert überein. Daher ist $h(x)$ im Punkt $x_{0}=0$ nicht stetig.
\subsection{} %d
\setcounter{subsubsection}{0}
\subsubsection{} %i
$\mathcal{B}$ enthält alle Folgen, die gegen eine reelle Zahl konvergieren, als auch solche, die zwischen zwei Werten oszillieren.
Lediglich uneigentlich konvergente Folgen sind nicht enthalten.
\subsubsection{} %ii
Aus $i)$ ergibt sich, dass nicht jede Folge in $\mathcal{B}$ konvergiert. Oszillierende Folgen konvergieren nicht, allerdings erfüllen sie die Bedingungen von $\mathcal{B}$.
\section{} %4
\subsection{} %a
\begin{alignat*}{2}
\lim\limits_{x \rightarrow \infty} \left(\frac{a^{x}}{x^{n}}\right) &=& \lim\limits_{x \rightarrow \infty} \left(\frac{a^{x} \cdot \ln a}{n \cdot x^{n-1}}\right) \\
\intertext{Nach n Ableitungen}
&=& \lim\limits_{x \rightarrow \infty} \left(\frac{a^{x} \cdot (\ln a)^{n}}{n! \cdot x^{0}}\right) \\
\intertext{Zähler geht gegen unendlich, Nenner gegen konstante Zahl $n!$. Daher existiert der Grenzwert nicht.}
&=& \lim\limits_{x \rightarrow \infty} \left(\frac{a^{x} \cdot (\ln a)^{n}}{n!}\right) = \infty
\end{alignat*}
\subsection{} %b
\begin{alignat*}{2}
\lim\limits_{x \rightarrow \infty} \left(\frac{x^{r}}{\ln^{k} x} \right) &=& \lim\limits_{x \rightarrow \infty} \left(\frac{rx^{r-1}}{k \cdot \ln^{k-1} x \cdot \frac{1}{x}} \right) \\
&=& \lim\limits_{x \rightarrow \infty} \left(\frac{rx^{r}}{k \cdot \ln^{k-1} x} \right) \\
\intertext{Nach der k-ten Ableitung sieht es so aus}
&=& \lim\limits_{x \rightarrow \infty} \left(\frac{r^{k} \cdot x^{r}}{k! \cdot \ln x} \right) \\
\intertext{Herausziehen der Konstanten}
&=& \frac{r^{k}}{k!} \cdot \lim\limits_{x \rightarrow \infty} \left(\frac{x^{r}}{\ln x} \right) \\
\intertext{Nach Satz 27 im Skript, existiert der Grenzwert $\lim\limits_{x \rightarrow \infty} \left(\frac{x^{r}}{\ln x} \right)$ nicht bzw. ist unendlich. Unendlich mal eine Konstante ist immer noch unendlich.}
&=& \infty
\end{alignat*}
\subsection{} %c
\setcounter{subsubsection}{0}
\subsubsection{} %i
\subsubsection{} %ii
\end{document}